Вы находитесь на странице: 1из 125

https://telegram.me/UPSCMaterials https://telegram.me/FreeUPSCMaterials https://telegram.

me/MaterialforExam

2019 - Test 2-Geography


Exam Title :
& C...
Email
Contact :

The second force that was suggested by Wegener — the tidal force is due to the attraction
of the moon and the sun that develops tides in oceanic waters. Wegener believed that
these forces would become effective when applied over many million years. However, most of
scholars considered these forces to be totally inadequate.

It is interesting to note that for continental drift, most of the evidence was collected from the
continental areas in the form of distribution of flora and fauna or deposits, like tillite. A number
of discoveries during the post–World War II period added new information to geological
literature. Particularly, the information collected from the ocean floor mapping provided
new dimensions for the study of distribution of oceans and continents .

QUESTION 12. MTU5Mzg0K0pheStqYXlhZGhlZXIuc3llQGdtYWlsLmNvbStRVUVTVElPTiAxMQ==


Arrange the following elements present in the earth’s crust in the decreasing order of their
composition

1. Oxygen

2. Aluminium

3. Silicon

4. Iron

5. Calcium

Choose the correct code

a) 3>2>1>5>4
b) 1>3>2>4>5
c) 1>2>4>3>5
d) 4>1>2>5>3
Correct Answer: B
Answer
Explanation

Solution (b)

The earth is composed of various kinds of elements. These elements are in solid form in the
outer layer of the earth and in hot and molten form in the interior .

About 98 per cent of the total crust of the earth is composed of eight elements like oxygen,
silicon, aluminium, iron, calcium, sodium, potassium and magnesium and the rest is
constituted by titanium, hydrogen, phosphorous, manganese, sulphur, carbon, nickel and other
elements.

The elements in the earth’s crust are rarely found exclusively but are usually combined with
other elements to make various substances . These substances are recognised as minerals

IASbaba
Score:
Web:
Email:
Page 13 150
https://telegram.me/UPSCMaterials https://telegram.me/FreeUPSCMaterials https://telegram.me/MaterialforExam

2019 - Test 2-Geography


Exam Title :
& C...
Email
Contact :

QUESTION 13. MTU5Mzg0K0pheStqYXlhZGhlZXIuc3llQGdtYWlsLmNvbStRVUVTVElPTiAxMg==


A certain mineral has the following characteristics.

1. Half of the earth’s crust is composed of this mineral.

2. It has light cream to salmon pink colour.

3. It is mainly used in ceramics and glass making.

Choose the correct mineral

a) Feldspar
b) Quartz
c) Mica
d) Amphibole
Correct Answer: A
Answer
Explanation

Solution (a)

Some major minerals and their characteristics

Feldspar

Silicon and oxygen are common elements in all types of feldspar and sodium, potassium,
calcium, aluminium etc. are found in specific feldspar variety . Half of the earth’s crust is
composed of feldspar. It has light cream to salmon pink colour . It is used in ceramics and
glass making.

Quartz

It is one of the most important components of sand and granite. It consists of silica. It is a
hard mineral virtually insoluble in water. It is white or colourless and used in radio and radar.
It is one of the most important components of granite.

Pyroxene

IASbaba
Score:
Web:
Email:
Page 14 150
https://telegram.me/UPSCMaterials https://telegram.me/FreeUPSCMaterials https://telegram.me/MaterialforExam

2019 - Test 2-Geography


Exam Title :
& C...
Email
Contact :

Pyroxene consists of calcium, aluminum, magnesium, iron and silica. Pyroxene forms 10 per ce
nt of the earth’s crust. It is commonly found in meteorites . It is in green or black colour.

Amphibole

Aluminium, calcium, silica, iron, magnesium are the major elements of amphiboles. They form
7 per cent of the earth’s crust . It is in green or black colour and is used in asbestos
industry . Hornblende is another form of amphiboles.

Mica

It comprises of potassium, aluminium, magnesium, iron, silica etc. It forms 4 per cent of
the earth’s crust. It is commonly found in igneous and metamorphic rocks . It is used in elec
trical instruments .

QUESTION 14. MTU5Mzg0K0pheStqYXlhZGhlZXIuc3llQGdtYWlsLmNvbStRVUVTVElPTiAxMw==


Which of the following is an example of Igneous rocks?

a) Granite
b) Pegmatite
c) Tuff
d) All of the above
Correct Answer: D
Answer
Explanation

Solution (d)

Igneous rocks form out of magma and lava from the interior of the earth , they are known
as primary rocks . The igneous rocks (Ignis – in Latin means ‘Fire’) are formed when magma
cools and solidifies. When magma in its upward movement cools and turns into solid form it is
called igneous rock.

The process of cooling and solidification can happen in the earth’s crust or on the surface
of the earth.

Igneous rocks are classified based on texture . Texture depends upon size and arrangement
of grains or other physical conditions of the materials. If molten material is cooled slowly at
great depths, mineral grains may be very large. Sudden cooling (at the surface) results in small
and smooth grains. Intermediate conditions of cooling would result in intermediate sizes of
grains making up igneous rocks.

Granite, gabbro, pegmatite, basalt, volcanic breccia and tuff are some of the examples
of igneous rocks .

QUESTION 15. MTU5Mzg0K0pheStqYXlhZGhlZXIuc3llQGdtYWlsLmNvbStRVUVTVElPTiAxNA==


Which of the following is the softest mineral?

IASbaba
Score:
Web:
Email:
Page 15 150
https://telegram.me/UPSCMaterials https://telegram.me/FreeUPSCMaterials https://telegram.me/MaterialforExam

2019 - Test 2-Geography


Exam Title :
& C...
Email
Contact :

a) Talc
b) Gypsum
c) Corundum
d) Calcite
Correct Answer: A
Answer
Explanation

Solution (a)

Hardness refers to relative resistance when being scratched.

Ten minerals are selected to measure the degree of hardness from 1-10.

They are: 1. talc; 2. gypsum; 3. calcite; 4. fluorite; 5. apatite; 6. feldspar; 7. quartz; 8. topaz; 9.
corundum; 10. diamond.

Compared to this for example, a fingernail is 2.5 and glass or knife blade is 5.5 .

QUESTION 16. MTU5Mzg0K0pheStqYXlhZGhlZXIuc3llQGdtYWlsLmNvbStRVUVTVElPTiAxNQ==


Consider the following statements about the different forces acting on the Earth.

1. The endogenic forces are mainly land building forces and the exogenic processes are mainly
land wearing forces.

2. Diastrophism and weathering are examples of exogenic geomorphic processes.

Choose the correct code

a) 1 only
b) 2 only
c) Both 1 and 2
d) None of the above
Correct Answer: A
Answer
Explanation

Solution (a)

The earth’s crust is dynamic. The differences in the internal forces operating from within the
earth which built up the crust have been responsible for the variations in the outer surface of
the crust. The earth’s surface is being continuously subjected to external forces induced
basically by energy (sunlight)

The earth’s surface is being continuously subjected to by external forces originating within the
earth’s atmosphere and by internal forces from within the earth. The external forces are
known as exogenic forces and the internal forces are known as endogenic forces . The
actions of exogenic forces result in wearing down (degradation) of relief/elevations and filling
up (aggradation) of basins/ depressions, on the earth’s surface.

IASbaba
Score:
Web:
Email:
Page 16 150
https://telegram.me/UPSCMaterials https://telegram.me/FreeUPSCMaterials https://telegram.me/MaterialforExam

2019 - Test 2-Geography


Exam Title :
& C...
Email
Contact :

The phenomenon of wearing down of relief variations of the surface of the earth through
erosion is known as gradation.

Do you know?

The endogenic forces continuously elevate or build up parts of the earth’s surface and
hence the exogenic processes fail to even out the relief variations of the surface of the earth.
So, variations remain as long as the opposing actions of exogenic and endogenic forces
continue. In general terms, the endogenic forces are mainly land building forces and the
exogenic processes are mainly land wearing forces.

Diastrophism and volcanism are endogenic geomorphic processes.

Weathering, mass wasting, erosion and deposition are exogenic geomorphic processes.

QUESTION 17. MTU5Mzg0K0pheStqYXlhZGhlZXIuc3llQGdtYWlsLmNvbStRVUVTVElPTiAxNg==


Which of the following is an example of physical weathering?

a) Hydration
b) Exfoliation
c) Carbonation
d) Oxidation and reduction
Correct Answer: B
Answer
Explanation

Solution (b)

Physical or mechanical weathering processes depend on some applied forces. The applied
forces could be:

· Gravitational forces such as overburden pressure, load and shearing stress, exfoliation.

· Expansion forces due to temperature changes, crystal growth or animal activity.

· Water pressures controlled by wetting and drying cycles.

Many of these forces are applied both at the surface and within different earth materials
leading to rock fracture. Most of the physical weathering processes are caused by thermal
expansion and pressure release.

These processes are small and slow but can cause great damage to the rocks because of
continued fatigue the rocks suffer due to repetition of contraction and expansion.

Solution, carbonation, hydration and oxidation and reduction are examples of chemical
weathering processes.

QUESTION 18. MTU5Mzg0K0pheStqYXlhZGhlZXIuc3llQGdtYWlsLmNvbStRVUVTVElPTiAxNw==


Waterfalls and rapids are associated with which stage of the river?

IASbaba
Score:
Web:
Email:
Page 17 150
https://telegram.me/UPSCMaterials https://telegram.me/FreeUPSCMaterials https://telegram.me/MaterialforExam

2019 - Test 2-Geography


Exam Title :
& C...
Email
Contact :

a) Youth stage
b) Mature stage
c) Old stage
d) All of the above
Correct Answer: A
Answer
Explanation

Solution (a)

In humid regions, which receive heavy rainfall running water is considered the most important
of the geomorphic agents in bringing about the degradation of the land surface. There are two
components of running water. One is overland flow on general land surface as a sheet. Another
is linear flow as streams and rivers in valleys.

The characteristics of each of the stages of landscapes developing in running water regimes are
as follows

Youth

Streams are few during this stage with poor integration and flow over original slopes showing
shallow V-shaped valleys with no floodplains or with very narrow floodplains along trunk
streams. Streams divides are broad and flat with marshes, swamp and lakes. Meanders if
present develop over these broad upland surfaces. These meanders may eventually entrench
themselves into the uplands. Waterfalls and rapids may exist where local hard rock bodies
are exposed.

Mature

During this stage streams are plenty with good integration. The valleys are still V-shaped but
deep; trunk streams are broad enough to have wider floodplains within which streams may flow
in meanders confined within the valley. The flat and broad inter stream areas and swamps and
marshes of youth disappear and the stream divides turn sharp. Waterfalls and rapids
disappear.

Old

Smaller tributaries during old age are few with gentle gradients. Streams meander freely over
vast floodplains showing natural levees, oxbow lakes, etc. Divides are broad and flat with lakes,
swamps and marshes. Most of the landscape is at or slightly above sea level.

QUESTION 19. MTU5Mzg0K0pheStqYXlhZGhlZXIuc3llQGdtYWlsLmNvbStRVUVTVElPTiAxOA==


Which of the following is a depositional landform formed by glacier?

a) Deltas
b) Moraines
c) Cirque
d) Arete
Correct Answer: B
Answer
Explanation

IASbaba
Score:
Web:
Email:
Page 18 150
https://telegram.me/UPSCMaterials https://telegram.me/FreeUPSCMaterials https://telegram.me/MaterialforExam

2019 - Test 2-Geography


Exam Title :
& C...
Email
Contact :

Solution (b)

Masses of ice moving as sheets over the land (continental glacier or piedmont glacier if a vast
sheet of ice is spread over the plains at the foot of mountains) or as linear flows down the slopes
of mountains in broad trough-like valleys (mountain and valley glaciers) are called glaciers.

A glacier in its valley is slow unlike water flow. The movement could be a few centimetres to
a few metres a day or even less or more. Glaciers move basically because of the force of
gravity.

Erosion by glaciers is tremendous because of friction caused by sheer weight of the ice .
The material plucked from the land by glaciers (usually large-sized angular blocks and
fragments) get dragged along the floors or sides of the valleys and cause great damage through
abrasion and plucking.

Glaciers can cause significant damage to even un-weathered rocks and can reduce high
mountains into low hills and plains. As glaciers continue to move, debris gets removed, divides
get lowered and eventually the slope is reduced to such an extent that glaciers will stop moving
leaving only a mass of low hills and vast outwash plains along with other depositional features.

Delta is a depositional landform formed by running river not glacier.

Erosional landforms formed by glaciers

1) Cirque

2) Horns and serrated ridges

3) Glacial valleys/troughs

Depositional landforms formed by glaciers

1) Moraines

2) Eskers

3) Outwash plains

4) Drumlins

QUESTION 20. MTU5Mzg0K0pheStqYXlhZGhlZXIuc3llQGdtYWlsLmNvbStRVUVTVElPTiAxOQ==


Which of the following act as the ideal conditions to speed up chemical weathering process?

a) Hot and humid


b) Hot and dry
c) Cold and humid
d) None of the above
Correct Answer: A
Answer
Explanation

Solution (a)

IASbaba
Score:
Web:
Email:
Page 19 150
https://telegram.me/UPSCMaterials https://telegram.me/FreeUPSCMaterials https://telegram.me/MaterialforExam

2019 - Test 2-Geography


Exam Title :
& C...
Email
Contact :

Weathering processes are conditioned by many complex geological, climatic, topographic


and vegetative factors . Climate is of particular importance.

Not only weathering processes differ from climate to climate, but also the depth of the
weathering mantle.

Chemical Weathering

Processes A group of weathering processes namely solution, carbonation, hydration, oxidation


and reduction act on the rocks to decompose, dissolve or reduce them to a fine clastic state
through chemical reactions by oxygen, surface and/or soil water and other acids.

Water and air (oxygen and carbon dioxide) along with heat must be present to speed up
all chemical reactions. Over and above the carbon dioxide present in the air, decomposition of
plants and animals increases the quantity of carbon dioxide underground.

These chemical reactions on various minerals are very much similar to the chemical reactions
in a laboratory.

QUESTION 21. MTU5Mzg0K0pheStqYXlhZGhlZXIuc3llQGdtYWlsLmNvbStRVUVTVElPTiAyMA==


Which of the following best describes the process of foliation?

a) It is a layered arrangement of minerals or grains in metamorphic rocks due to deformation


caused by tectonic shearing together with high temperature or pressure or both.
b) It is an arrangement of minerals or grains in igneous rocks due to deformation caused by
tectonic shearing together with high temperature or pressure or both.
c) It is an arrangement of minerals or grains in sedimentary rocks due to deformation caused
by tectonic shearing together with high temperature or pressure or both.
d) It is a process in which minerals or materials of different groups are arranged into circular
crystals in igneous rocks.
Correct Answer: A
Answer
Explanation

Solution (a)

Mechanical disruption and reorganisation of the original minerals within rocks due to breaking
and crushing without any appreciable chemical changes is called dynamic metamorphism.

The materials of rocks chemically alter and recrystallize due to thermal metamorphism. There
are two types of thermal metamorphism — contact metamorphism and regional
metamorphism. In contact metamorphism the rocks come in contact with hot intruding
magma and lava and the rock materials recrystallize under high temperatures.

Quite often new materials form out of magma or lava are added to the rocks. In regional
metamorphism, rocks undergo recrystallization due to deformation caused by tectonic
shearing together with high temperature or pressure or both . In the process of
metamorphism in some rocks, grains or minerals get arranged in layers or lines. Such an
arrangement of minerals or grains in metamorphic rocks is called foliation or
lineation . Sometimes minerals or materials of different groups are arranged into alternating

IASbaba
Score:
Web:
Email:
Page 20 150
https://telegram.me/UPSCMaterials https://telegram.me/FreeUPSCMaterials https://telegram.me/MaterialforExam

2019 - Test 2-Geography


Exam Title :
& C...
Email
Contact :
thin to thick layers appearing in light and dark shades. Such a structure in metamorphic
rocks is called banding and rocks displaying banding are called banded rocks.

QUESTION 22. MTU5Mzg0K0pheStqYXlhZGhlZXIuc3llQGdtYWlsLmNvbStRVUVTVElPTiAyMQ==


The boundary between Indian and the Antarctic plate is marked by oceanic ridge. What type of
plate boundary makes it possible?

a) Continent-Continent convergence
b) Continent-Oceanic convergence
c) Continent-Oceanic divergence
d) Oceanic-Oceanic divergence
Correct Answer: D
Answer
Explanation

Solution (d)

The Indian plate includes Peninsular India and the Australian continental portions. The
subduction zone along the Himalayas forms the northern plate boundary in the form of contine
nt— continent convergence . In the east, it extends through Rakinyoma Mountains of
Myanmar towards the island arc along the Java Trench. The eastern margin is a spreading site
lying to the east of Australia in the form of an oceanic ridge in SW Pacific. The Western margin
follows Kirthar Mountain of Pakistan. It further extends along the Makrana coast and joins the
spreading site from the Red Sea rift south-eastward along the Chagos Archipelago.

The boundary between Indian and the Antarctic plate is also marked by oceanic ridge
(divergent boundary) running in roughly W-E direction and merging into the spreading site, a
little south of New Zealand.

QUESTION 23. MTU5Mzg0K0pheStqYXlhZGhlZXIuc3llQGdtYWlsLmNvbStRVUVTVElPTiAyMg==


Consider the following statements about earthquake waves.

1. Body waves are generated due to the release of energy at the focus and are considered to be
the most damaging waves.

2. P waves are similar to sound waves, as they travel through gaseous, Solid and liquid
mediums.

3. The shadow zone of S-wave is much larger than that of the P-waves.

Choose the correct code

a) 1 and 2 only
b) 2 and 3 only
c) 1 and 3 only
d) All of the above
Correct Answer: B
Answer

IASbaba
Score:
Web:
Email:
Page 21 150
https://telegram.me/UPSCMaterials https://telegram.me/FreeUPSCMaterials https://telegram.me/MaterialforExam

2019 - Test 2-Geography


Exam Title :
& C...
Email
Contact :
Explanation

Solution (b)

Earthquake waves are basically of two types — body waves and surface waves. Body waves
are generated due to the release of energy at the focus and move in all directions travelling
through the body of the earth. Hence, the name body waves.

The body waves interact with the surface rocks and generate new set of waves called surface
waves. These waves move along the surface. The velocity of waves changes as they travel
through materials with different densities. The denser the material, the higher is the
velocity . Their direction also changes as they reflect or refract when coming across materials
with different densities.

There are two types of body waves. They are called P and S-waves. P-waves move faster and
are the first to arrive at the surface. These are also called ‘primary waves’ . The P-waves
are similar to sound waves. They travel through gaseous, liquid and solid materials. S-waves
arrive at the surface with some time lag. These are called secondary waves. An important fact
about S-waves is that they can travel only through solid materials. This characteristic of the S-
waves is quite important. It has helped scientists to understand the structure of the interior of
the earth.

Do you know?

All natural earthquakes take place in the lithosphere. An instrument called ‘seismograph’
records the waves reaching the surface.

Emergence of Shadow Zone

Earthquake waves get recorded in seismographs located at far off locations. However, there
exist some specific areas where the waves are not reported. Such a zone is called the ‘shadow
zone’. The study of different events reveals that for each earthquake, there exists an altogether
different shadow zone.

It was observed that seismographs located at any distance within 105° from the epicentre,
recorded the arrival of both P and S-waves. However, the seismographs located beyond 145°
from epicentre; record the arrival of P-waves, but not that of S-waves. Thus, a zone between
105° and 145° from epicentre was identified as the shadow zone for both the types of waves.
The entire zone beyond 105° does not receive S-waves. The shadow zone of S-wave is much
larger than that of the P-waves.

The shadow zone of P-waves appears as a band around the earth between 105° and 145° away
from the epicentre.

The shadow zone of S-waves is not only larger in extent but it is also a little over 40 per cent of
the earth surface.

QUESTION 24. MTU5Mzg0K0pheStqYXlhZGhlZXIuc3llQGdtYWlsLmNvbStRVUVTVElPTiAyMw==


When subjected to the process of metamorphosis, limestone gets converted into which of the
following rocks?

a) Marble
b) Gneiss

IASbaba
Score:
Web:
Email:
Page 22 150
https://telegram.me/UPSCMaterials https://telegram.me/FreeUPSCMaterials https://telegram.me/MaterialforExam

2019 - Test 2-Geography


Exam Title :
& C...
Email
Contact :

c) Schist
d) Quartzite
Correct Answer: A
Answer
Explanation

Solution (a)

Rocks do not remain in their original form for long but may undergo transformation. Rock cycle
is a continuous process through which old rocks are transformed into new ones. Igneous rocks
are primary rocks and other rocks (sedimentary and metamorphic) form from these primary
rocks.

QUESTION 25. MTU5Mzg0K0pheStqYXlhZGhlZXIuc3llQGdtYWlsLmNvbStRVUVTVElPTiAyNA==


Consider the following statements about Volcanoes

1. These volcanoes outpour highly fluid lava that flows for long distances.

2. The Deccan Traps from India, presently covering most of the Maharashtra plateau are
examples of this volcano.

Which is the volcano that is referred to here?

a) Shield volcano
b) Composite volcano
c) Cone Volcano
d) Caldera
Correct Answer: A
Answer

IASbaba
Score:
Web:
Email:
Page 23 150
https://telegram.me/UPSCMaterials https://telegram.me/FreeUPSCMaterials https://telegram.me/MaterialforExam

2019 - Test 2-Geography


Exam Title :
& C...
Email
Contact :
Explanation

Solution (a)

Flood Basalt Provinces or Shield Volcano - These volcanoes outpour highly fluid lava
that flows for long distances. Some parts of the world are covered by thousands of sq. km of
thick basalt lava flows. There can be a series of flows with some flows attaining thickness of
more than 50 m.

Individual flows may extend for hundreds of km. The Deccan Traps from India, presently
covering most of the Maharashtra plateau, are a much larger flood basalt province.

It is believed that initially the trap formations covered a much larger area than the present.

Do you know?

Composite Volcanoes

These volcanoes are characterised by eruptions of cooler and more viscous lavas than
basalt. These volcanoes often result in explosive eruptions. Along with lava, large quantities
of pyroclastic material and ashes find their way to the ground. This material accumulates in the
vicinity of the vent openings leading to formation of layers, and this makes the mounts appear
as composite volcanoes.

QUESTION 26. MTU5Mzg0K0pheStqYXlhZGhlZXIuc3llQGdtYWlsLmNvbStRVUVTVElPTiAyNQ==


Which of the following best describes Asthenosphere?

a) It is a part of the upper portion of the mantle


b) It is the region extending beyond the mantle portion
c) It is that portion of the earth which includes crust and the upper most solid mantle
d) It is the portion of the interior beyond the crust
Correct Answer: A
Answer
Explanation

Solution (a)

STRUCTURE OF THE EARTH

The Crust

It is the outermost solid part of the earth . It is brittle in nature. The thickness of the crust
varies under the oceanic and continental areas. Oceanic crust is thinner as compared to
the continental crust . It is made up of heavier rocks having density of 3 g/cm3. This type of
rock found in the oceanic crust is basalt. The mean density of material in oceanic crust is 2.7 g/
cm3.

The Mantle

The portion of the interior beyond the crust is called the mantle . The mantle extends
from Moho’s discontinuity to a depth of 2,900 km. The upper portion of the mantle is called
asthenosphere. The word astheno means weak. It is considered to be extending from 200 km to
700 km.(Note different sources have given different depths of asthenosphere. However, this is

IASbaba
Score:
Web:
Email:
Page 24 150
https://telegram.me/UPSCMaterials https://telegram.me/FreeUPSCMaterials https://telegram.me/MaterialforExam

2019 - Test 2-Geography


Exam Title :
& C...
Email
Contact :
the most accepted one.) It is the main source of magma that finds its way to the surface d
uring volcanic eruptions. It has a density higher than the crust’s (3.4 g/cm3) . The crust
and the uppermost solid part of the mantle are called lithosphere. Its thickness ranges from
10-200 km. The lower mantle extends beyond the asthenosphere. It is in solid state .

The Core

The core-mantle boundary is located at the depth of 2,900 km. The outer core is in liquid
state while the inner core is in solid state. The density of material at the mantle core
boundary is around 5 g/cm3 and at the centre of the earth at 6,300 km, the density value is
around 13g/cm3.

The core is made up of very heavy material mostly constituted by nickel and iron. It is
sometimes referred to as the nife layer.

QUESTION 27. MTU5Mzg0K0pheStqYXlhZGhlZXIuc3llQGdtYWlsLmNvbStRVUVTVElPTiAyNg==


Which of the following statements are correct regarding Potential Resources?

1. They are being widely used across the globe.

2. They are present in a very small quantity.

3. They are not being currently used but can be used in future if technology is developed.

Select the code from below:

a) 1 and 2
b) 2 and 3
c) 3 only
d) All of the above
Correct Answer: C
Answer
Explanation

Solution (c)

Potential resources are those whose entire quantity may not be known and these are not being
used at present. These resources could be used in the future.

IASbaba
Score:
Web:
Email:
Page 25 150
https://telegram.me/UPSCMaterials https://telegram.me/FreeUPSCMaterials https://telegram.me/MaterialforExam

2019 - Test 2-Geography


Exam Title :
& C...
Email
Contact :

The level of technology we have at present may not be advanced enough to easily utilise these
resources. The uranium found in Ladakh is an example of potential resource that could be used
in the future. High speed winds were a potential resource two hundred years ago. Today they
are an actual resource and wind farms generate energy using windmills like in Netherlands.
You will find some in Nagercoil in Tamil Nadu and on the Gujarat coast.

QUESTION 28. MTU5Mzg0K0pheStqYXlhZGhlZXIuc3llQGdtYWlsLmNvbStRVUVTVElPTiAyNw==


Mulching is a method of soil conservation. Which of the following statements correctly defines
the process of Mulching?

a) Different crops are grown in alternate rows and are sown at different times to protect the
soil from rain wash.
b) Ploughing parallel to the contours of a hill slope to form a natural barrier for water to flow
down the slope
c) Rows of trees are planted to check the wind movement to protect soil cover
d) The bare ground between plants is covered with a layer of organic matter like straw.
Correct Answer: D
Answer
Explanation

Solution (d)

Soil erosion and depletion are the major threats to soil as a resource. Both human and natural
factors can lead to degradation of soils. Factors which lead to soil degradation are
deforestation, overgrazing, overuse of chemical feritilisers or pesticides, rain wash, landslides
and floods.

Some methods of soil conservation are listed below:

Mulching: The bare ground between plants is covered with a layer of organic matter like stra
w. It helps to retain soil moisture.

Contour barriers : Stones, grass, soil are used to build barriers along contours. Trenches are
made in front of the barriers to collect water.

Rock dam : Rocks are piled up to slow down the flow of water. This prevents gullies and
further soil loss.

Terrace farming : Broad flat steps or terraces are made on the steep slopes so that flat
surfaces are available to grow crops. They reduce surface run-off and soil erosion.

Intercropping : Different crops are grown in alternate rows and are sown at different times to
protect the soil from rain wash.

Contour ploughing : Ploughing parallel to the contours of a hill slope to form a natural barrier
for water to flow down the slope.

Shelter belts : In the coastal and dry regions, rows of trees are planted to check the wind
movement to protect soil cover.

IASbaba
Score:
Web:
Email:
Page 26 150
https://telegram.me/UPSCMaterials https://telegram.me/FreeUPSCMaterials https://telegram.me/MaterialforExam

2019 - Test 2-Geography


Exam Title :
& C...
Email
Contact :

QUESTION 29. MTU5Mzg0K0pheStqYXlhZGhlZXIuc3llQGdtYWlsLmNvbStRVUVTVElPTiAyOA==


Minerals that lie near the surface are simply dug out. This process of extraction of mineral is
called

a) Shaft Mining
b) Drilling
c) Quarrying
d) Fracking
Correct Answer: C
Answer
Explanation

Solution (c)

The process of taking out minerals from rocks buried under the earth’s surface is called mining.
Minerals that lie at shallow depths are taken out by removing the surface layer; this is known as
open cast mining. Deep bores, called shafts, have to be made to reach mineral deposits that lie
at great depths. This is called shaft mining. Petroleum and natural gas occur far below the
earth’s surface. Deep wells are bored to take them out, this is called drilling. Minerals that lie
near the surface are simply dug out, by the process known as quarrying.

QUESTION 30. MTU5Mzg0K0pheStqYXlhZGhlZXIuc3llQGdtYWlsLmNvbStRVUVTVElPTiAyOQ==


Which of the following minerals are referred to as ‘Buried Sunshine’?

a) Coal
b) Copper
c) Mercury
d) Petroleum
Correct Answer: A
Answer
Explanation

Solution (a)

Coal

This is the most abundantly found fossil fuel. It is used as a domestic fuel, in industries such as
iron and steel, steam engines and to generate electricity. Electricity from coal is called thermal
power. The coal which we are using today was formed millions of years ago when giant ferns an
d swamps got buried under the layers of earth. Coal is therefore referred to as Buried Sunshine.

QUESTION 31. MTU5Mzg0K0pheStqYXlhZGhlZXIuc3llQGdtYWlsLmNvbStRVUVTVElPTiAzMA==


Wind is a renewable source of energy. Windmills have been used from immemorial. Which of the
following were done with the help of wind mills?

IASbaba
Score:
Web:
Email:
Page 27 150
https://telegram.me/UPSCMaterials https://telegram.me/FreeUPSCMaterials https://telegram.me/MaterialforExam

2019 - Test 2-Geography


Exam Title :
& C...
Email
Contact :

1. Grinding of grains

2. Lifting water

3. Generation of electricity

4. Providing wind to people during summers

Select the code from following:

a) 1,2 and 3
b) 2,3 and 4
c) 1,3 and 4
d) All of the above
Correct Answer: A
Answer
Explanation

Solution (a)

Wind Energy

Wind is an inexhaustible source of energy. Wind mills have been used for grinding grain and
lifting water since times immemorial. In modern time wind mills, the high speed winds rotate
the wind mill which is connected to a generator to produce electricity. Wind farms having
clusters of such wind mills are located in coastal regions and in mountain passes where strong
and steady winds blow. Windfarms are found in Netherlands, Germany, Denmark, UK, USA and
Spain are noted for their wind energy production.

India's southern state of Tamil Nadu is a world leader when it comes to renewable energy . Its
wind turbines have a combined installed capacity of 7.9 gigawatts (GW).

QUESTION 32. MTU5Mzg0K0pheStqYXlhZGhlZXIuc3llQGdtYWlsLmNvbStRVUVTVElPTiAzMQ==


Geothermal energy of the earth is due to

1. Radioactive decay of elements in the earth’s interior

2. Primordial heat of earth

3. Absorption of solar radiation

Which of the above statements are correct?

a) 1 and 2
b) 2 and 3
c) 1 and 3
d) All of the above
Correct Answer: A
Answer
Explanation

Solution (a)

IASbaba
Score:
Web:
Email:
Page 28 150
https://telegram.me/UPSCMaterials https://telegram.me/FreeUPSCMaterials https://telegram.me/MaterialforExam

2019 - Test 2-Geography


Exam Title :
& C...
Email
Contact :

Geothermal Energy

Geothermal energy is the heat from the Earth. It's clean and sustainable. Resources of
geothermal energy range from the shallow ground to hot water and hot rock found a few miles
beneath the Earth's surface, and down even deeper to the extremely high temperatures of
molten rock called magma.

The geothermal energy of the Earth's crust originates from the original formation of the planet
and from radioactive decay of materials (in currently uncertain but possibly roughly equal
proportions). The geothermal gradient, which is the difference in temperature between the core
of the planet and its surface, drives a continuous conduction of thermal energy in the form of
heat from the core to the surface.

QUESTION 33. MTU5Mzg0K0pheStqYXlhZGhlZXIuc3llQGdtYWlsLmNvbStRVUVTVElPTiAzMg==


Which of the following statements regarding ‘Streak’ (in relation to minerals) are correct?

1. Colour of the ground powder of any mineral is called Streak.

2. Streaks may be of same colour as the mineral or different.

Select the code from following

a) 1 only
b) 2 only
c) Both 1 and 2
d) Neither 1 nor 2
Correct Answer: C
Answer
Explanation

Solution (c)

Streak - It refers to the colour of the ground powder of any mineral. It may be of the same
colour as the mineral or may differ — malachite is green and gives green streak, fluorite is
purple or green but gives a white streak.

QUESTION 34. MTU5Mzg0K0pheStqYXlhZGhlZXIuc3llQGdtYWlsLmNvbStRVUVTVElPTiAzMw==


Which of the following statements correctly defines the term ‘Petrology’?

a) It is science of soils
b) It is science of rocks
c) It is science of petroleum
d) It is science of coal
Correct Answer: B
Answer
Explanation

Solution (b)

IASbaba
Score:
Web:
Email:
Page 29 150
https://telegram.me/UPSCMaterials https://telegram.me/FreeUPSCMaterials https://telegram.me/MaterialforExam

2019 - Test 2-Geography


Exam Title :
& C...
Email
Contact :

Petrology is science of rocks.

A petrologist studies rocks in all their aspects viz., mineral composition, texture, structure,
origin, occurrence, alteration and relationship with other rocks.

Think

· Pedology

QUESTION 35. MTU5Mzg0K0pheStqYXlhZGhlZXIuc3llQGdtYWlsLmNvbStRVUVTVElPTiAzNA==


Which of the following process leads to the formation of folding mountains?

a) Orography
b) Epeirogeny
c) Volcanism
d) Earthquakes
Correct Answer: A
Answer
Explanation

Solution (a)

All processes that move, elevate or build up portions of the earth’s crust come under
diastrophism. They include:

(i) orogenic processes involving mountain building through severe folding and affecting long
and narrow belts of the earth’s crust;

(ii) epeirogenic processes involving uplift or warping of large parts of the earth’s crust;

(iii) isostatic movements maintaining the crustal equilibrium;

(iv) plate tectonics involving horizontal movements of crustal plates.

QUESTION 36. MTU5Mzg0K0pheStqYXlhZGhlZXIuc3llQGdtYWlsLmNvbStRVUVTVElPTiAzNQ==


Which of the following processes are correctly matched with their deriving forces/energies?

Process Driving Force

1. Weathering molecular stresses/chemical action

2. Mass wasting Gravitational force

3. Erosion Kinetic Energy

Select the code from the following:

a) 1 and 3
b) 2 and 3

IASbaba
Score:
Web:
Email:
Page 30 150
https://telegram.me/UPSCMaterials https://telegram.me/FreeUPSCMaterials https://telegram.me/MaterialforExam

2019 - Test 2-Geography


Exam Title :
& C...
Email
Contact :

c) 1 and 2
d) All of the above
Correct Answer: D
Answer
Explanation

Solution (d)

All are correctly matched.

Mass wasting , also known as slope movement or mass movement, is the geomorphic process
by which soil, sand, regolith, and rock move downslope typically as a mass , largely under the
force of gravity, but frequently affected by water and water content as in submarine
environments and mudflows.

Note: Point to be considered is that that Mass wasting is only because of the
gravitational pull and not due to any other external agent.

QUESTION 37. MTU5Mzg0K0pheStqYXlhZGhlZXIuc3llQGdtYWlsLmNvbStRVUVTVElPTiAzNg==


Salt Weathering is a kind of:

a) Physical Weathering
b) Chemical weathering
c) Biological weathering
d) None of the above
Correct Answer: A
Answer
Explanation

Solution (a)

Salt weathering is a type of physical weathering.

Salt Weathering

Salts in rocks expand due to thermal action, hydration and crystallisation. Many salts like
calcium, sodium, magnesium, potassium and barium have a tendency to expand. Expansion of
these salts depends on temperature and their thermal properties. High temperature ranges
between 30 and 50oC of surface temperatures in deserts favour such salt expansion. Salt
crystals in near-surface pores cause splitting of individual grains within rocks, which eventually
fall off. This process of falling off of individual grains may result in granular disintegration or
granular foliation.

QUESTION 38. MTU5Mzg0K0pheStqYXlhZGhlZXIuc3llQGdtYWlsLmNvbStRVUVTVElPTiAzNw==


Meanders are the sinous bends of the river in its course. Which of the following statements are
correct about meanders?

1. Meanders are the result of both erosional and depositional processes .

IASbaba
Score:
Web:
Email:
Page 31 150
https://telegram.me/UPSCMaterials https://telegram.me/FreeUPSCMaterials https://telegram.me/MaterialforExam

2019 - Test 2-Geography


Exam Title :
& C...
Email
Contact :

2. They are found in the upper course of a river

Select the code from below:

a) 1 only
b) 2 only
c) Both 1 and 2
d) Neither 1 nor 2
Correct Answer: A
Answer
Explanation

Solution (a)

Meanders:

· A meander, in general, is a bend in a sinuous watercourse or river .

· Meanders are the result of both erosional and depositional processes . A meander forms
when moving water in a stream erodes the outer banks and widens its valley, and the inner part
of the river has less energy and deposits silt.

· They are typical of the middle and lower course of a river .

QUESTION 39. MTU5Mzg0K0pheStqYXlhZGhlZXIuc3llQGdtYWlsLmNvbStRVUVTVElPTiAzOA==


An imaginary line connecting the deepest points in a river channel is called

a) Meander
b) Thalweg
c) Contour
d) Isohyte
Correct Answer: B
Answer
Explanation

Solution (b)

Thalweg

In geography and fluvial geomorphology, a thalweg or talweg is the line of lowest elevation
within a valley or watercourse. Under international law, a thalweg is the middle of the primary
navigable channel of a waterway that defines the boundary line between states.

IASbaba
Score:
Web:
Email:
Page 32 150
https://telegram.me/UPSCMaterials https://telegram.me/FreeUPSCMaterials https://telegram.me/MaterialforExam

2019 - Test 2-Geography


Exam Title :
& C...
Email
Contact :

QUESTION 40. MTU5Mzg0K0pheStqYXlhZGhlZXIuc3llQGdtYWlsLmNvbStRVUVTVElPTiAzOQ==


Which of the following statements correctly defines Isostacy ?

a) Isostasy is the process by which the oceanic lithosphere is consumed at subduction zones.
b) Isostasy is the creation of oceanic crust at mid ocean ridges.
c) Isostasy is the balance of the Earth’s upper mantle on top of the rigid crust.
d) Isostasy is the phenomenon that explains the balance of the Earth’s rigid crust on top of the
upper mantle.
Correct Answer: D
Answer
Explanation

Solution (d)

Isostacy

Isostasy, ideal theoretical balance of all large portions of Earth’s lithosphere as though they
were floating on the denser underlying layer, the asthenosphere, a section of the upper mantle
composed of weak, plastic rock that is about 110 km (70 miles) below the surface. Isostasy
controls the regional elevations of continents and ocean floors in accordance with the densities
of their underlying rocks.

Following Video will make the concept of Isostacy clearer.

https://www.britannica.com/science/isostasy-geology

QUESTION 41. MTU5Mzg0K0pheStqYXlhZGhlZXIuc3llQGdtYWlsLmNvbStRVUVTVElPTiA0MA==


Which of the following correctly explains ‘ Cratonic Landscape’?

a) Region of young Alpine mountain ranges


b) Regions around Mid Oceanic ridge
c) Regions of Hot Spot Activity
d) Old and tectonically stable part of Continent

IASbaba
Score:
Web:
Email:
Page 33 150
https://telegram.me/UPSCMaterials https://telegram.me/FreeUPSCMaterials https://telegram.me/MaterialforExam

2019 - Test 2-Geography


Exam Title :
& C...
Email
Contact :
Correct Answer: D
Answer
Explanation

Solution (d)

Craton

Craton, the stable interior portion of a continent characteristically composed of ancient


crystalline basement rock. The term craton is used to distinguish such regions from mobile
geosynclinal troughs, which are linear belts of sediment accumulations subject to subsidence
(i.e., downwarping). The extensive central cratons of continents may consist of both shields and
platforms. A shield is that part of a craton in which (usually) Precambrian basement rocks crop
out extensively at the surface. By contrast, in a platform the basement is overlain by horizontal
or subhorizontal sediments.

The Orange/red areas in the above map show the craton regions of the World.

QUESTION 42. MTU5Mzg0K0pheStqYXlhZGhlZXIuc3llQGdtYWlsLmNvbStRVUVTVElPTiA0MQ==


Faults are fractures in the Earth’s crust. Which of the following statements are correct
regarding Faults?

1. They are formed only due to tension.

2. Rifts valleys are formed due to Faulting

3. Faults in igneous rocks lead to formation of synclines and anticlines

4. Reverse Faults are formed in case of compression

Select the code from following:

a) 1,2 and 3
b) 2,3 and 4
c) 2 and 4
d) 1,3 and 4
Correct Answer: C

IASbaba
Score:
Web:
Email:
Page 34 150
https://telegram.me/UPSCMaterials https://telegram.me/FreeUPSCMaterials https://telegram.me/MaterialforExam

2019 - Test 2-Geography


Exam Title :
& C...
Email
Contact :
Answer
Explanation

Solution (c)

Faults:
A fault is a fracture in the earth’s crust due to tension force. It can also occur due to
compression in hard and brittle rocks.

· When there is tension the crust ruptures. One block is thrown upwards and the other
downwards. The upthrown block is called Horst while the downthrown block is called Graben .
The line along which the fault occurs is called strike. This fault is called normal fault and is
most common. In case of a normal fault, new surface is generated in the form of scarp.

· When there is compression, in case of hard rocks, instead of folding, the faulting occurs. The
block with hanging wall is thrown upwards while the one with footwall is thrown downwards.
This is called a reverse fault. In case of a reverse fault there is net destruction of the surface.

· When the forces are acting parallel to each other, along the line of fault the blocks move past
each other without being upthrown or downthrown. This is called lateral fault.

The landforms formed due to faulting of land are block mountains, rift valleys, step mountains,
hinge faults, scissors fault etc.

QUESTION 43. MTU5Mzg0K0pheStqYXlhZGhlZXIuc3llQGdtYWlsLmNvbStRVUVTVElPTiA0Mg==


Kilauea Volcano has been in news for quite some time for its continuous activity and oozing of
Lava. Where is it located?

a) Hawaii
b) Japan
c) Italy

IASbaba
Score:
Web:
Email:
Page 35 150
https://telegram.me/UPSCMaterials https://telegram.me/FreeUPSCMaterials https://telegram.me/MaterialforExam

2019 - Test 2-Geography


Exam Title :
& C...
Email
Contact :

d) Indonesia
Correct Answer: A
Answer
Explanation

Solution (a)

Kilauea Volcano lies in Hawaii state of USA.

https://www.theguardian.com/us-news/2018/jul/24/hawaiis-kilauea-volcano-eruption-could-last-
for-years-geologists-warn

https://www.youtube.com/watch?v=xgiZCrkRMhE

QUESTION 44. MTU5Mzg0K0pheStqYXlhZGhlZXIuc3llQGdtYWlsLmNvbStRVUVTVElPTiA0Mw==


Which of the following are intrusive volcanic landforms?

1. Crater

2. Sill

3. Dyke

4. Batholith

Select the code from below:

a) 1,2 and 3
b) 2,3 and 4
c) 1,3 and 4
d) All of the above
Correct Answer: B
Answer
Explanation

Solution (b)

Intrusive volcanic landforms:

IASbaba
Score:
Web:
Email:
Page 36 150
https://telegram.me/UPSCMaterials https://telegram.me/FreeUPSCMaterials https://telegram.me/MaterialforExam

2019 - Test 2-Geography


Exam Title :
& C...
Email
Contact :

Rocks formed by cooling of magma within the crust are called ‘Plutonic rocks’ or ‘Intrusive’
landforms.

Batholith: Batholiths are formed deep below the surface when large masses of magma cool
and solidify. As the magma cools slowly, large crystals are formed in the rock (e.g. granite).
Batholiths are often dome-shaped and exposed by later erosion.

Laccolith: Sometimes smaller injections of magma fom a lens shape that is intruded between
layers of rock. This then forces the overlying strata (layers of rock) to arch upawards, forming a
dome. This feature is knwon as a laccolith, and it may be exposed by later weathering and
erosion to form a small range of hills, for example the Eildon Hills on the Scottish Borders.

Dykes: These are vertical intrusions with horizontal colloing crakcs. They cut across the
bedding planes of the rocks into which they have been intruded. Dykes often occur in groups
where they are knwon as dyke swarms.

Fig: Dyke

Sill: These are horizontal intrusions along the lines of bedding planes. Sills have vertical
cooling cracks.

IASbaba
Score:
Web:
Email:
Page 37 150
https://telegram.me/UPSCMaterials https://telegram.me/FreeUPSCMaterials https://telegram.me/MaterialforExam

2019 - Test 2-Geography


Exam Title :
& C...
Email
Contact :

Fig: Sill

Lapolith : This is an intrusion of a saucer-shaped feature in between rock strata of sedimentary


rocks.

QUESTION 45. MTU5Mzg0K0pheStqYXlhZGhlZXIuc3llQGdtYWlsLmNvbStRVUVTVElPTiA0NA==


Which of the following statements are correct regarding Tsunamis?

1. They are triggered by earthquakes and vertical displacement of water in ocean.

2. As a Tsunami wave approaches the coast its velocity increases.

3. As a Tsunami wave approaches the coast its height increase.

Select the code from following:

a) 1 and 2
b) 2 and 3
c) 1 and 3
d) All of the above
Correct Answer: C
Answer
Explanation

Solution (c)

Tsunamis:
The seismic waves travelling through the ocean and sea water results into high sea waves
which are known as tsunamis. ‘Tsunami’ is a Japanese term which has been universally adopted
to describe a large seismically generated sea wave. These waves are responsible for causing
considerable destruction in certain coastal areas where submarine earthquakes occur.

Pre – conditions for Tsunami:

For a Tsunami to occur, two conditions are required:

IASbaba
Score:
Web:
Email:
Page 38 150
https://telegram.me/UPSCMaterials https://telegram.me/FreeUPSCMaterials https://telegram.me/MaterialforExam

2019 - Test 2-Geography


Exam Title :
& C...
Email
Contact :

· There should be an earthquake from which energy can be transferred.

· There should be a vertical displacement of the water. i.e. during earthquake the crust should
move vertically. That’s why Tsunamis are originated near oceanic trenches where plates are
being subducted. In Atlantic Ocean, a number of earthquakes occur on Mid – Oceanic ridge but
since there is no sudden vertical movement, Tsunamis are not formed. Tsunamis can also be
triggered if seamounts break. This can cause vertical displacement of water.

Process of Generation of Tsunami:

When a tsunami is generated, its steepness i.e. height to length ratio, is very less. This enables
it to pass unnoticed beneath the ships in the sea. As the wave approaches shore, the height of
the wave rapidly increases because of rebound from the shallow surface. The period of the
wave remains constant, velocity drops and the height increases as it approaches near
the coast from the Open Ocean. In confined coastal waters relatively close to their point of
origin, tsunamis can reach a height of more than 30m. Tsunamis travels at the speed of 100
-150 km/h which may pick up 650-900 km/h.

It may travel considerable distance. The frequency of tsunami is highest in Pacific Ocean.

QUESTION 46. MTU5Mzg0K0pheStqYXlhZGhlZXIuc3llQGdtYWlsLmNvbStRVUVTVElPTiA0NQ==


Which of the following forces is responsible for Landslides?

a) Kinetic energy of Running water


b) Glaciers
c) Gravity
d) Strong winds
Correct Answer: C
Answer
Explanation

Solution (c)

Landslides
A landslide is the movement of rock, debris or earth down a slope. They result from the failure
of the materials which make up the hill slope and are driven by the force of gravity. Landslides a
re known also as landslips, slumps or slope failure.

Types of landslide Movements:

· Falls are masses dislodged from very steep slopes or escarpments which then free-fall,
bounce, or roll downslope. Falls usually move extremely rapidly.

· Topples are a forward rotation around a pivot point low or below one or more masses.

· Lateral spreads are the result of movement involving lateral extension accommodated by
shear or tensile fractures. This type of movement is earthquake-induced.

· Slides displace masses along one or more discrete planes. Slides may either be rotational or
translational in their movement.

IASbaba
Score:
Web:
Email:
Page 39 150
https://telegram.me/UPSCMaterials https://telegram.me/FreeUPSCMaterials https://telegram.me/MaterialforExam

2019 - Test 2-Geography


Exam Title :
& C...
Email
Contact :

· Rotational movement is where the plane is curved. The mass rotates backwards around a
common point with an axis parallel to the slope.

· Translational movement is where the plane is more or less planar or gently undulating. The
mass moves roughly parallel to the ground surface.

· Flows are masses moving as a deforming, viscous unit without a discrete failure plane.

· More than one form of movement may be represented in some landslides. Movement in this
case is often described as complex.

QUESTION 47. MTU5Mzg0K0pheStqYXlhZGhlZXIuc3llQGdtYWlsLmNvbStRVUVTVElPTiA0Ng==


What type of rocks show Karst Landform formation?

IASbaba
Score:
Web:
Email:
Page 40 150
https://telegram.me/UPSCMaterials https://telegram.me/FreeUPSCMaterials https://telegram.me/MaterialforExam

2019 - Test 2-Geography


Exam Title :
& C...
Email
Contact :

a) Sand Stone
b) Limestone and Dolomite
c) Granite
d) Pumice
Correct Answer: B
Answer
Explanation

Solution (b)

Karst Landforms
Karst is a topography formed from the dissolution of soluble rocks such as limestone, dolomite,
and gypsum. It is characterized by underground drainage systems with sinkholes and caves. It
has also been documented for more weathering-resistant rocks, such as quartzite, given the
right conditions. Subterranean drainage may limit surface water, with few to no rivers or lakes.
However, in regions where the dissolved bedrock is covered (perhaps by debris) or confined by
one or more superimposed non-soluble rock strata, distinctive karst features may occur only at
subsurface levels and be totally missing above ground.

Surface Landforms

Different depressions are formed on the surface by seeping of water –

Sink hole – It is a small deep depression formed on the surface. A sinkhole is a depression in
the ground that has no natural external surface drainage. Basically this means that when it
rains, all of the water stays inside the sinkhole and typically drains into the subsurface.
Sinkholes are most common on Karst Terrain.

Doline – Few sinkholes combine to form a larger depression called Doline. Sometimes clay gets
settled on the bottom of Doline stopping water to seep through it. When water gets
accumulated in the doline, it is known as a Doline Lake.

Uvala – several Dolines combine to form a Uvala.

IASbaba
Score:
Web:
Email:
Page 41 150
https://telegram.me/UPSCMaterials https://telegram.me/FreeUPSCMaterials https://telegram.me/MaterialforExam

2019 - Test 2-Geography


Exam Title :
& C...
Email
Contact :

Polje – When an underground cave collapse, over the surface a large depression is formed. It is
called Polje.

Subsurface Landforms

Stalactites and Stalagmites –

Stalactite and stalagmite, elongated forms of various minerals deposited from solution by slowly
dripping water. A stalactite hangs like an icicle from the ceiling or sides of a cavern. A
stalagmite appears like an inverted stalactite, rising from the floor of a cavern.

Coloumn – When stalactite and stalagmite meet, they form a complete pillar in the
underground cave. This is called a coloumn.

QUESTION 48. MTU5Mzg0K0pheStqYXlhZGhlZXIuc3llQGdtYWlsLmNvbStRVUVTVElPTiA0Nw==


Which of the following statements are correct regarding Soils?

1. Finely divided, partially decomposed organic matter in soil is called humus.

2. Air in soils may have high levels of CO 2 and methane and low O 2 .

3. Water present in soil does not contain any dissolved nutrient.

Select the code from following:

a) 1 only
b) 2 and 3
c) 1 and 2
d) 2 only
Correct Answer: C
Answer
Explanation

Solution (c)

Soil consists of matter in all the three states – solid, liquid and gaseous.

IASbaba
Score:
Web:
Email:
Page 42 150
https://telegram.me/UPSCMaterials https://telegram.me/FreeUPSCMaterials https://telegram.me/MaterialforExam

2019 - Test 2-Geography


Exam Title :
& C...
Email
Contact :

Mineral Matter: Inorganic materials, or those materials that are not living, include weathered
rocks and minerals.

Organic matter: Organic matter has biological origin and can be living or dead.

Humus

Finely divided, partially decomposed organic matter in soil is called humus. (Note: This is the
complete definition of humus. learn it this way.) When abundant, humus particles can give the
soil brown or black colouration.

Both air and water are found in soil. Water may tend to contain high levels of dissolved
nutrients.

Air in soils may have high levels of CO 2 and methane and low O 2 .

Solid, liquid and gaseous matter in soil are constantly changing and interacting through
physical and chemical processes. This makes the soil a very dynamic layer. The soil science is
called ‘ Pedology’.

QUESTION 49. MTU5Mzg0K0pheStqYXlhZGhlZXIuc3llQGdtYWlsLmNvbStRVUVTVElPTiA0OA==


World’s longest Sandstone cave named ‘ Krem Puri ’ has been discovered in India. It is located
in

a) Maharashtra
b) Manipur
c) Meghalaya
d) Madhya Pradesh
Correct Answer: C
Answer
Explanation

Solution (c)

Krem Puri

The world's longest sandstone cave named Krem Puri was recently discovered in Meghalaya. It
has a staggering length of 24.5 km, almost three times the height of Mount Everest, and
contains some dinosaur fossils from 66-76 million years ago as well.

'Krem' means 'cave' in Khasi language. The cave system which is 24,583 metres long was
discovered in near Laitsohum village, located in the Mawsynram area in Meghalaya's East
Khasi Hills district.

Though Krem Puri had been discovered in 2016 itself, its actual length was mapped only
recently when the Meghalaya Adventurers' Association (MAA) took a 25-day expedition to this
end.

QUESTION 50. MTU5Mzg0K0pheStqYXlhZGhlZXIuc3llQGdtYWlsLmNvbStRVUVTVElPTiA0OQ==

IASbaba
Score:
Web:
Email:
Page 43 150
https://telegram.me/UPSCMaterials https://telegram.me/FreeUPSCMaterials https://telegram.me/MaterialforExam

2019 - Test 2-Geography


Exam Title :
& C...
Email
Contact :

A new Epoch has been added in the geological History named as Meghalayan Age. It refers to
history from

a) 4200 years BP (Before Present)


b) 4200 Thousand Years BP
c) 4200 Million years BP
d) 42 million years BP
Correct Answer: A
Answer
Explanation

Solution (a)

Meghalayan Age

In the geologic time scale, the Meghalayan is the latest age or uppermost stage of the
Quaternary. It is also the upper or latest of three subdivisions of the Holocene epoch or series.
Its Global Boundary Stratotype Section and Point (GSSP) is a Mawmluh cave formation from
Meghalaya, northeast India.

The Meghalayan begins 4,200 years BP ie. before 1950 , leaving open room for the
possible creation of the Anthropocene from 1950 forward. The age began with a 200-year
drought which impacted human civilizations in Egypt, Greece, Syria, Palestine, Mesopotamia,
the Indus Valley and the Yangtze River Valley.

"The fact that the beginning of this age coincides with a cultural shift caused by a global
climate event makes it unique", according to Stanley Finney, Secretary General of the
International Union of Geological Sciences.

The age was officially ratified by the ICS in July 2018 along with the Greenlandian and the
Northgrippian.

QUESTION 51. MTU5Mzg0K0pheStqYXlhZGhlZXIuc3llQGdtYWlsLmNvbStRVUVTVElPTiA1MA==


Consider the following statements about the term ‘Base Erosion and Profit Shifting (BEPS)’

1. It is concerned with exploitation of genetic resources of a country by multinational


companies

2. BEPS Project is formulated by World Economic Forum and United Nations Environment
Programme (UNEP)

Select the correct statements

a) 1 Only
b) 2 Only
c) Both 1 and 2
d) Neither 1 nor 2
Correct Answer: D
Answer
Explanation

IASbaba
Score:
Web:
Email:
Page 44 150
https://telegram.me/UPSCMaterials https://telegram.me/FreeUPSCMaterials https://telegram.me/MaterialforExam

2019 - Test 2-Geography


Exam Title :
& C...
Email
Contact :

Solution (d)

Base erosion and profit shifting (BEPS)

· It refers to tax avoidance strategies that exploit gaps and mismatches in tax rules to artificially
shift profits to low or no-tax locations.

· The Inclusive Framework on BEPS brings together over 115 countries and jurisdictions to
collaborate on the implementation of the OECD/ G20 Base Erosion and Profit Shifting (BEPS)
Package.

· OECD/G20 Inclusive Framework on Base Erosion and Profit Shifting (BEPS) and its members
tackles tax avoidance by ensuring the implementation of the measures agreed through the
BEPS Project, which targets multinational enterprises’ aggressive tax planning practices.

· In particular, four “minimum standards” are at the core of the BEPS measures: harmful tax
practices, treaty abuse, country-by-country reporting and dispute resolution mechanisms.

Source: https://www.thehindu.com/business/us-reacting-to-beps-with-barriers-to-free-
fair-trade/article24505566.ece

QUESTION 52. MTU5Mzg0K0pheStqYXlhZGhlZXIuc3llQGdtYWlsLmNvbStRVUVTVElPTiA1MQ==


Which of the following is/are included in the ‘UNESCO’s List of the Intangible Cultural Heritage
of Humanity’?

1. Kutiyattam

2. Ramlila

3. Kumb Mela

Select the correct code:

a) 1 and 2
b) 2 Only
c) 1, 2 and 3
d) 1 and 3
Correct Answer: C
Answer
Explanation

Solution (c)

Kumbh Mela was inscribed in 2017 in UNESCO’s Lists of Intangible Cultural Heritage. Ramlila
and Kutiyattam were inscribed in 2008.

Source: http://pib.nic.in/PressReleseDetail.aspx?PRID=1539899 (Implied Question)

In News: ‘Safeguarding the Intangible Cultural Heritage and Diverse Cultural Traditions of
India’

· Implemented by - Ministry of Culture

IASbaba
Score:
Web:
Email:
Page 45 150
https://telegram.me/UPSCMaterials https://telegram.me/FreeUPSCMaterials https://telegram.me/MaterialforExam

2019 - Test 2-Geography


Exam Title :
& C...
Email
Contact :

· Objective – To reinvigorate and revitalize various institutions, groups, individuals, etc. so that
they may engage in activities/ projects for strengthening, protecting, preserving and promoting
the rich intangible cultural heritage of India.

· The scheme is being implemented through Sangeet Natak Akademi, an autonomous


organisation under the Ministry of Culture.

Kutiyattam - https://ich.unesco.org/en/RL/kutiyattam-sanskrit-theatre-00010

Ramlila - https://ich.unesco.org/en/RL/ramlila-the-traditional-performance-of-the-
ramayana-00110

Kumbh Mela - https://ich.unesco.org/en/RL/kumbh-mela-01258

QUESTION 53. MTU5Mzg0K0pheStqYXlhZGhlZXIuc3llQGdtYWlsLmNvbStRVUVTVElPTiA1Mg==


Which of the following Indian States borders ‘Bhutan’?

1. West Bengal

2. Sikkim

3. Arunachal Pradesh

4. Meghalaya

Select the correct code:

a) 1 and 2 Only
b) 2 and 3 Only
c) 1, 2 and 3 Only
d) 2, 3 and 4 Only
Correct Answer: C
Answer
Explanation

Solution (c)

The border is 699 km long, and adjoins the Indian states of Assam (267 km), Arunachal Pradesh
(217 km), West Bengal (183 km), and Sikkim (32 km).

THINK!

· Indian States bordering Myanmar

Source: https://www.thehindu.com/opinion/op-ed/befriending-the-neighbour/
article24497565.ece

QUESTION 54. MTU5Mzg0K0pheStqYXlhZGhlZXIuc3llQGdtYWlsLmNvbStRVUVTVElPTiA1Mw==


The term ‘AT2018Cow’ was recently in news in the context of

IASbaba
Score:
Web:
Email:
Page 46 150
https://telegram.me/UPSCMaterials https://telegram.me/FreeUPSCMaterials https://telegram.me/MaterialforExam

2019 - Test 2-Geography


Exam Title :
& C...
Email
Contact :

a) Quantum Mechanics
b) A newly developed ransomware attacking MacOS devices
c) A celestial object
d) A new diseases discovered in Switzerland affecting cows and buffaloes
Correct Answer: C
Answer
Explanation

Solution (c)

AT2018cow

· It is a very bright celestial object seen close to the very small galaxy CGCG 137-068

· It is a powerful astronomical explosion, 10 – 100 times brighter than a normal supernova

· It is a Type Ic supernova . It is caused by the explosion of an extremely massive star which


has lost its outer layers of hydrogen and helium.

Supernova

· It is a transient astronomical event that occurs during the last stellar evolutionary stages of a
star's life, either a massive star or a white dwarf, whose destruction is marked by one final,
titanic explosion.

· This causes the sudden appearance of a "new" bright star, before slowly fading from sight over
several weeks or months or years.

THINK!

· ATLAS Telescope

Source: https://www.thehindu.com/todays-paper/tp-opinion/the-deep-and-far-of-
science/article24499199.ece

QUESTION 55. MTU5Mzg0K0pheStqYXlhZGhlZXIuc3llQGdtYWlsLmNvbStRVUVTVElPTiA1NA==


‘Foreign Direct Investment Confidence (FDI) Index’ is published by

a) World Economic Forum


b) World Trade Organisation
c) World Bank
d) None of the above
Correct Answer: D
Answer
Explanation

Solution (d)

The Foreign Direct Investment Confidence (FDI) Index prepared by A.T. Kearney is an annual
survey which tracks the impact of likely political, economic, and regulatory changes on the
foreign direct investment intentions and preferences of CEOs, CFOs, and other top executives
of Global 1000 companies.

IASbaba
Score:
Web:
Email:
Page 47 150
https://telegram.me/UPSCMaterials https://telegram.me/FreeUPSCMaterials https://telegram.me/MaterialforExam

2019 - Test 2-Geography


Exam Title :
& C...
Email
Contact :

India was ranked 11 th in the2018 index.

India remains the second highest ranked emerging market on the Index. A variety of recent
reforms have made its regulatory environment more business friendly and economic growth is
forecast to rebound this year.

DO YOU KNOW?

· After the Foreign Investment Promotion Board (FIPB) was abolished, Individual
departments of the government have been empowered to clear FDI proposals in consultation
with DIPP which will also issue the standard operating procedures for processing applications.

· All FDI from Pakistan and Bangladesh and FDI proposals requiring approval of private security
agencies and manufacture of small arms will require to be approved by Ministry of Home
Affairs. While foreign investments by non-resident Indians and FDI in retail and export oriented
units will be approved by DIPP, FDI in banks will be approved by the Department of Financial
Services. DIPP or Department of Economic Affairs will undertake a quarterly review of FDI
proposals.

· Financial Services not regulated by a regulator or where there is more than one regulator or
in respect of which there is a doubt about the regulator will be approved by Department of
Economic Affairs, Ministry of Finance.

Source: https://www.thehindu.com/business/Economy/india-drops-three-ranks-in-at-
kearney-fdi-confidence-index/article24497785.ece

QUESTION 56. MTU5Mzg0K0pheStqYXlhZGhlZXIuc3llQGdtYWlsLmNvbStRVUVTVElPTiA1NQ==


‘ Shekatkar Committee’ is NOT associated with which of the following?

1. Enhancing combat capability of the armed forces

2. Rebalancing defence expenditure of the armed forces

3. Border fencing along the India-Pakistan border

Select the correct code:

a) 1 and 3
b) 2 and 3
c) 3 Only
d) 2 Only
Correct Answer: C
Answer
Explanation

Solution (c)

The Ministry of Defence had constituted a Committee of Experts under the Chairmanship of Lt
Gen (Retd) (Dr.) DB Shekatkar with a mandate to recommend measures for enhancing of
Combat Capability & Rebalancing Defence Expenditure of the Armed Forces with an aim to
increase "teeth to tail ratio".

IASbaba
Score:
Web:
Email:
Page 48 150
https://telegram.me/UPSCMaterials https://telegram.me/FreeUPSCMaterials https://telegram.me/MaterialforExam

2019 - Test 2-Geography


Exam Title :
& C...
Email
Contact :

Recommendations & Source: http://pib.nic.in/newsite/PrintRelease.aspx?relid=180848

QUESTION 57. MTU5Mzg0K0pheStqYXlhZGhlZXIuc3llQGdtYWlsLmNvbStRVUVTVElPTiA1Ng==


Consider the following statements about ‘Uganda’

1. It is part of the African Great Lakes region

2. It is bordered by Kenya, Rwanda and Burundi

Select the correct statements

a) 1 Only
b) 2 Only
c) Both 1 and 2
d) Neither 1 nor 2
Correct Answer: A
Answer
Explanation

Solution (a)

Uganda is a landlocked country in East Africa. It is bordered to the east by Kenya, to the north
by South Sudan, to the west by the Democratic Republic of the Congo, to the south-west by
Rwanda, and to the south by Tanzania. The southern part of the country includes a substantial
portion of Lake Victoria, shared with Kenya and Tanzania. Uganda is in the African Great Lakes
region. Uganda also lies within the Nile basin, and has a varied but generally a modified
equatorial climate.

The African Great Lakes are a series of lakes constituting the part of the Rift Valley lakes in and
around the East African Rift. They include Lake Victoria, the third-largest fresh water lake in
the world by area, and Lake Tanganyika, the world's second-largest freshwater lake by volume
and depth. Countries in the African Great Lakes region (sometimes also called Greater Lakes
region) include Burundi, the Democratic Republic of the Congo, Kenya, Rwanda, Tanzania and
Uganda.

IASbaba
Score:
Web:
Email:
Page 49 150
https://telegram.me/UPSCMaterials https://telegram.me/FreeUPSCMaterials https://telegram.me/MaterialforExam

2019 - Test 2-Geography


Exam Title :
& C...
Email
Contact :

Source: https://economictimes.indiatimes.com/news/politics-and-nation/india-
donates-200-cows-for-rwandan-economic-development-project/
articleshow/65116913.cms

QUESTION 58. MTU5Mzg0K0pheStqYXlhZGhlZXIuc3llQGdtYWlsLmNvbStRVUVTVElPTiA1Nw==


Consider the following statements about ‘Salicornia’

1. Andhra Pradesh High Court has ordered the State government to enact a law on prohibition
of Salicornia as it releasing toxic chemicals into water bodies causing health problems in the
state.

2. It grows in abundance on salty marshes in the mangrove wetlands

Select the correct statements

a) 1 Only
b) 2 Only
c) Both 1 and 2
d) Neither 1 nor 2
Correct Answer: B
Answer
Explanation

Solution (b)

Salicornia

In News: Andhra Pradesh is exploring the possibilities to extract the salt substitute through
various methods.

About

· It is a genus of succulent, halophyte (salt tolerant) flowering plants

· It is a plant that grows in abundance on salty marshes in the mangrove wetlands.

· The plant yields a salt substitute with low sodium content.

· They are widely distributed over the Northern Hemisphere and in southern Africa, ranging
from the subtropics to subarctic regions. They are absent from South America and Australia

THINK

· Halophytes

· Pneumatophores

Source: https://www.thehindu.com/todays-paper/state-to-tap-benefits-of-salicornia/
article24490729.ece

IASbaba
Score:
Web:
Email:
Page 50 150
https://telegram.me/UPSCMaterials https://telegram.me/FreeUPSCMaterials https://telegram.me/MaterialforExam

2019 - Test 2-Geography


Exam Title :
& C...
Email
Contact :
QUESTION 59. MTU5Mzg0K0pheStqYXlhZGhlZXIuc3llQGdtYWlsLmNvbStRVUVTVElPTiA1OA==
Consider the following statements

1. GDP Deflator is the ratio of Nominal GDP to Real GDP

2. Nominal GDP is economic output without the inflation adjustment

3. Real GDP is usually higher than the Nominal GDP

Select the correct statements

a) 1 and 2
b) 2 and 3
c) 1 and 3
d) All of the above
Correct Answer: A
Answer
Explanation

Solution (a)

GDP deflator

· It also called implicit price deflator.

· It is a measure of inflation.

· It is the ratio of the value of goods and services an economy produces in a particular year at
current prices to that of prices that prevailed during the base year.

· This ratio helps show the extent to which the increase in gross domestic product has happened
on account of higher prices rather than increase in output.

· Since the deflator covers the entire range of goods and services produced in the economy — as
against the limited commodity baskets for the wholesale or consumer price indices — it is seen
as a more comprehensive measure of inflation.

Real vs nominal GDP

· GDP price deflator measures the difference between real GDP and nominal GDP.

· Nominal GDP doesn’t include inflation. Real GDP includes inflation.

· As a result, nominal GDP will most often be higher than real GDP in an expanding economy
(inflation is typically a positive number).

The formula to find the GDP price deflator:

· GDP price deflator = (nominal GDP ÷ real GDP) x 100

WPI and CPI

· A consumer price index (CPI) measures changes over time in the general level of prices of
goods and services that households acquire for the purpose of consumption.

IASbaba
Score:
Web:
Email:
Page 51 150
https://telegram.me/UPSCMaterials https://telegram.me/FreeUPSCMaterials https://telegram.me/MaterialforExam

2019 - Test 2-Geography


Exam Title :
& C...
Email
Contact :

· However, since CPI is based only a basket of select goods and is calculated on prices included
in it, it does not capture inflation across the economy as a whole.

· The wholesale price index basket has no representation of the services sector and all the
constituents are only goods whose prices are captured at the wholesale/producer level.

Deflator <> CPI

· Changes in consumption patterns or introduction of goods and services are automatically


reflected in the GDP deflator.

· This allows the GDP deflator to absorb changes to an economy’s consumption or investment
patterns.

· Often, the trends of the GDP deflator will be similar to that of the CPI.

· GDP deflator reflects up-to-date expenditure patterns.

· GDP deflator is available only on a quarterly basis along with GDP estimates, whereas CPI and
WPI data are released every month.

Source: https://www.thehindu.com/business/Economy/what-is-the-gdp-deflator/
article24489279.ece

QUESTION 60. MTU5Mzg0K0pheStqYXlhZGhlZXIuc3llQGdtYWlsLmNvbStRVUVTVElPTiA1OQ==


Which of the following statements about Protection of Children from Sexual Offences Act
(POCSO) is/are correct?

1. The Act defines a child as any person below 16 years of age

2. An online complaint box for reporting child sexual abuse, the POCSO e-Box was launched by
the Minister for Women and Child Development

Select the correct statements

a) 1 Only
b) 2 Only
c) Both 1 and 2
d) Neither 1 nor 2
Correct Answer: B
Answer
Explanation

Solution (b)

Protection of Children from Sexual Offences (POCSO)

News: Proposed for enhanced punishment in cases of sexual assault of male children

· It was established to protect the children against offences like sexual abuse, sexual
harassment and pornography.

IASbaba
Score:
Web:
Email:
Page 52 150
https://telegram.me/UPSCMaterials https://telegram.me/FreeUPSCMaterials https://telegram.me/MaterialforExam

2019 - Test 2-Geography


Exam Title :
& C...
Email
Contact :

· It was formed to provide a child-friendly system for trial underneath which the perpetrators
could be punished.

· National Commission for Protection of Child Rights (NCPCR) will monitor the implementation
of the Act

· It defines a child as any person below eighteen years of age. It also makes provisions for
avoiding the re-victimisation of the child at the hands of the judicial system.

· It makes it mandatory to report such cases. It makes it the legal duty of a person aware of the
offence to report the sexual abuse.

· It also prescribes punishment to the people who traffic children for sexual purposes.

· It states that the evidence of the child should be recorded within a period of thirty days.

· The Special Court taking cognizance of the matter should be able to complete the trial within
the period of one year from the date of taking cognizance of the abuse.

· It provides that the Special Court proceedings should be recorded in camera and the trial
should take place in the presence of parents or any other person in whom the child has trust or
confidence.

· Ordinance in April 2018 - death penalty will be given to those convicted of raping a child up to
12 years of age.

· The POCSO e-box is an easy and direct medium for reporting any case of sexual assault under
Protection of Children from Sexual Offences {POCSO) Act, 2012. It is displayed prominently in
the home page of NCPCR website where the user has to simply press a button named, POCSO
e-box which will navigate to a page with the window having a short animation movie.

Source: https://www.thehindu.com/todays-paper/tp-national/wcd-to-move-proposal-to-
amend-pocso-act/article24491808.ece

QUESTION 61. MTU5Mzg0K0pheStqYXlhZGhlZXIuc3llQGdtYWlsLmNvbStRVUVTVElPTiA2MA==


What is the purpose of the recently proposed ‘SRIMAN Scheme’?

a) To enable the famous foreign education institutions to open their campuses in India
b) To increase the quality of education provided in government schools by taking help
from the private sector and the community
c) To address various issues for effective utilization of Government’s research
infrastructure
d) To encourage voluntary monetary contribution from private individuals and
organizations so as to improve the infrastructure facilities for primary and secondary
schools
Correct Answer: C
Answer
Explanation

Solution (c)

Scientific Research Infrastructure Management and Networks (SRIMAN)

IASbaba
Score:
Web:
Email:
Page 53 150
https://telegram.me/UPSCMaterials https://telegram.me/FreeUPSCMaterials https://telegram.me/MaterialforExam

2019 - Test 2-Geography


Exam Title :
& C...
Email
Contact :

NEWS: SRIMAN proposed to transform scientific instruments in government labs into lucrative
assets generating a steady rental income

About

· To address various issues for effective utilization of its research infrastructure.

· It plans to hire out to researchers all lab equipment that cost more than ₹10 lakh.

Objective

· Simplified but smarter ways of procurement and maintenance of scientific equipment and
infrastructure.

· Providing greater access to instruments funded by the Government of India and its agencies
through a network all over the country.

· Providing a framework for smarter ways of disposal of dated equipments and infrastructure

· Providing a framework to improve and promote efficiency of operations of research


infrastructure

· A broad template to monitor utilization of expensive research infrastructure.

Benefits

· It will promote creation of a regional ecosystem, by galvanizing relevant stakeholders of


scientific research infrastructure that provides wider access to scientists, researchers and
industry professionals across the country.

· It will also help in creating a pool of trained operators for operation and maintenance of the
equipment.

Source: https://www.thehindu.com/education/rent-a-lab-policy-to-bring-revenues-to-
institutions/article24477344.ece

QUESTION 62. MTU5Mzg0K0pheStqYXlhZGhlZXIuc3llQGdtYWlsLmNvbStRVUVTVElPTiA2MQ==


‘Reverse Charge Mechanism’, recently in news, is associated with

a) Goods and Services Tax (GST)


b) Initial Public Offering (IPO)
c) Litigation over bounced cheques
d) Regional Connectivity Scheme (RCS)
Correct Answer: A
Answer
Explanation

Solution (a)

GST Reverse Charge Mechanism (RCM) basically means that the GST is to be paid and
deposited with the Govt by the recipient of Goods/ Services and not by the supplier of Goods/
Services.

IASbaba
Score:
Web:
Email:
Page 54 150
https://telegram.me/UPSCMaterials https://telegram.me/FreeUPSCMaterials https://telegram.me/MaterialforExam

2019 - Test 2-Geography


Exam Title :
& C...
Email
Contact :

Source: http://pib.nic.in/newsite/PrintRelease.aspx?relid=180825

QUESTION 63. MTU5Mzg0K0pheStqYXlhZGhlZXIuc3llQGdtYWlsLmNvbStRVUVTVElPTiA2Mg==


Consider the following statements about ‘ Kamsale ’

1. It is a folk dance from Odisha performed solely by women

2. It is performed by the devotees of Lord Jagannath

Select the correct statements

a) 1 Only
b) 2 Only
c) Both 1 and 2
d) Neither 1 nor 2
Correct Answer: D
Answer
Explanation

Solution (d)

Kamsale

· It is a folk dance belonging to Mysore region.

· It is a unique folk art performed by the devotees of God Mahadeshwara.

· Kamsale is a brass made musical instrument. Its origin is traced to the Mythological period.

· The instrument kamsale has a gari (plate) and an adi battalu (cup). If gari signifies the sky, adi
battalu represents the earth. Together the kamsale taala is a symbol of this whole creation by
god.

· Traditionally, only men used to dance this. However, in the present times girls are not barred
from learning it.

Source: https://www.thehindu.com/entertainment/dance/cling-clang-fly-high-all-for-
the-god/article24470913.ece

QUESTION 64. MTU5Mzg0K0pheStqYXlhZGhlZXIuc3llQGdtYWlsLmNvbStRVUVTVElPTiA2Mw==


Consider the following statements about Pradhan Mantri Sahaj Bijli Har Ghar Yojana –“ Saubha
gya ”

1. Poor households identified using Socio Economic and Caste Census (SECC) 2011 would be
provided with free electricity

2. Deendayal Upadhyaya Gram Jyoti Yojana (DDUGJY) and Integrated Power Development
Scheme (IPDS) have been subsumed under the ‘ Saubhagya ’ scheme

IASbaba
Score:
Web:
Email:
Page 55 150
https://telegram.me/UPSCMaterials https://telegram.me/FreeUPSCMaterials https://telegram.me/MaterialforExam

2019 - Test 2-Geography


Exam Title :
& C...
Email
Contact :

Select the correct statements

a) 1 Only
b) 2 Only
c) Both 1 and 2
d) Neither 1 nor 2
Correct Answer: D
Answer
Explanation

Solution (d)

SAuBHaGYa: Pradhan Mantri Sahaj Bijli Har Ghar Yojana

· To achieve universal household electrification in the country

· To provide last mile connectivity and electricity connections to all households in rural and
urban areas.

· Free of cost electricity connections to all remaining un-electrified households with at least one
deprivation on the basis of SECC data in rural areas and economically poor households in urban
areas would be given.

· Others would be charged a sum of Rs. 500 per household in ten equal instalments with the bill.

· The households located in remote and inaccessible areas would be provided with Solar
Photovoltaic (SPV) based standalone systems with LED lights, fan, power plug etc.

· The beneficiaries will be identified on the basis of socio economic conditions using SECC 2011
data.

· Target - universal household electrification in the country by 31st March 2019

· Saubhagya has been launched to plug gaps of DDUGJY & IPDS and comprehensively address
the issues of entry barrier, last mile connectivity and release of electricity connections to all un-
electrified households in rural and urban areas.

· ‘Saubhagya’ Web-Portal – a Platform for transparently monitoring Universal Household


Electrification

DO YOU KNOW?

Deendayal Upadhyaya Gram Jyoti Yojana (DDUGJY)

· It envisages creation of basic electricity infrastructure in villages / habitations, strengthening


& augmentation of existing infrastructure, metering of existing feeders / distribution
transformers / consumers to improve quality and reliability of power supply in rural areas.

· Last mile connectivity and free electricity connections are also provided to BPL households
only identified by the States as per their list

Integrated Power Development Scheme (IPDS)

· To provide quality and reliable 24x7 power supply in the urban area

IASbaba
Score:
Web:
Email:
Page 56 150
https://telegram.me/UPSCMaterials https://telegram.me/FreeUPSCMaterials https://telegram.me/MaterialforExam

2019 - Test 2-Geography


Exam Title :
& C...
Email
Contact :

· It provides for creation of necessary infrastructure to provide electricity access but some
households are not yet connected mainly on account of their economic condition as they are not
capable of paying the initial connection charges.

Source: http://pib.nic.in/PressReleseDetail.aspx?PRID=1539203

QUESTION 65. MTU5Mzg0K0pheStqYXlhZGhlZXIuc3llQGdtYWlsLmNvbStRVUVTVElPTiA2NA==


‘Strategic Arms Reduction Treaty’ was a treaty between

a) Countries carved out of Yugoslavia


b) Germany, Italy and France post World War II
c) United States of America and Union of Soviet Socialist Republics (USSR)
d) Japan and United States of America
Correct Answer: C
Answer
Explanation

Solution (c)

START (Strategic Arms Reduction Treaty)

START 1

· It was a bilateral treaty between the United States of America and the Union of Soviet
Socialist Republics (USSR) on the reduction and limitation of strategic offensive arms.

· It barred its signatories from deploying more than 6,000 nuclear warheads atop a total of
1,600 inter-continental ballistic missiles (ICBMs) and bombers.

· The START I treaty expired 5 December 2009.

NEW START

· New START (Strategic Arms Reduction Treaty) is a nuclear arms reduction treaty between the
United States and the Russian Federation with the formal name of Measures for the Further
Reduction and Limitation of Strategic Offensive Arms.

· It was signed on 8 April 2010 in Prague

· It is set to expire in 2021

THINK!

Strategic Offensive Reductions Treaty

Source: https://www.thehindu.com/opinion/editorial/rocky-summit/article24454560.ece

QUESTION 66. MTU5Mzg0K0pheStqYXlhZGhlZXIuc3llQGdtYWlsLmNvbStRVUVTVElPTiA2NQ==


Consider the following statements about ‘International Comparison Programme (ICP)’

IASbaba
Score:
Web:
Email:
Page 57 150
https://telegram.me/UPSCMaterials https://telegram.me/FreeUPSCMaterials https://telegram.me/MaterialforExam

2019 - Test 2-Geography


Exam Title :
& C...
Email
Contact :

1. It supports inter-country comparisons of Gross Domestic Product (GDP) and its components,
using Purchasing Power Parity (PPP)

2. It is led by the World Bank under the auspices of the United Nations Statistical Commission.

Select the correct statements

a) 1 Only
b) 2 Only
c) Both 1 and 2
d) Neither 1 nor 2
Correct Answer: C
Answer
Explanation

Solution (c)

International Comparison Program (ICP)

· The ICP is a worldwide statistical initiative led by the World Bank under the auspices of the
United Nations Statistical Commission, with the main objective of providing comparable price
and volume measures of gross domestic product (GDP) and its expenditure aggregates among
countries within and across regions.

· Through a partnership with international, regional, sub-regional and national agencies, the
ICP collects and compares price data and GDP expenditures to estimate and publish purchasing
power parities (PPPs) of the world’s economies.

PPPs

· PPPs measure the total amount of goods and services that a single unit of a country’s currency
can buy in another country.

· PPPs are widely used to convert national accounts data, like GDP, into a common currency,
while also eliminating the effect of price level differences between countries.

News: 50th Anniversary of the ICP

QUESTION 67. MTU5Mzg0K0pheStqYXlhZGhlZXIuc3llQGdtYWlsLmNvbStRVUVTVElPTiA2Ng==


Consider the following statements about Communications Compatibility and Security
Agreement

1. It is one of United States’ foundational agreements, which is important for enhanced bilateral
defence cooperation with India

2. It will enable Indian military to obtain critical, secure and encrypted defence technologies

Select the correct statements

a) 1 Only
b) 2 Only
c) Both 1 and 2

IASbaba
Score:
Web:
Email:
Page 58 150
https://telegram.me/UPSCMaterials https://telegram.me/FreeUPSCMaterials https://telegram.me/MaterialforExam

2019 - Test 2-Geography


Exam Title :
& C...
Email
Contact :

d) Neither 1 nor 2
Correct Answer: C
Answer
Explanation

Solution (c)

COMCASA is one of three foundational agreements that guide US high technology cooperation
in defence sector with other countries. Other two agreements are Logistics Exchange
Memorandum of Agreement (LEMOA) and BECA.

COMCASA creates the conditions for the Indian military to receive modern secure and net-
enabled weapons systems such as precision armament, air-to-air missiles, space systems and
navigation systems that are critical components in platforms like fighter aircraft and unmanned
aerial systems.

Being a ‘Major Defence Partner’ of the US, it is imperative for India to sign the mandatory three
foundational pacts which allows greater interoperability between critical technologies and
smooth facilitation of classified information.

So far, India has signed only one out of the three foundational agreements, called the Logistics
Exchange Memorandum of Agreement (LEMOA). It enables access to each other’s military
facilities for purposes of refuelling and replenishment.

Source: https://www.thehindu.com/news/national/india-going-ahead-with-purchase-of-
s-400-air-defence-systems-from-russia-nirmala-sitharaman/article24413391.ece

QUESTION 68. MTU5Mzg0K0pheStqYXlhZGhlZXIuc3llQGdtYWlsLmNvbStRVUVTVElPTiA2Nw==


Consider the following statements about Voter Verifiable Paper Audit Trail (VVPAT)

1. It allows voters to verify if their vote has gone to the intended candidate

2. It will completely replace the electronic voting machines (EVMs)

Select the correct statements

a) 1 Only
b) 2 Only
c) Both 1 and 2
d) Neither 1 nor 2
Correct Answer: A
Answer
Explanation

Solution (a)

What is VVPAT?

· A Voter-verified paper audit trail (VVPAT) unit provides feedback to voters using EVMs for
voting.

IASbaba
Score:
Web:
Email:
Page 59 150
https://telegram.me/UPSCMaterials https://telegram.me/FreeUPSCMaterials https://telegram.me/MaterialforExam

2019 - Test 2-Geography


Exam Title :
& C...
Email
Contact :

· The VVPAT functions as an independent verification system for EVMs and allows voters to
verify that their votes are cast as intended.

Working

· When the voter presses the button against the name of the candidate of her/his choice on the
EVM unit, the VVPAT unit generates a paper slip, also called ‘ballot slip’.

· This paper slip contains the name, serial number, and symbol of the chosen candidate.

· The voter can see this slip through a screened window where it stays for seven seconds and
then it automatically gets cut and falls into a sealed drop box. Thus, the ballot slip neither goes
into the hands of the voter nor others get to see it.

· The Ballot Slip can only be accessed by the polling officers in the rarest of the rare cases.

· It gives the voter an opportunity to challenge her/his vote on the basis of the paper receipt for
the first time.

DO YOU KNOW?

· Election Commission of India is mandated under Article 324 to 329 of the Constitution of India
to provide superintendence, direction and control for conduct of elections in accordance with
the law in force and not for catering to any speculation or debate.

THINK!

· NOTA

· First-past-the-post (FPTP) electoral system

· Single non-transferable vote

Source: http://pib.nic.in/PressReleseDetail.aspx?PRID=1540055

QUESTION 69. MTU5Mzg0K0pheStqYXlhZGhlZXIuc3llQGdtYWlsLmNvbStRVUVTVElPTiA2OA==


Montague-Chelmsford proposals were related to

a) Social reforms
b) Constitutional reforms
c) Educational reforms
d) Imposition of restrictions on freedom of the press
Correct Answer: B
Answer
Explanation

Solution (b)

Montagu-Chelmsford Report (MCR)

News: 100th year of the publication of the ‘Report on Indian constitutional reforms’,

IASbaba
Score:
Web:
Email:
Page 60 150
https://telegram.me/UPSCMaterials https://telegram.me/FreeUPSCMaterials https://telegram.me/MaterialforExam

2019 - Test 2-Geography


Exam Title :
& C...
Email
Contact :

About

· It was introduced to introduce self-governing institutions gradually to India.

· Edwin Montagu, then Secretary of State for India, had advocated for increased participation of
Indians in the British Indian administration and had begun consultations nearly a year earlier.

· It drew up a report, with the help of Bhupendra Nath Bose, Lord Donoghmore, William Duke
and Charles Roberts.

· Montagu and the then Governor-General, Lord Chelmsford, published the MCR in 1918.

· It advocated the need “to emancipate the local governments and legislatures from central
control; and to advance, by successive stages, in the direction of conferring responsible
government on the provinces.”

· It established the framework for devolution of powers and gave credence to the cry for self-
governance.

· The key principles of responsible government, self-governance and federal structure grew out
of these reforms.

THINK!

· Government of India Act, 1919

Source: https://www.thehindu.com/opinion/op-ed/indias-magna-carta/
article24513886.ece

QUESTION 70. MTU5Mzg0K0pheStqYXlhZGhlZXIuc3llQGdtYWlsLmNvbStRVUVTVElPTiA2OQ==


The area known as ‘Golan Heights’ sometimes appears in the news in the context of the events
related to

a) Central Asia
b) West Asia
c) South-East Asia
d) Central Africa
Correct Answer: B
Answer
Explanation

Solution (b)

It is a region in the Levant. The western two-thirds of this region are currently occupied by
Israel, whereas the eastern third is controlled by Syria.

Source: https://www.thehindu.com/news/international/syrian-white-helmet-members-
flee-to-jordan/article24488994.ece

IASbaba
Score:
Web:
Email:
Page 61 150
https://telegram.me/UPSCMaterials https://telegram.me/FreeUPSCMaterials https://telegram.me/MaterialforExam

2019 - Test 2-Geography


Exam Title :
& C...
Email
Contact :
QUESTION 71. MTU5Mzg0K0pheStqYXlhZGhlZXIuc3llQGdtYWlsLmNvbStRVUVTVElPTiA3MA==
Consider the following statements about ‘LiDAR’

1. It uses radio waves and sound waves to detect objects and determine their range, angle, and/
or velocity

2. LIDAR pulses are unaffected by atmospheric weather conditions

Select the correct statements

a) 1 Only
b) 2 Only
c) Both 1 and 2
d) Neither 1 nor 2
Correct Answer: D
Answer
Explanation

Solution (d)

LiDAR (Light Detection and Ranging)

News: National Institute of Wind Energy (NIWE), an autonomous institution under the Ministry
of New and Renewable Energy has installed a remote sensing instrument- LiDAR for
assessment of offshore wind resource at Gulf of Khambat.

About

· It is a remote sensing method that uses light in the form of a pulsed laser to measure ranges
(variable distances) to the Earth.

· These light pulses—combined with other data recorded by the airborne system— generate
precise, three-dimensional information about the shape of the Earth and its surface
characteristics.

· A LIDAR instrument principally consists of a laser, a scanner, and a specialized GPS receiver.

· Airplanes and helicopters are the most commonly used platforms for acquiring LIDAR data
over broad areas.

· LIDAR works well in all light conditions, but starts failing with increases in snow, fog, rain, and
dust particles in the air due to its use of light spectrum wavelengths.

Types

· Topographic LIDAR typically uses a near-infrared laser to map the land.

· Bathymetric LIDAR uses water-penetrating green light to also measure seafloor and riverbed
elevations.

THINK!

· RADAR

· SONAR

IASbaba
Score:
Web:
Email:
Page 62 150
https://telegram.me/UPSCMaterials https://telegram.me/FreeUPSCMaterials https://telegram.me/MaterialforExam

2019 - Test 2-Geography


Exam Title :
& C...
Email
Contact :

Source: http://pib.nic.in/PressReleseDetail.aspx?PRID=1539271

QUESTION 72. MTU5Mzg0K0pheStqYXlhZGhlZXIuc3llQGdtYWlsLmNvbStRVUVTVElPTiA3MQ==


Consider the following statements about ‘Arctic Council’

1. All the Scandinavian countries are members of the council

2. Observer status in the Arctic Council is open to non-Arctic states

Select the correct statements

a) 1 Only
b) 2 Only
c) Both 1 and 2
d) Neither 1 nor 2
Correct Answer: C
Answer
Explanation

Solution (c)

Arctic Council

It is the leading intergovernmental forum promoting cooperation, coordination and interaction


among the Arctic States, Arctic indigenous communities and other Arctic inhabitants on
common Arctic issues, in particular on issues of sustainable development and environmental
protection in the Arctic.

Member States – Canada, Denmark, Finland, Iceland, Norway, Russia, Sweden and United
States

Observer status in the Arctic Council is open to non-Arctic states, along with
intergovernmental, inter-parliamentary, global, regional and non-governmental organizations
that the Council determines can contribute to its work. Arctic Council Observers primarily
contribute through their engagement in the Council at the level of Working Groups. India is an
observer state.

Scandinavian Countries – Denmark, Norway, Sweden, Finland, and Iceland.

Source: https://www.thehindu.com/sci-tech/energy-and-environment/india-to-expand-
polar-research-to-arctic-as-well/article24463607.ece

QUESTION 73. MTU5Mzg0K0pheStqYXlhZGhlZXIuc3llQGdtYWlsLmNvbStRVUVTVElPTiA3Mg==


Which of the following committees have been set up to suggest measures and legal framework
to effectively deal with incidents of mob violence and lynching?

a) Rajiv Gauba Committee


b) Bezbaruah Committee

IASbaba
Score:
Web:
Email:
Page 63 150
https://telegram.me/UPSCMaterials https://telegram.me/FreeUPSCMaterials https://telegram.me/MaterialforExam

2019 - Test 2-Geography


Exam Title :
& C...
Email
Contact :

c) Ratan Watal Committee


d) Ashok Dalwai Committee
Correct Answer: A
Answer
Explanation

Solution (a)

Source: https://www.thehindu.com/news/national/centre-constitutes-high-level-
committees-to-deal-with-mob-violence-lynching/article24496480.ece

QUESTION 74. MTU5Mzg0K0pheStqYXlhZGhlZXIuc3llQGdtYWlsLmNvbStRVUVTVElPTiA3Mw==


Which of the following sectors registered highest FDI flow in 2017-18?

a) Computer Software & Hardware


b) Services Sector
c) Telecommunications
d) Construction (Infrastructure) Activities
Correct Answer: B
Answer
Explanation

Solution (b)

IASbaba
Score:
Web:
Email:
Page 64 150
https://telegram.me/UPSCMaterials https://telegram.me/FreeUPSCMaterials https://telegram.me/MaterialforExam

2019 - Test 2-Geography


Exam Title :
& C...
Email
Contact :

* Services sector includes Financial, Banking, Insurance, Non-Financial / Business,


Outsourcing, R&D, Courier, Tech. Testing and Analysis

Source: https://www.thehindu.com/todays-paper/tp-national/karnataka-sees-300-jump-
in-fdi-inflows-tn-rebounds/article24499216.ece

QUESTION 75. MTU5Mzg0K0pheStqYXlhZGhlZXIuc3llQGdtYWlsLmNvbStRVUVTVElPTiA3NA==


Consider the following statements about Fair and Remunerative Price (FRP)

1. Under the FRP system, the farmers are not required to wait till the end of the season or for
any announcement of the profits by sugar mills or the Government.

2. FRP is approved by the Cabinet Committee on Economic Affairs (CCEA)

Select the correct statements

IASbaba
Score:
Web:
Email:
Page 65 150
https://telegram.me/UPSCMaterials https://telegram.me/FreeUPSCMaterials https://telegram.me/MaterialforExam

2019 - Test 2-Geography


Exam Title :
& C...
Email
Contact :

a) 1 Only
b) 2 Only
c) Both 1 and 2
d) Neither 1 nor 2
Correct Answer: C
Answer
Explanation

Solution (c)

FRP is the minimum price that the sugar mills have to pay to farmers. It is supposed to signal to
farmers the need to plant more or less cane for the coming year.

Under the FRP system, the farmers are not required to wait till the end of the season or for any
announcement of the profits by sugar mills or the Government. The new system also assures
margins on account of profit and risk to farmers, irrespective of the fact whether sugar mills
generate profit or not and is not dependent on the performance of any individual sugar mill.

In order to ensure that higher sugar recoveries are adequately rewarded and considering
variations amongst sugar mills, the FRP is linked to a basic recovery rate of sugar, with a
premium payable to farmers for higher recoveries of sugar from sugarcane.

It is approved by the Cabinet Committee on Economic Affairs.

Source: https://www.thehindubusinessline.com/economy/agri-business/govt-hikes-
sugarcane-frp-by-rs-20/article24450995.ece

IASbaba
Score:
Web:
Email:
Page 66 150
https://telegram.me/UPSCMaterials https://telegram.me/FreeUPSCMaterials https://telegram.me/MaterialforExam

2019 - Test 2-Geography


Exam Title :
& C...
Email
Contact :
Review in Hindi
QUESTION 1.
�बग�ब�ग��स�ांत�के �बारे�म���न�न�ल�खत�कथन��पर��वचार�क��जए
:

1. यह�हाइ�ोजन�व�ही�लयम�क��सौर�नीहा�रका�से�पृ�वी�क��उ�प���क���ा�या�करता�है।�

2. इसके �अनुसार���ांड�को��न�म�त�करने�वाला����एक�लघु�ग�द�के ��प�म��अ��त�व�म��था��जसका�आयतन�अ�य�धक�कम�तथा�तापमान�एवं


घन�व�अनंत�थे।�

3. इस��स�ांत�का��योग�कर���ांड�के ��व�तार�क���ा�या�क��जा�सकती�है।�

सही�कू ट�का�चुनाव�क��जए�

a) के वल�1 और�2
b) के वल�2 and 3
c) के वल�1 और�3
d) उपयु���सभी�
Correct Answer: B
AnswerExplanation

Solution (b)

��ांड�क��उ�प���के �संबंध�म���बग�ब�ग��स�ांतसबसे
� �लोक��य�तक� �है।�इसे�व�ता�रत�होते
� ���ांड�क��प�रक�पना�भी�कहा�जाता�
है। एड�वन�हबल�ने�1920 म���माण��दया��क���ांड�का��व�तार�हो�रहा�है�।�समय�के �साथ�आकाशगंगाएं�एक-�सरे�से��र�होती�जाती�
ह�।�
-
�बग�ब�ग��स�ांत���ांड�के ��वकास�म���न�न�ल�खत�चरण�स��म�लत�है
(i) आरंभ�म��, ��ांड�का��नमा�ण�करने�वाले�सभी�पदाथ��एक��थान�पर�एक�“ छोट��ग�द�” ( एकल�अणु�) के ��प�म��मौजूद�थे��जनका
आयतन�अ��या�शत��प�से�कम�तथा�तापमान�एवं�घन�व�अनंत�थे।�

(ii) �बग�ब�ग�क��घटना�के �समय�“ छोट��ग�द�” म��तेजी�से��व�फोट��आ।�इससे�काफ��बड़ा��सार��आ।�आज�यह�बात�सामा�यत: �वीकार�क�


जाती�है��क��बग�ब�ग�क��घटना�आज�से�13.7 �ब�लयन�वष��पूव��घ�टत��ई।�यह��सार�आज�भी�जारी�है।�जैसे-जैसे�यह�बड़ा��आ, कु छ�उजा�
पदाथ��म��बदल�गई�। �बग�ब�ग�के �घ�टत�होने�के �प�ात�सेकं ड�से�भी�कम�समय�के �भीतर�ती���व�तार��आ� ।�त�प�ात�, �सार�क��ग�त
धीमी�पड़�गई।��बग�ब�ग�घटना�के �तीन��मनट�के �भीतर�पहले�अणु�ने�आकार�लेना�आरंभ�कर��दया।�

(iii) �बग�ब�ग�के 300,000


� वष��के �भीतर�, तापमान�4,500 के ��वन�तक��गर�गया�और�इसने�आ��वक�पदाथ��को�ज�म��दया।���ांड
पारदश��बन�गया।�

��ांड�के ��व�तार�का�अथ��है�आकाशगंगा��के �बीच�क���री�बढ़�जाना� ।�होयले�क����थर�अव�था�क��अवधारणा�इसका��वक�प�है।�इस


का�मानना�था��क���ांड��कसी�भी�समय��ब���पर�लगभग�उतने�ही�आकार�का�रहा�है।�हालां�क, ��ांड�के ��व�तार�से�संबंधी�और�बड़े��माण��के
सामने�आने�से�वै�ा�नक�समुदाय�आज���ांड�के ��व�तार�के �तक� �का�समथ�न�करता�है।�

�या�आप�जानते�ह��?

आप���ांड�के ��व�तार�क��बात�को��न�न�ल�खत��योग�से�जाँच�सकते�ह�-

एक�गु�बारा�ली�जए�और�आकाशगंगा��के �नाम�से�उस�पर�कु छ��ब����न�म�त�क��जए।�अब , य�द�आप�गु�बारे�को�फु लाना�आरंभ�कर�गे�तो�उस


पर�बनाए�गए��ब���एक��सरे�से��र�जाते��दखाई�द� गे।�इसी��कार, आकाशगंगा��के �बीच�क���री�भी�बढ़�रही�है�तथा�इस�लए�माना�जाता�है��क
��ांड�का��व�तार�हो�रहा�है।�हालां�क�, आप�दे ख�गे��क�गु�बारे�के ��ब����क���री�बढ़ने�के �अ�त�र���ब���भी�बड़े�हो�रहे�ह�।�यह�बात�उपरो�
त�य�से�सा�यता�नह��रखती।�वै�ा�नक��का�मानना�है��क�य��प�आकाशगंगा��के �बीच�क���री�बढ़�रही�है , पय�वे�ण��से�आकाशगंगा��के �बड़े
होने�का��माण�नह���मलता।�अत: गु�बारे�का�उदाहरण�अंशत: ही�सही�है।�

QUESTION 2.

IASbaba
Score:
Web:
Email:
Page 67 150
https://telegram.me/UPSCMaterials https://telegram.me/FreeUPSCMaterials https://telegram.me/MaterialforExam

2019 - Test 2-Geography


Exam Title :
& C...
Email
Contact :

�न�न�ल�खत�म��से�कौन�इस�त�य�क��सही��ा�या�करता�है��क�आंत�रक��ह�च�ानी�एवं�बाहरी��ह�अ�धकतर�गैसीय�ह��?

1. पा�थ�व��ह�द�घ��ह��और�उनका��न�न�गु��वाकष�ण�पलायन�करने�वाली�गैस��को�रोक�नह��पाया।�

2. सूय��के ��नकट�सौर
-पवन��सवा��धक�श��शाली�थी।�अत: ये�गैस��और�धूल�को�पा�थ�व��ह��से�उड़ा�ले�गई।�

सही�कू ट�का�चुनाव�क��जए�

a) के वल�1
b) के वल�2
c) 1 और�2 दोन��
d) उपरो��म��से�कोई�नह��
Correct Answer: B
Answer
Explanation

Solution (b)

हमारे�सौर�मंडल�म��सूय�� -कण��व�गैस�क���वशाल�रा�श
पाई�जाती�है।�आठ��ह��म� ( �तारा�),
से�बुध, 8शु� , पृ�63
�ह�, वी�और�मं
चं�मा�,गल�को�आं त�रक��ह�कहा�जाता�है
लाख��छोट��सं ��य��क�वेम�सू
रचनाएं�जैसे��ु��ह�तथा�धू केय��व�धूतुम�एवं
के �धूल तु��क��मेखला�के �बीच���थत
ह��जब�क�शेष�चार��ह��को�बाहरी��ह�कहा�जाता�है।�

वैक��पक��प�से, पहले�चार��ह��को�पा�थ�व�अथा�त�पृ�वी�जैसे��ह�कहा�जाता�है��य��क�ये�च�ान�और�धातु��के �बने�अपे


ह��और�
�ाकृ त
अ�धक�घन�व�वाले�ह�।�बाक��चार�को�जो�वयन�अथवा�गैस�जायंट��ह�कहा�जाता�है।�जो�वयन�का�अथ��है�बृह�प�त�जैसा।�इनम��से�अ�धकाँश
पा�थ�व��ह��से�काफ��बड़े�ह��और�इनका�वातावरण�घना�है�जो�ही�लयम�और�हाइ�ोजन�का�बना�है।�

�न�न�ल�खत�प�र��थ�तयां�पा�थ�व�और�जो�वयन��ह��के �अंतर�के ��लए�उ�रदायी�ह�


’:

(i) पा�थ�व��ह�अपने��पतृ�तारे�के �काफ���नकट��न�म�त��ए�थे�जहाँ


गैस� ��के �संघनन��ारा�ठोस�कण��म��प�रव�त�त�होने�के ��लहाज�से�काफ�
गम��थी�।�जो�वयन��ह��का��नमा�ण�एक��र�थ��थान�पर��आ।�

(ii) सूय��के ��नकट�सौर�पवन��काफ���चंड�थी


; अत: ये�अनेक��गैस��और�धूल�को�पा�थ�व��ह��से��र�बहा�ले�गई�।��सरी�ओर, जो�वयन��ह�
के ��नकट�सौर�पवन��इतनी��चंड�नह��थी��क�वे�गैस��को�इसी��कार�वहाँ�से�हटा�सक� ।�

(iii) पा�थ�व��ह�छोटे �ह��और�उनका�कम�गु��वाकष�ण�पलायन�करने�वाली�गैस��को�रोककर�नह��रख�पाया।�

�या�आप�जानते�ह��?

आरं�भक�सौरमंडल�का�तापमान��ा�या�करता�है��क��य��आंत�रक��ह�च�ानयु��और�बाहरी��ह�गैसीय�ह�।��य�ही��ोटोसन�के ��नमा�ण�के ��लए


गैस��एक�भूत��ई, सौरमंडल�का�तापमान�बढ़�गया।�

आंत�रक�सौरमंडल�म��तापमान�2000 के ��वन��जतना�अ�धक�था�जब�क�बाहरी�सौरमंडल�म��यह�मा��50 के ��वन��जतना�ही�था।�आंत�रक


सौरमंडल�म��के वल�उ�च�गलनांक��ब���वाले�पदाथ��ही�ठोस�बने�रह�सकते�थे।�बाक��सभी�को�वा�पीकृ त�हो�जाना�था।�अत
: आंत�रक�सौरमंडल
क��व�तुए�ं लौह, �स�लकॉन, मै�नी�शयम, स�फर, ए�युमी�नयम, कै ��शयम�और��नकल�क��बनी�थी।�इनम��से�अनेक�ऑ�सीजन�के �साथ�यौ�गक
के ��प�म��मौजूद�थी।�ठोस�अव�था�म��अ�य��कार�के �त�व��क��मा�ा�इतनी�नह��थी��क�वे�आंत�रक��ह��का��नमा�ण�कर�पाते।�

आंत�रक��ह�बाहरी��ह��क��अपे�ा�काफ��छोटे �ह��और�इस�कारण�उनका�गु��वाकष�ण�अपे�ाकृ त��प�से�कम�है।�इसी�कारण�वे�अपने


वातावरण�म��गैस�क��बड़ी�मा�ा�को�आक�ष�त�करने�म��स�म�नह��थे।�सौरमंडल�के �बाहरी�भाग��म��जहाँ�यह�ठं डा�था
, पानी�और�मीथेन�जैसे�अ�य
त�व�वा�पीकृ त�नह���ए�और�बड़े��ह��का��नमा�ण�करने�म��स�म�बन�पाए।�ये��ह�आंत�रक��ह��क��अपे�ा�काफ��बड़े�थे�तथा�हाइ�ोजन�और
ही�लयम�क��बड़ी�मा�ा�को�आक�ष�त�करने�म��स�म�थे।�यही�कारण�है�ये�मु�यत: हाइ�ोजन�और�ही�लयम�के �बने�ह��जो��क�सौरमंडल�और
��ांड�का�सबसे��चुर�त�व�है।�

QUESTION 3.
�न�न�ल�खत�म��से�कौन�वत�मान�वायुमंडल�के ��नमा�ण�अथवा�उसमे�प�रवत�न�से�संबं�धत�है
? �

IASbaba
Score:
Web:
Email:
Page 68 150
https://telegram.me/UPSCMaterials https://telegram.me/FreeUPSCMaterials https://telegram.me/MaterialforExam

2019 - Test 2-Geography


Exam Title :
& C...
Email
Contact :

a) �वगैसीयकरण�
b) सौर�पवन��
c) �वभेदन�
d) a और�b दोन��
Correct Answer: D
AnswerExplanation

Solution (d)

पृ�वी�का�वत�मान�वायुमंडल�मु�यत: नाइ�ोजन�और�ऑ�सीजन�से��न�म�त�है।�

वत�मान�वायुमंडल�के ��वकास�के �तीन�चरण�ह�


-

पहले�चरण�म��इसके आरं
� �भक�वायुमंडल�का��य��आ।�

��तीय�चरण�म�, पृ�वी�के �गम��आंत�रक�भाग�ने�वायुमंडल�के ��वकास�म��योगदान��दया।�

अंतत:, वायुमंडल�क��संरचना�को�जै�वक�जगत�ने��काश-सं�ेषण�क�����या��ारा�बदल��दया�।�आरं�भक�वायुमंडल��जसम�
हाइ�ोजन�और�ही�लयम�थी, सौर�पवन��के �कारण�समा�त�हो�गया।�ऐसा�न�के वल�पृ�वी�के �साथ��आ�अ�पतु�सभी�पा�थ�व��ह��के �साथ�भी�ऐसा
ही��आ।�

�या�आप�जानते�ह��?

पृ�वी�के �ठं डा�होने�के �दौरान


, इसके �ठोस�आंत�रक�भाग�से�गैस��और�जलवा�प��नमु����ए�थे।�इससे�वत�मान�वायुमंडल�का��वकास�आरंभ��आ।
आरं�भक�वायुमंडल�म��अ�धकांशतया�जलवा�प, नाइ�ोजन, काब�न�डाइऑ�साइड, मीथेन, अमो�नया�और�अ�यंत�अ�प�मा�ा�म��मु�
अव�था�म��ऑ�सीजन�मौजूद�थी।�

आंत�रक�भाग�से�गैस��के �बाहर��नकलने�क�����या�को��वगैसीयकरण�कहते�।��नरं ह�� तर�होने�वाले��वालामुखीय�उदगार��ने�वायुमंडल�म�


जलवा�प�और�गैस��का�योगदान��दया।�जैसे-जैसे�पृ�वी�ठं डी��ई, मु���ई�जलवा�प�संघ�नत�होनी�आरंभ�हो�गई।�

वायुमंडल�क��काब�न�डाइऑ�साइड�वषा�जल�म��घुल�गई�तथा�तापमान�और�बढ़�गया��जससे�और�अ�धक�संघनन�व�वषा���ई।�सतह�पर
पड़ने�वाला�जल�ग���म��जमा�होना�शु��हो�गया�और�इस��कार�महासागर��का��नमा�ण��आ।�

पृ�वी�पर�महासागर��का��नमा�ण�पृ�वी�बनने�के 500
� �म�लयन�वष��के �भीतर�हो�गया�था।�इससे�हम��पता�लगता�है��क�महासागर�4,000 �म�ल
यन�वष��पुराने�ह�।�लगभग�3,800 �म�लयन�वष��पूव�, जीवन�क��उ�प���आरंभ��ई।�हालां�क, आज�से�लगभग�2,500-3,000 �म�लयन�वष�
पूव���काश-सं�ेषण�क�����या��वक�सत��ई।�लंबे�समय�तक�जीवन�महासागर��तक�सी�मत�रहा।��काश-सं�ेषण�के �मा�यम�से
महासागर��ने�ऑ�सीजन��नमा�ण�म��योगदान�दे ना�आरंभ�कर��दया।�अंतत: महासागर�ऑ�सीजन�से�संतृ�त�हो�गए�तथा�2,000 �म�लयन�वष�
पूव��ऑ�सीजन�ने�वायुमंडल�को��ला�वत�करना�शु��कर��दया।�

QUESTION 4.
�न�न�ल�खत�म��से�कौन�से�कथन�पृ�वी�क���कृ �त�के �संदभ��म��सही�ह�
? �

1. पृ�वी�के �आंत�रक�भाग�को�के वल�अ��य����ोत��के �मा�यम�से�पूण���प�से�समझा�जा�सकता�है।�

2. ब�हज��नत�बल�ही�भू��य�को��नरंतर�आकार�दे ने�हेतु�उ�रदायी�ह��न��क�अंतज��नत�बल�जो��क�कम��बल�ह�।�

सही�कू ट�का�चुनाव�क��जए�

a) के वल�1
b) के वल�2
c) 1 और�2 दोन��
d) उपरो��म��से�कोई�नह��

IASbaba
Score:
Web:
Email:
Page 69 150
https://telegram.me/UPSCMaterials https://telegram.me/FreeUPSCMaterials https://telegram.me/MaterialforExam

2019 - Test 2-Geography


Exam Title :
& C...
Email
Contact :
Correct Answer: A
AnswerExplanation

Solution (a)

पृ�वी�के �आंत�रक�भाग�को�के वल�अ��य����ोत��के �मा�यम�से�सही��कार�समझा�जा�सकता�है��य��क�न�तो�अब�तक�कोई�पृ�वी�के


आंत�रक�भाग�तक�प�ँचा�है�और�न�ही�प�ँच�सकता�है।�

पृ�वी�क��सतह�का��व�यास��ापक��प�से�पृ�वी�के �आंत�रक�भाग�म��होने�वाली����या��का�प�रणाम�है।�

ब�हज��नत�के �साथ -साथ�अंतज��नत����याएं��नरंतर�भू��य�का��नमा�ण�कर�रही�ह�।�अंतज��नत����या��के ��भाव��को�जाने��बना��कसी


�े��क���ाकृ �तक�भूगोलीय��वशेषता��नह��समझा�जा�सकता।�मानव�जीवन��ापक��प�से��े��क���ाकृ �तक�भूगोलीय��वशेषता���ारा
�भा�वत�होता�है।�

QUESTION 5.
�न�न�ल�खत�म��से�कौन�पृ�वी�क��आंत�रक�संरचना�के �संबंध�म��जानकारी�का�अ��य����ोत�है
? �

a) भूकं पीय�तरंग��
b) गु��वाकष�ण�
c) चुंबक�य��े��
d) उपयु���सभी�
Correct Answer: D
AnswerExplanation

Solution (d)

पदाथ��क���वशेषता��का��व�े�ण�परो���प�से�आंत�रक�भाग�के �बारे�म��सूचना��दान�करता�है �
। खनन�ग�त�व�धय��क े �मा�यम�से�
हम�जानते�ह���क�पृ�वी�क��सतह�से�नीचे�गहराई�म��जाने�पर�तापमान�और�दबाव�बढ़ता�है।�साथ�हह,ी य �भी�पता�चलता�है��क�गहराई�बढ़ने�के �साथ
चीज��का�घन�व�भी�बढ़ता�है।�इन��वशेषता��म��प�रवत�न�क��दर�को�जानना�भी�संभव�है।�

सूचना�का�अ�य���ोत�पृ�वी�पर�समय-समय�पर��गरने�वाले�उ�का�प�ड�ह��।�हालां�क, उ�का�प�ड��से�उपल�ध�साम�ी�पृ�वी�के �आंत�रक�भाग


से�नह��होती।�उ�का�प�ड��क��साम�ी�और�संरचना�पृ�वी�के �समान�होती�है।�वे�ठोस�पदाथ��होते�ह��और�उसी�साम�ी�के �बने�होते�ह���जससे�हमारा
�ह�बना�है।�इस��कार, ये�भी�पृ�वी�के �आंत�रक�भाग�क��सूचना�का���ोत�होते�ह�।�

अ�य�परो����ोत��म��गु��व, चुंबक�य��े��व�भूकं पीय�ग�त�व�ध� स��म�लत�ह�।�गु��वाकष�ण�बल�सतह�पर��व�भ��अ�ांश��पर�समान


नह��है।�यह��ुव��पर�अ�धक�और��वषुवत�रेखा�पर�कम�है।�इसका�कारण��वषुवत�रेखा�पर��ुव��क��अपे�ा��री�का�अ�धक�होना�है�।�गु��व
मान�भी�साम�ी�के ���मान�के �साथ�बदलता�है।�पृ�वी�के �भीतर�साम�ी�के ���मान�का�असमान��वतरण�इस�मान�को��भा�वत�करता�है।��व�भ�
�थान��पर�गु��व�का�मान�कई�अ�य�कारक���ारा��भा�वत�होता�है।�ये�मान�अपे��त�मू�य��से�अलग�होते�ह�।�ऐसे�अंतर�को�गु��व�असंग�त�कहा
जाता�है।�गु��व�असंग�तयां�हम��भूपप�ट��पर�साम�ी�के ���मान�के ��वतरण�के �बारे�म��सूचना�दे तचुी�ह� ।�
ंबक�य�सव� �ण�भी�पप�ट��के �भाग
पर�चुंबक�य�साम�ी�के ��वतरण�के �बारे�म��सूचना��दान�करते�ह��और�इस��कार�इस�भाग�म��साम�ी�के ��वतरण�क��सूचना�दे
भूत
के�ं ह�पीय�ग�त
।�
�व�ध�पृ�वी�के �आंत�रक�भाग�के �बारे�म��सूचना�का�एक�अ�यंत�मह�वपूण����ोत�है।�

�या�आप�जानते�ह��?

��य����ोत�

सबसे�आसानी�से�उपल�ध�ठोस�पा�थ�व�पदाथ��ह��सतही�च�ान�अथवा�खनन��े���से��ा�त�च�ान��।�द. अ��का�म����थत��वण��खदाने�3 - 4
�कमी. गहरी�ह�।�इस�गहराई�से�आगे�जाना�संभव�नह���य��क�इस�गहराई�पर�अ�यंत�गम��होती�है।�

खनन�के �अ�त�र�, वै�ा�नक��ने�पप�ट��के �भाग��को�जानने�के �उ�े �य�से�गहराई�तक�भेदन�करने�क��अनेक��प�रयोजनाएं�शु��क��ह�।��व��भर�क


वै�ा�नक�दो�बड़ी�प�रयोजना��पर�काय��कर�रहे�ह�- “ डीप�ओ�सयन����ल�ग��ोजे�ट�” और�“ इंट��ेटेड�ओ�सयन����ल�ग�
�ोजे�ट
�”

IASbaba
Score:
Web:
Email:
Page 70 150
https://telegram.me/UPSCMaterials https://telegram.me/FreeUPSCMaterials https://telegram.me/MaterialforExam

2019 - Test 2-Geography


Exam Title :
& C...
Email
Contact :
आक� �टक�के �कोला�म���कया�गया�सबसे�गहरा��छ��अब�तक�12 �कमी�. क��गहराई�तक�प�ँच�चुका�है।�यह�तथा�अनेक��गहरी����ल�ग
प�रयोजनाएं��व�भ��गहराइय��से��ा�त�साम��य��के �मा�यम�से�काफ��सारी�सूचनाएं�उपल�ध�करवा�चुक��ह�।�

�वालामुखीय�उदगार���य��सूचना�का�एक�और���ोत�ह�।�जब�कभी�भी��पघला�पदाथ��( मै�मा�) �वालामुखीय�उदगार�के �समय�पृ�वी�क�


सतह�पर�आता�है, यह��योगशाला�म���व�े�ण�के ��लए�उपल�ध�हो�जाता�है।�हालां�क
, ऐसे�मै�मा�के ���ोत�क��गहराई�को��न��त�करना
क�ठन�है।�

QUESTION 6.
पृ�वी�क��संरचना�के �संदभ��म���न�न�कथन��पर��वचार�क��जए
:

1. महासागरीय�पप�ट��क��मोटाई�महा��पीय�पप�ट��से�अ�धक�है।�

2. पप�ट��का�ऊपरी�भाग��ब�लतामंडल�कहलाता�है�और�यही�मै�मा�का�मु�य���ोत�है।�

गलत�कथन�का�चुनाव�क��जए�

a) के वल�1
b) के वल�2
c) 1 और�2 दोन��
d) उपरो��म��से�कोई�नह��
Correct Answer: C

Answer

Explanation

Solution (c)

नोट�: गलत�कथन�पहचानने�को�कहा�गया�है।�

पप�ट�� ।�यह�भुरभुरे��वभाव�का�है।�इसक��मोटाई�महासागरीय�एवं�महा��पीय��े���म���भ
भ�-� �
होती�है।�महासागरीय�पप�ट��एवं�महा��पीय�पप�ट��क��तुलना�म��पतली�होती�है�।�महासागरीय�पप�ट��क��औसत�मोटाई�5 �कमी�है�जब�क
यह�पृ�वी�के �ठोस�भाग�का�सबसे
महा��पीय�पप� �बाहरी�भाग�है
ट��क��मोटाई�लगभग�30 �
�कमी�. है।�महा��पीय�पप� ट��मु�य�पव�त��णा�लय��वाले��थान��पर�अ�धक�मोट��है।��हमालयी��े��म�
इसक��मोटाई�70 �कमी�तक�है।�यह�भारी�च�ान��से��न�म�त�है��जसका�घन�व�3 g/cm3 है।�महासागरीय�पप�ट��म��पाई�जाने�वाली�ऐसी
च�ान�बेसा�ट�है।�इसका�अथ��है��क�महासागरीय�पप�ट��म��साम�ी�का�औसत�घन�व�2.7 g/cm3 है।�

मै�टल�

पप�ट��से�आगे�का�भाग�मै�टल�कहलाता�है�। मै�टल�मोहो�असंब�ता�से�लेकर�2,900 �कमी�. क��गहराई�तक��व�तृत�है।�मै�टल�का


ऊपरी�भाग��ब�लतामंडल�कहलाता�है।�ए�थेनो�श�द�का�अथ��है- कमजोर।�इसका��व�तार�400 �कमी�. तक�माना�जाता�है।�यह�मै�मा�का
मु�य���ोत�है�जो��वालामुखीय�उदगार��के �समय�पृ�वी�क��सतह�पर�प�ँचता�है।�इसका�घन�व�पप�ट��के �घन�व�
(3.4 g/cm3) से�अ�धक�है।�

पप�ट��और�मै�टल�का�सबसे�ऊपरी�भाग��थलमंडल�कहलाता�है�।�इसक��मोटाई�10-200 �कमी�. तक�है।�मै�टल�का��नचला�भाग


�ब�लतामंडल�से�भी�आगे�तक��व�तृत�है।�यह�ठोस�अव�था�म��है।�

�ोड�

भूकं प�तरंग��के �वेग�ने�पृ�वी�के ��ोड़�के �अ��त�व�को�समझने�म��मदद�क�।�


�ोड़�- मै�टल�सीमा�2,900 �कमी�. क��गहराई�पर���थत�है।�
बाहरी��ोड़�तरल�अव�था�म��है�जब�क�आंत�रक��ोड़�ठोस�अव�था�म�।��ोड़�- मै�टल�सीमा�पर�साम�ी�का�घन�व�लगभग�5 g/cm3 है
और�पृ�वी�के �क� ��म�6,300
� km क��गहराई�पर�घन�व�का�यह�मान�लगभग�13g/cm3 है।��ोड़�भारी�धातु��का�बना�है��जसम��अ�धकां
शत: �नकल�और�लौह�है।�इसे�सामा�यत: �नफे �परत�कहा�जाता�है।�

IASbaba
Score:
Web:
Email:
Page 71 150
https://telegram.me/UPSCMaterials https://telegram.me/FreeUPSCMaterials https://telegram.me/MaterialforExam

2019 - Test 2-Geography


Exam Title :
& C...
Email
Contact :

QUESTION 7.
अ��े ड�वेगेनर��ारा���ता�वत�महा��पीय��व�थापन��स�ांत�के ��माण��के ��प�म���न�न�ल�खत�म��से��क�ह��बताया�गया�था�
?

a) महासागर�के �दोन��ओर�च�ान��क��आयु�म��समानता�
b) महा��पीय��कनार��का��ु�टर�हत�सा�य�
c) म�य�महासागरीय�कटक�
d) a और�b दोन��
Correct Answer: D
AnswerExplanation

Solution (d)

अ��े ड�वेगेनर�— जो��क�एक�जम�न�मौसम��व�ानी�थे, ने�1912 म��“ महा��पीय��व�थापन: महासागर��और�महा��प��का��वतरण�


अ�याय
�स�ांत�” के ��प�म��एक��ापक�तक� ���तुत��कया।�यह�महासागर��और�महा��प��के ��वतरण�से�संबं�धत�था।�

वेगेनर�के �अनुसार�
, सारे�महा��प�भू�म�के �एकल�महा��प�के ��प�म���व�मान�थे�और�एक�वृहत�महासागर�से��घरे��ए�थेइस��वशाल�महा��प
।�
का�नाम�प��जया�था��जसका�अथ��है�संपूण��धरती।��वशाल�महासागर�का�नाम�प�थलांसा�था��जसका�अथ��है�संपूण��जल।�

छोटे -छोटे
उ�ह�ने �तक �उन�टु कड़��म��टू टना�जारी�रहा��ज�ह�
� ��दया��क�लगभग� 200 �म�लयन�वष� �हम�आज�दे खते�ह�।�
�पूव��, यह��वशाल�महा��प, प��जया�टू टना�आरंभ��आ।�यह�पहले�दो�भाग��नामत
: लारे�शया
और�ग�डवानाल�ड�म���वभा�जत��आ��जससे�इसके �उ�री�और�द��णी�भाग��का��नमा�ण��आ।�आगे�चलकर
IASbaba , लारे�शया�और�ग�डवानाल�ड�का
Score:
Web:
Email:
Page 72 150
https://telegram.me/UPSCMaterials https://telegram.me/FreeUPSCMaterials https://telegram.me/MaterialforExam

2019 - Test 2-Geography


Exam Title :
& C...
Email
Contact :

महा��पीय��व�थापन�के �प��म��अनेक��माण��दए�गए।�इनम��से�कु छ��न�न�ल�खत�ह�


-

महा��पीय��व�थापन�के �प��म���माण�

महा��प��का�सा�य�( �जग-शॉ-�फट�)

अ��का�और�द��ण�अमे�रका�क��आमने-सामने�क��तटरेखा��का��वल�ण�और��ु�टर�हत�सा�य�है।�

महासागर��के �दोन��ओर�समान�आयु�क��च�ान��

हाल�के �वष��म���वक�सत�क��गई�रे�डयोमे��क��त�थ��नधा�रण��व�ध�ने�पता�लगाया��क�महासागर��के �दोन��ओर�क��च�ान��क��आयु�समान�है।


रे�डयोमे��क��त�थ��नधा�रण��व�ध�ने��वशाल�महासागर�के �दोन��ओर���थत��व�भ��महा��प��क��च�ान��के ��नमा�ण�म��सहसंबंध��था�पत�करना
सु�वधाजनक�बना��दया�है।��ाजील�तट�क���ाचीन�च�ान��क��2,000 �म�लयन�वष��पुरानी�प���प��मी�अ��का�क��च�ान��से�मेल�खाती�है।
द��ण�अमे�रका�और�अ��का�क��तटरेखा�पर�आरं�भक�समु����न�ेप�जुरा�सक�काल�के �ह�।�इससे�पता�चलता�है��क�इस�समय�से�पूव��महासागर
का�अ��त�व�नह��था।�

�टलाईट�

यह��ले�शयर��के ��न�ेप�से��न�म�त�अवसाद��च�ान�है।�भारत�क
� े �अवसाद��के �ग�डवाना�तं��के �समान�द��णी�गोलाध��म�:�छ�भ�-�भ�
भूभाग�ह�।�इसके �आधार�म�
, �टलाईट�क��मोट��परत�है�जो��ापक�और�लंबी�अव�ध�तक��हमा�छादन�क��ओर�संके त�करती�है।�इस��म�के
समवत��अ��का, फाकल�ड���प, मेडागा�कर, अंटा�ट�का�और�ऑ��े �लया�म��पाए�जाते�ह�।�कु ल��मलाकर�ग�डवाना
-स�श�अवसाद��से�सा�यता
�द�श�त�करती�है��क�इन�भूभाग��का�इ�तहास�समान�था।��हमनद�य��टलाईट��ागै�तहा�सक�जलवायु�( paleoclimates ) व�महा��पीय
�व�थापन�के ��प���माण�उपल�ध�करवाते�ह�।�

�लेसर��न�ेप�

घाना�तट�पर��वण��के �समृ���लेसर��न�ेप��क��मौजूदगी�और�मूल�च�ान�क��पूण��अनुप��थ�त�अपने�आप�म��एक�आ�य�जनक�त�य�है।��वण�-
�शराएं��ाजील�म��ह��और�यह��प��है��क�घाना�के ��वण���न�ेप��ाजील�पठार�से�उस�समय��नकले�ह��जब�दोन��महा��प�एक�साथ�जुड़े�थे।�

जीवा�म��का��वतरण�

जब�पादप��और�पशु��क��वे�समान��जा�तयाँ�जो�पृ�वी�अथवा��व�छ�जल�म��रहने�हेतु�अनुकू ,�लत�ह�
दोन��ओर�पाई�जाती�ह�, तब�ऐसे��वतरण
के �कारण�के �संबंध�म��सम�या�उ�प��होती�है।�यह�पय�वे�ण��क�लैमुर�भारत, मेडागा�कर�और�अ��का�म��पाया�जाता�है, हम��यह�अनुमान�लगाने
को��ववश�करता�है��क�यह�‘लेमु�रया’ नामक�एक�ही�भूभाग�था�जो�इन�तीन��भूभाग��को�जोड़ता�था।�मेसोसॉरस�एक�छोटा�सरीसृप�था�जो
�छछले�खारे�पानी�हेतु�अनुकू �लत�था।�इसके �कं काल�के वल�दो��थान��पर�पाए�जाते
: द.�ह�अ��का�के �सदन��के प��ो�व�स�और��ाजील�के �इरावर
म�।�ये�दोन���थान�वत�मान�म��एक��सरे�से�4,800 �कमी��र���थत�ह�।�

QUESTION 8.
सागर�तल�के ��सार�क��अवधारणा�के �संबंध�म���न�न�ल�खत�म��से�कौन�सा
/से�कथन�स�य�है/ह��?

1. महासागर�के �सभी�गहरे�गत��के ��कनार��पर��वालामुखीय�उदगार�होना�सामा�य�ह��और�ये�उदगार�लावा�क���वशाल�रा�श�को�गत��से�सतह�पर


लाते�ह�।�

2. म�य�महासागरीय�कटक��के ��नकट�क��च�ान��म��चु�बक�व��ुवण�सामा�य�है�और�वे�सबसे�नवीन�ह�।�

3. महासागरीय�पप�ट��क��च�ान��महा��पीय�च�ान��क��अपे�ा�काफ��पुरानी�ह�।�

सही�कू ट�का�चुनाव�क��जए�

a) के वल�1 और�2
b) के वल�2 न�
c) के वल�2 और�3
d) के वल�1 और�3
Correct Answer: B

IASbaba
Score:
Web:
Email:
Page 73 150
https://telegram.me/UPSCMaterials https://telegram.me/FreeUPSCMaterials https://telegram.me/MaterialforExam

2019 - Test 2-Geography


Exam Title :
& C...
Email
Contact :
Answer
Explanation

Solution (b)

समु��तल�के ��सरण�क��अवधारणा�

�व�थापन�प�ात�के �अ�ययन��ने�ऐसी�पया��त�सूचनाएं�उपल�ध�करवाई�जो�वेगेनर�के �महा��पीय��व�थापन�का��स�ांत��दए�जाने�के �समय�मौजूद


नह��थी।��वशेषकर, सागर�तल�के �मान�च�ण�और�महासागरीय��े���क��च�ान��के �पे�लयोमै�ने�टक�अ�ययन��ने��न�न�त�य��का�खुलासा��कया
-

1. यह�महसूस��कया�गया��क�म�य�महासागीय�कटक�क��संपूण���री�म���वालामुखी�उदगार�आम�थे�और�वे�लावा�क���वशाल�मा�ा�को
उन��े���तक�लेकर�आए।�

2. म�य�महासागीय�कटक��के ��शखर�के �दोन��ओर�समान��री�पर���थत�च�ान��अपने�बनने�के �समय


, रासाय�नक�संघटन, और�चुंबक�य
�वशेषता��के �मामले�म��एक�अ��त�समानता��द�श�त�करट��ह�।म�य�महासागीय�कटक��क
� े ��नकट�क��च�ान��क���ुवीयता�सामा�य�है�और
वे�सबसे�युवा�ह��। �शखर�से��र�जाते�जाने�पर�च�ान��क��आयु�बढ़ती�जाती�है।�

3. सागरीय�पप�ट��क��च�ान��महा��पीय�च�ान��क��अपे�ा�काफ��युवा�ह��।�महासागरीय�पप�ट��म��च�ान��क��आयु�कह��भी�200 �म�लयन
वष��से�अ�धक�नह��है।�कु छ�महा��पीय�च�ान��ा�प�
3,200 �म�लयन�वष��तक�पुराने�ह�।�

4. महासागर�तल�के �अवसाद�अनपे��त��प�से�ब�त�पतले�ह�।�वै�ा�नक�उ�मीद�कर�रहे�थे��क�महासागरीय�तक�महा��प���जतने�ही�पुराने
ह��और�उनने�अवसाद��का��म�काफ��लंबी�अव�ध�वाला�है।�हालां�क�, �कसी�भी��थान�पर�अवसाद��तंभ�200 �म�लयन�वष��से�अ�धक�पुराना
नह��था।�

5. गहरी�गत��म��गहराई�तक�आने�वाले�भूकं पो�के �संके त�थे�


, जब�क�म�यमहासागरीय�कटक��े���म��भूकं पमूल�क��गहराई��छछली�है।�

QUESTION 9.
�लेट��ववत��नक��के �संदभ��म���न�न�ल�खत�कथन��पर��वचार�क��जए�

1. एक��ववत��नक��लेट, जो�ठोस�च�ान�का�अ�नय�मत�आकार�वाला�पृ�वी�का�खंड�है, के वल�महा��पीय��थलमंडल�से��न�म�त�होता�है।�

2. म�य�महासागरीय�कटक�अ�भसारी��लेट�सीमा�का�एक�उदाहरण�है।�

3. अ�भसारी��लेट�सीमा�से�नई�पप�ट��का��नमा�ण�होता�है।�

सही�कू ट�चु�नए�

a) के वल�1
b) के वल�2
c) 1 और�2 दोन��
d) उपरो��म��से�कोई�नह��
Correct Answer: D

Answer

Explanation

Solution (d)

�लेट��ववत��नक���स�ांत�

मै�क�जी, पाक� र�और�मॉग�न��ारा�1967 मर���ता�वत�

एक��ववत��नक��लेट�( �जसे��थलमंडलीय��लेट�भी�कहा�जाता�

है� ) एक��वशाल, अ�नय�मत��ा�प�वाली�ठोस�च�ान�का�एक�फलक�होता


IASbaba
है�जो�सामा�यत: महा��पीय�और�महासागरीय��थलमंडल�दोन��से��न�म�त�होता�है�।��लेट���ब�लतामंडल�के �ऊपर�कठोर�इकाईय��क े ��प�म
Score:
Web:


Email:�
ै तज�गमन�करती�ह� । ��थलमं ड ल�म� � पप� ट ��और�म� ट ल�का�ऊपरी��ह�सा�शा�मल�होता�है � तथा�इसक��मोटाई�महासागरीय�भाग�म� � 5 से � 100 �क
मी�के �बीच�रहती�है�और�महा��पीय�भाग�म��यह�200 �कमी�रहती�है Page ।� 74 150
https://telegram.me/UPSCMaterials https://telegram.me/FreeUPSCMaterials https://telegram.me/MaterialforExam

2019 - Test 2-Geography


Exam Title :
& C...
Email
Contact :

एक��लेट�से�आशय�महा��पीय��लेट�अथवा�महासागरीय��लेट�से�है�जो�इस�बात�पर��नभ�र�करता�है��क�दोन��म��से��कस�पर��लेट�का�बड़ा��ह�सा
��थत�है।��शांत��लेट�मु�यत: एक�महासागरीय��लेट�है�जब�क�यूरे�शयन��लेट�को�महा��पीय��लेट�कहा�जा�सकता�है�।��लेट��ववत��नक��
का��स�ांत�बताता�है��क�पृ��वी�का��थलमंडल�सात�मु�य�और�कु छ�छोट���लेट��म���वभा�जत�हैइन��ले
।� ट��के �चार��ओर�युवा�व�लत�पव�त
कटक, गत��और/अथवा��ंश�मौजूद�ह�।�

मु�य��लेट���न�न�ल�खत�ह��:

1. अंटा�ट�का�और�उसके �समीप�क��महासागरीय��लेट�

2. उ�र�अमे�रक��( �जसम��कै �र�बयाई���प��के �साथ�प��मी�अटलां�टक�तल�द��ण�अमे�रक���लेट�से�अलग�होता�है) ��लेट�

3. द��ण�अमे�रक��( �जसम��कै �र�बयाई���प��के �साथ�प��मी�अटलां�टक�तल�उ�र�अमे�रक���लेट�से�अलग�होता�है) ��लेट�

4. �शांत��लेट�

5. इंडो-ऑ��े �लया-�यूज़ीलै�ड��लेट�

6. पूव��अटलां�टक�तल�के �साथ�अ��क���लेट�

7. यूरे�शया�और�साथ�लगती�महासागरीय��लेट�

कु छ�मह�वपूण��छोट���लेट��क��सूची�नीचे�द��गई�है�:

1. कोकोस��लेट�: म�य�अमे�रका�और��शांत��लेट�के �बीच�

2. नजका��लेट�: द��ण�अमे�रका�और��शांत��लेट�के �बीच�

3. अरे�बयन��लेट�: अ�धकाँशत: सऊद��अरब�क��भू�म�

4. �फलीप�स��लेट�: ए�शयाई�और��शांत��लेट�के �म�य�

5. कै रोलीन��लेट�: �फलीप�स�और�भारतीय��लेट�के �बीच�( �यू��गनी�के �उ�र�म�)�

6. �यूजी��लेट�: ऑ��े �लया�के �उ�र-पूव��म�।�

�लेट�सीमा�के �तीन��कार�ह�:�

अपसारी�सीमा�

जहाँ��लेट��के �एक��सरे�से��र�हटने�से�नई�पप�ट��का��नमा�ण�होता�है।�वह��थान�जहाँ�ये��लेट��एक��सरे�से��र�हटती�ह�
, को��सार��थान�कहा
जाता�है।�अपसारी�सीमा�का�सव��म�उदाहरण�म�य-अटलां�टक�कटक�है।�यहाँ�पर, अमे�रक���लेट�यूरे�शयन�व�अ��कन��लेट��से�अलग�होती
है।�

अ�भसारी�सीमा�

इसके �अंतग�त�एक��लेट�के ��सरी�के �नीचे�धंस�जाने�से�पप�ट��का��वंस�हो�जाता�है।�जहाँ�ऐसा�होता�है�उसे��ेपण��े ��


(स�ड�शन�जोन) कहा
जाता�है।�अ�भसारण�होने�के �तीन�तरीके �ह�।�ये�:ह�(i)
� महासागरीय�और�महा��पीय��लेट�के �बीच�; (ii) दो�महासागरीय��लेट��के �बीच�; एवं�(ii
i) दो�महा��पीय��लेट��के �बीच�

IASbaba
Score:
Web:
Email:
Page 75 150
https://telegram.me/UPSCMaterials https://telegram.me/FreeUPSCMaterials https://telegram.me/MaterialforExam

2019 - Test 2-Geography


Exam Title :
& C...
Email
Contact :

�पांत�रत�सीमा�

यहाँ�पप�ट��का�न�तो��नमा�ण�होता�है�और�न�ही��वंस��य��क��लेट���ै�तज��प�से�एक��सरे�से�होकर�गुजरती�ह�।��पांतरण��ंश��वलगाव�के
सम�े��ह��जो�म�यमहासागरीय�कटक�के �लंबवत�होते�ह�।�चूँ�क���फु , संटन पूण���शखर�के �साथ�एक�ही�समय�म��नह��होता
, पृ�वी�क��धुरी�से��र
�लेट�के �भाग�क��ग�त��भ��रहती�है।�साथ�ही�, पृ�वी�के �घूण�न�का��लेट�के �भाग��के �अलग��ए��लोक��पर�अपना��भाव�रहता�

है।� QUESTION 10.


�न�न�ल�खत�म��से�कौनसी�एक�छोट�( minor ) �लेट�है�?

a) नजका��लेट�
b) अंटाक� �टक�एवं�इसके �समीप�क��महासागरीय��लेट�
c) �फलीप�स��लेट�
d) a और�c दोन��
Correct Answer: D

Answer

Explanation
:
Solution (d)
1. कोकोस��लेट�: म�य�अमे�रका�और��शांत��लेट�के �बीच�
कु छ�मह�वपूण��छोट���लेट��क��सूची�नीचे�द��गई�है�
2. नजका��लेट�: द��ण�अमे�रका�और��शांत��लेट�के �बीच�

3. अरे�बयन��लेट�: अ�धकाँशत: सऊद��अरब�क��भू�म�

4. �फलीप�स��लेट�: ए�शयाई�और��शांत��लेट�के �म�य�

5. कै रोलीन��लेट�: �फलीप�स�और�भारतीय��लेट�के �बीच�( �यू��गनी�के �उ�र�म�)�

6. �यूजी��लेट�: ऑ��े �लया�के �उ�र-पूव��म�।�

IASbaba
Score:
Web:
Email:
Page 76 150
https://telegram.me/UPSCMaterials https://telegram.me/FreeUPSCMaterials https://telegram.me/MaterialforExam

2019 - Test 2-Geography


Exam Title :
& C...
Email
Contact :
QUESTION 11.
�न�न�ल�खत�म��से�कौन-से�कारक�महा��पीय��व�थापन��स�ांत�क���वफलता�का�कारण�बने�?

1. महा��पीय��व�थापन�हेतु�उ�रदायी��ुवीय��लीइंग�बल�एवं��वारीय�बल�पूण�त
: अपया��त��स���ए।�

2. महासागरीय�सतह�के �मान�च�ण�से��ा�त�सूचना�ने�महासागर��एवं�महा��प��के ��वतरण�के �संबंध�म��नए�आयाम��दान��कए।�

�न�न�ल�खत�म��से�सही�कू ट�का�चयन�क��जए�
:

a) के वल�1
b) के वल�2
c) 1 और�2 दोन��
d) न�तो�1 न�ही�2
Correct Answer: C

Answer

Explanation

Solution
बल�का�संबंध(c)
�पृ�वी�के �घूण�न�के �साथ�है।�

वे गेनर�ने�सुझाया��क�महा��पीय��व�थापन�हे
आपको�पता�होगा��क�पृ तु��ज�मेदार�ग�त�व�ध��ु
�वी�एक�पूण�तया�गोलाकार�नह�; �वषुवत�रेखवा�क
ीय��लीइं ग�बल�और��वारीय�बल�क
े �पास�इसम� ।� े �कारण�
�ई�थी।��ु
�इसम��एक�उभार�हैयह�उभार�पृ वीय�
�वी�के �घू ण�न�के �कारण
है�। �लीइं ग

वेगेनर��ारा�सुझाया�गया��सरा�बल�(�वारीय�बल) चं�मा�और�सूय��के �आकष�ण�के �कारण�है�जो�महासागरीय�जल�म���वार�ला�दे त।ा�है�


वेगेनर�का�मानना�था��क�बल�तभी��भावी�ह�गे�जब�वे�कई��म�लयन�वष��तक�लगते�रह�गे।�हालां�क, अ�धकाँश��व�ान��ने�इन�बल��को
पूण�तया�अपया��त�माना�।

यह�जानना��दलच�प�होगा��क�महा��पीय��व�थापन�हेतु�अ�धकाँश��माण�महा��पीय��े���से�पादप��और�जीव�जंतु��के ��न�े
(जैपस��े
�टलाईट) के ��वतरण�के ��प�म��एक���कए�गए�थे।���तीय��व��यु��के �दौरान�क��अव�ध�म��क��गई�अनेक�खोज��ने�भूवै�ा�नक�सा�ह�य�को
समृ���कया।��वशेषकर, महासागर�के �सतह�मान�च�ण�से��ा�त�सूचना�ने�महासागर��एवं�महा��प��के ��वतरण�के �संबंध�म��नए�
आयाम �दान��कए।�

QUESTION 12.
भूपप�ट��म���व�मान��न�न�ल�खत�त�व��को�उनके �संघटक�के �अनुसार�अवरोही��म�म���व��थत�क��जए�

1. ऑ�सीजन�

2. ए�युमी�नयम�

3. �स�लकॉन�

4. लौह�

5. कै ��शयम�

सही�कू ट�का�चुनाव�क��जए�

a) 3>2>1>5>4
b) 1>3>2>4>5
c) 1>2>4>3>5
d) 4>1>2>5>3

IASbaba
Score:
Web:
Email:
Page 77 150
https://telegram.me/UPSCMaterials https://telegram.me/FreeUPSCMaterials https://telegram.me/MaterialforExam

2019 - Test 2-Geography


Exam Title :
& C...
Email
Contact :
Correct Answer: B

Answer
Explanation

Solution (b)

पृ�वी�अनेक��कार�के �त�व��से�बनी�है।�ये�त�व�
पृ�वी�क��बाहरी�परत�म��ठोस�अव�था�म��और�आंत�रक�भाग�म��गम��व��पघली�अव�था�म�
ह��।

भूपप�ट��का�98 ��तशत�भाग�आठ�त�व��नामत: ऑ�सीजन, �स�लकॉन, ए�युमी�नयम, लौह, कै ��शयम, सो�डयम, पोटै �शयम�और
मै�नी�शयम�से�बना�है�तथा�शेष�म��टाइटे �नयम, हाइ�ोजन, फ़ा�फ़रोस, म�गनीज, स�फर, काब�न, �नकल�और�अ�य�त�व�ह�।�

भूपप�ट��के �त�व��ल�भ:तही�मु��अव�था�म���मलते�ह��और�अ�धकांशत: ये�अ�य�त�व��के �साथ�यौ�गक�के ��प�म���ा�त�होते�ह�इन�पदाथ�


।�
को�ख�नज�कहा�जाता�है।�

QUESTION 13.
�कसी��वशेष�ख�नज�क���न�न�ल�खत��वशेषताएं�ह�-

1. भूपप�ट��का�आधा�भाग�इस�ख�नज�से��न�म�त�है।�

2. इसका�रंग�ह�का���म�से�ह�का�गुलाबी�तक�होता�है।�

3. इसका�उपयोग�मु�यत: �सरे�मक�और�कांच�बनाने�म��होता�है।�

सही�ख�नज�का�चयन�क��जए�

a) फे �सपार�
b) �वॉट्� ज�
c) अ�क�
d) ऐ��फ़बोल�
Correct Answer: A

Answer

Explanation

Solution (a)

कु छ��मुख�ख�नज�और�उनक���वशेषताएं�
IASbaba
Score:
Web:
Email:
Page 78 150
https://telegram.me/UPSCMaterials https://telegram.me/FreeUPSCMaterials https://telegram.me/MaterialforExam

2019 - Test 2-Geography


Exam Title :
& C...
Email
Contact :

फे �सपार�

�स�लकॉन�और�ऑ�सीजन�सभी��कार�के �फे �सपार�म��एक�साझा�त�व�है�तथा�सो�डयम, पोटै �शयम, कै ��शयम, ए�युमी�नयम�आ�द


फे �सपार�क���व�श���क�म�म���मलते�ह�।�भूपप�ट��का�आधा�भाग�फे �सपार�का�बना�है।�इसका�रंग�ह�का���म�से�लेकर�सा�मन��प�क�जैसा
होता�है।�यह�मु�यत: �सरे�मक�और�कांच�बनाने�म���यु��होता�है।�

�वॉट्� स�

यह�बालू�व��ेनाइट�का�सबसे�मह�वपूण��अवयव��म��से�एक�है।�इसम���स�लका�होती�है।�यह�एक�कठोर�ख�नज�है�जो�जल�म��लगभग
अघुलनशील�है।�यह�सफ़े द�अथवा�रंगहीन�होता�है�और�
रे�डयो�व�राडार�म���यु��होता�है�।�यह��ेनाइट�के �सवा��धक�मह�वपूण��अवयव��म��से
एक�है।�

पाइरॉ�सीन�

पाइरॉ�सीन�म��कै ��शयम, ए�युमी�नयम, मै�नी�शयम, लौह�और��स�लका�होती�है।�पाइरॉ�सीन�भूपप�ट��के 10


� ��तशत�भाग�का��नमा�ण
करता�है।�यह�उ�का�प�ड��म��आमतौर�पर�पाया�जाता�है�।�इसका�रंग�हरा�अथवा�काला�होता�है।�

ए�फ�बोल�

ए�युमी�नयम, कै ��शयम, �स�लका, लौह, मै�नी�शयम�ए�फ�बोल�के ��मुख�त�व�ह�।�भूपप�ट��का�7 ��तशत�भाग�इनसे�बना�है�। इसका�रंग


हरा�अथवा�काला�होता�है�और�ए�बे�टस�उ�ोग�म��इनका��योग�होता�है।�हॉन��ल�डे�ए�फ�बोल�का�अ�य��कार�है।�

अ�क�

इसम��पोटै �शयम, ए�युमी�नयम, मै�नी�शयम, लौह, �स�लका�इ�या�द�होते�ह�।�भूपप�ट��का�4 ��तशत�भाग�इससे�बना�है।�इले���कल


उपकरण��म��इसका��योग�होता�है।�

QUESTION 14.
�न�न�ल�खत�म��से�कौन�आ�नेय�च�ान�का�उदाहरण�है�?

a) �ेनाइट�
b) पे�माटाइट�
c) घना�म�(ट�फ)
d) उपयु���सभी�
Correct Answer: D

Answer

Explanation

Solution (d)

आ�नेय�च�ान��पृ�वी�के �आंत�रक�भाग�से��नकलने�वाले�मै�मा�व�लावा�से��

न�म�त� होती�ह��और�इ�ह���ाथ�मक�च�ान��कहा�जाता�है।�आ�नेय
च�ान��( इ��नस�– ‘ फायर’ हेतु�लै�टन�श�द�) तब�बनती�ह��जब�मै�मा�ठं डा�होकर�ठोस�होता�है।�जब�मै�मा�ऊपर�आने�के ��म�म��ठं डा�होता�
है और�ठोस�म��बदलता�है, इसे�आ�नेय�च�ान�कहा�जाता�है।�

ठं डा�और�ठोस�होने�क�����या�भूपप�ट��अथवा�पृ�वी�क��सतह�पर�हो�सकती�है�।

आ�नेय�च�ान��का�वग�करण�उनक��बनावट�के �आधार�पर��कया�जाता�है।�बनावट, आकार�व�रव��क���मस�जा�अथवा�साम�ी�क��


अ�य भौ�तक�दशा��पर��नभ�र�करती�है।�य�द��पघला�पदाथ��अ�यंत�गहराई�म�-धीरे �धीरे�ठं डा�हो�तो�ख�नज�रवे�ब�त�बड़े�हो�सकते�ह�।�तेजी�से�ठं डा
होने�( सतह�पर�) रवे�छोटे �और��चकने�होते�ह�।�ठं डा�होने�म��इन�दोन��के �बीच�क��अव�था�से�आ�नेय�च�ान��के �रवे�भी�बीच�के �आकार�के �हो
जाते�ह�।�

�ेनाइट, ग��ो, पे�मेटाइट, बेसा�ट, वो�कै �नक��े�सया�और�टफ�आ�नेय�च�ान��के �कु छ�उदाहरण�


IASbaba
Score:
Web:
ह�।� QUESTION 15.
Email:
Page 79 150
https://telegram.me/UPSCMaterials https://telegram.me/FreeUPSCMaterials https://telegram.me/MaterialforExam

2019 - Test 2-Geography


Exam Title :
& C...
Email
Contact :

�न�न�ल�खत�म��से�कौन�सबसे�मुलायम�ख�नज�है�?

a) टै �क�
b) �ज�सम�
c) कोरंडम�
d) कै �साइट�
Correct Answer: A

Answer

Explanation

Solution (a)

कठोरता�से�आशय�है�खुरचे�जाने�पर�होने�वाला�सापे����तरोध।�

कठोरता�के ��तर�को�1-10 तक�मापने�के ��लए�दस�ख�नज��का�चुनाव��कया�जाता�है।�

वे�ह��: 1. टै �क�; 2. �ज�सम�; 3. कै �साइट�; 4. �लोराइट�; 5. एपेटाइट�; 6. फे �सपार�; 7. �वॉट्� स�; 8. टोपाज�; 9. कर�डम�; 10.

ही
रा।�

उदाहरण�के ��लए�तुलना�करने�पर, ऊँ गली�का�नाख़ून�2.5 एवं�चाकू �का��लेड�5.5

होता�है।�

QUESTION 16.
पृ�वी�पर�काय�शील��व�भ��बल��के �संदभ��म���न�न�ल�खत�कथन��पर��वचार�क��जए।�

1. अंतज��नत�बल�सामा�यत: �थल��नमा�ण�करने�वाले�बल�ह��और�ब�हज��नत�बल��थल�अप�रण�करने�वाले�बल�ह�।�

2. पटल�व�पण�एवं�अप�य�ब�हज��नत�भू-आकृ �तक����या��के �उदाहरण�

ह�।�सही�कू ट�का�चुनाव�क��जए�

a) के वल�1
b) के वल�2
c) 1 और�2 दोन��
d) उपरो��म��से�कोई�नह��
Correct Answer: A
म��अंतर��हेतु�उ�रदायी�है।�पृ�वी�क��सतह�पर��नरंतर�उजा��(सौर �काश) से��े�रत�ब�हज��नत�बल�काय��करते�रहते�ह�।�
Answer
पृ�वी�क��सतह�पर�पृ�वी�के �वायुमंडल�से�उ�प��ब�हज��नत�बल�तथा�पृ�वी�के �भीतर�उ�प��होने�वाले�आंत�रक�बल��नरंतर�काय��करते�रहते�ह�।�
Explanation
बा��बल��को�ब�हज��नत�बल�और�आंत�रक�बल��को�अंतज��नत�बल�कहा�जाता�है।�अंतज��नत�बल��के ��भाव�से�उ�चावच�का��रण� ( �न
�नीकरण�) और�पृ
Solution (a) � वी�क��सतह�पर�मौजू द �बे � सन/गत��का�भराव�होता�है । �

अपरदन�क े �मा�यम�से
भूपप�ट��ग�या�मक�है �पृ��वी�क
।�पृ वी�क��सतह�के �उ�चावच�क
े �भीतर�से े ��रण�म�
�काय��कर�रहे �वे�अंतज���अं तर�क��प�रघटना�
नत�बल��ज�ह�ने �े�भू
णपीकरण�(�े डेशणन)
प�ट��का��नमा� कहलाती�है
��कया�है �। पप�ट��क��बा��सतह
, का�अंतर�भू
�या�आप�जानते�ह��?

अंतज��नत�बल��नरंतर�पृ�वी�क��सतह�के �भाग��को�उठाते�व��न�म�त�करते�रहते�ह�और�इस��कार�ब�हज�
� �नत����याएं�पृ�वी�क��सतह�के
उ�चावच�के �अंतर��को�पाटने�म���वफल�रहती�ह�।�अत:, अंतर�तब�तक�बने�रहते�ह��जब�तक�ब�हज��नत�व�अंतज��नत�बल��क���वरोधी�काय�वाही

IASbaba
Score:
Web:
Email:
Page 80 150
https://telegram.me/UPSCMaterials https://telegram.me/FreeUPSCMaterials https://telegram.me/MaterialforExam

2019 - Test 2-Geography


Exam Title :
& C...
Email
Contact :
जारी�रहती�है।�सामा�य�अथ��म�, अंतज��नत�बल�भू�म�का��नमा�ण�करने�वाले�बल�ह��जब�क�ब�हज��नत�बल�मु�यत: भू�म��रण�करने
वाले�बल�ह��।

पटल�व�पण�और�वो�कै �न�म�भू-आकृ �तक����याएं�ह�।�

अप�य, वृहत��रण�, अपरदन�तथा��न�ेपण�ब�हज��नक�भू-आकृ �तक����याएं�ह

�।�

QUESTION 17.
�न�न�म��से�कौन�भौ�तक�अप�य�का�उदाहरण�है�?

a) जलयोजन�
b) अपप�ण�( Exfoliation )
c) काब�नेशन�
d) ऑ�सीकरण�व��यूनीकरण( reduction )
Correct Answer: B

Answer

Explanation (ए�लाइड) बल��पर��नभ�र�करती�ह�।�अनु�यु��बल��न�न�ल�खत�हो�सकते�ह��:

1) गु��व�बल�जै
Solution (b)से�अ�त-बोझ�दाब, बोझ�और�अप�पक���तबल, अपपण�न�(ए��फोलीएशन)।�

तापमान�प�रवत�
भौ�तक�अथवा�यां
2) न, ���टल�वृ���अथवा�जं
��क�अप�य����याएं तु�ग�त�व�ध�क
�कु छ�अनु �यु�� े �कारण��सार�बल�

3) नम�व�शु�क�च����ारा��नयं��त�होने�वाले�जल�दाब�

इनम��से�अनेक�बल�सतह�और�भू�साम��य��के �भीतर�लगते�ह���जनसे�च�ान��वभंजन�होता�है।�अ�धकतर�भौ�तक�अप�य����याएं�तापीय��सार
और�दाब��नमु�����ारा�घ�टत�होती�ह��

ये����याएं�छोट��एवं�धीमी�होती�ह���क�तु�च�ान�को�ब�त���त�प�ँचा�सकती�ह���य��क��नरंतर�थकान�के �कारण�च�ान�संकु चन�और��व�तारण�का


सामना�करती�है।�

�म�ण, काब�नेशन, हाइ�ेशन�और�ऑ�सीकरण�व�अवकरण�रासाय�नक�अप�य����या��के �उदाहरण�ह�।�

QUESTION 18.
जल�पात�एवं�����काएं�नद��क���कस�अव�था�के �साथ�संबं�धत�ह�?�

a) त�ण�अव�था�
b) �ौढ़�अव�था�
c) जीण��अव�था�
d) उपयु���सभी�
Correct Answer: A

Answer

Explanation

Solution (a) , �जसके �कारण�भू�म�क��सतह�का


�न�नीकरण�होता�है।�बहते�जल�के �दो�अवयव�ह�।�एक�है�एक�शीट�क��तरह�क��भू�म�सतह�पर�बहने�वाला�ओवरल�ड�बहाव।��सरा�है�रेखीय�बहाव
नम��े �, जहाँ��भारी�वषा�
जैसे�घा�टय��म� �होती�है।�, बहते�जल�को�सबसे�मह�वपूण��भूआकृ
धाराएं�व�न�दयां �तक�कारक�समझा�जाता�

है

IASbaba
Score:
Web:
Email:
Page 81 150
https://telegram.me/UPSCMaterials https://telegram.me/FreeUPSCMaterials https://telegram.me/MaterialforExam

2019 - Test 2-Geography


Exam Title :
& C...
Email
Contact :

बहते�जल�के ��भाव�म��भू��य��वकास�के ���येक�चरण�क���वशेषताएं��न�न�ल�खत�ह�


-

त�ण�

इस�अव�था�म��धाराएं�कम�होती�ह��तथा�मूल�ढलान�पर�उनका�एक�करण�और�बहाव�कमजोर�होता�है।�इसके �प�रणाम�व�प�
V- आकार�क�
घा�टयाँ�बनती�ह���जनका��ं क�धारा��के ��नकट�बनने�वाला�बाढ़�का�मैदान�या�तो�नह��होता�या�अ�यंत�संकरा�होता�है।�धारा��वभाजन�वृहत�और
सपाट�होते�ह���जनम��माश�, दलदल�और�झील��होती�ह�।��वसप��इन��व�तृत�ऊपरी�भू :
अपने�चार��ओर�खाई�का��नमा�ण�कर�लेते�ह�।�जहाँ�कठोर�च�ान��अनावृत�हो�जाती�ह�, वहाँ�जल�पात��और�����का��का��नमा�ण�हो
सकता�है�। -सतह��पर��वक�सत�होते�ह�।�ये��वसप��ऊँ ची�भू�म�म��सामा�यत

�ौढ़�

इस�चरण�के �दौरान�धारा��म��ब�तायत�म��एक�करण�होता�है।�घा�टयाँ�अभी�भी�V- आकार�क���क�तु�गहरी�होती�ह�; �ं क�����स�इतनी��व�तृत


होती�ह���क�उनका�बाढ़�का��े��भी�इतना�बड़ा�होता�है��जसम��घाट��के �भीतर�रहकर�ही�धाराएं��वसप���कार�से�बह�सक� ।�सपाट�व��व�तृत
आंत�रक�धारा�का��े��और�युवाव�था�म���न�म�त�दलदली��े��व�माश��गायब�हो�जाते�ह��तथा�धारा��वभाजक�ती��हो�जाते�ह�।�जल�पात�व
����काएं�भी�गायब�हो�जाती�ह��।

जीण��

जीण��आयु�के �दौरान�छोट��सहायक�न�दयाँ�कोमल�ढाल�के �साथ�सं�या�म��कम�होती�ह�।�धाराएं��वशाल�बाढ़�मैदान�पर��वसप��आकार�म��मु���प


से�बहती�ह��और��ाकृ �तक�तटबंध�व�गोखुर�झील��का��नमा�ण�करती�ह�।��वभाजक�झील�
, दलदली��े���व�माश��के �साथ��व�तृत�व�सपाट�होते�ह�।
अ�धकांश�भू��य�समु���तर�पर�अथवा�उससे�थोड़ा�ऊपर�होता�है।�

QUESTION 19.
�न�न�ल�खत�म��से�कौन-सी��हमनद�के ��न�ेपण�से�बनी�भू-आकृ �त�है�?

a) डे�टा�
b) �हमोढ़�
c) सक� �
d) ती�ण�कटक( Arete )
Correct Answer: B

Answer

Explanation

Solution (b) (महा��पीय��हमनद�अथवा��पडमांट��हमनद, य�द�बफ� �क��एक�बड़ी�शीट�समतल


मैदान�अथवा��ग�रपाद��े��म��फै ली�है) अथवा��व�तृत�कुं डनुमा�घा�टय��
(पव�त�और�घाट���हमनद) म��पव�त��क��ढलान��पर�रेखीय��प�से�बहने
बफ � �क��बड़ी�मा�ा�का�भू।��म�पर�शीट�के ��प�म��ग�त�करना�
को��हमनद�कहा�जाता�है

अपनी�घाट��म��एक��हमनद�जल��वाह�क��अपे�ा�धीमा�होता�है।�यह�ग�त���त�दन�कु छ�स�ट�मीटर�से�लेकर�कु छ�मीटर�या�इससे�कु छ�कम


-
�यादा�हो�सकती�है।��हमनद�मु�यत: गु��वाकष�ण�के �कार�
ण�ग�त�करते�ह�।�

बफ� �के �कु ल�भार�के �कारण�होने�वाले�घष�ण


के��चलते��हमनद���ारा��कया�जाने�वाला�अपरदन�जबरद�त�होता�है।��हमनद���ारा�जमीन�से
उठाई�गई�साम�ी�( सामा�यत: बड़े�आकार�के �नुक�ले��लॉ�स�व�टु कड़े)� सतह��अथवा�घा�टय��के ��कनार��के �साथ-साथ��घसटती�जाती�है�और
अपकष�ण�तथा�उ�पाटन�के �मा�यम�से�काफ����त�करती�है।�

�हमनद�यहाँ�तक��क��बना�अप�य��ई�च�ान��को�भी�भारी���त�प�ँचा�सकते�ह��और�उ�ह��ऊँ चे�पव�त��को�नीची�पहा�ड़य��व�समतल�मैदान�म�
बदल�सकते�ह�।�जब��हमनद�ग�त�करते�ह�, मलबा�हट�जाता�है, �वभाजक�नीचे�हो�जाते�ह��और�ढलान�इतनी�नीची�हो�जाती�ह���क��हमनद�ग�त
करना�बंद�कर�दे ते�ह��और�अपने�पीछे �अ�य��न�े�पत��वशेषता��के �साथ
-साथ�नीची�पहा�ड़यां �व��व�तृत��हमानीधौत-मैदान�छोड़�जाते�ह�।�

डे�टा�एक��न�े�पत�भूआकृ �त�है��जसका��नमा�ण�बहते�जल��ारा�होता�है
, �हमनद��ारा�नह��।

�हमनद���ारा��न�म�त�अपर�दत�भूआकृ �तयां�

IASbaba
Score:
Web:
Email:
Page 82 150
https://telegram.me/UPSCMaterials https://telegram.me/FreeUPSCMaterials https://telegram.me/MaterialforExam

2019 - Test 2-Geography


Exam Title :
& C...
Email
Contact :

1) सक� �

2) �ग�र�ृंग�व��सरेटेड�कटक�

3) �हमनद�घा�टयाँ/गत��

�हमनद���ारा��न�म�त��न�े�पत�भूआकृ �तयां�

1) �हमोढ़�

2) ए�कर�

3) �हमानीधौत-मैदान�

4) �म�लन�

QUESTION 20.
�न�न�ल�खत�म��से�कौन�रासाय�नक�अप�य����या�को�ती��करने�हेतु�एक�आदश��अव�था�क��भां�त�काय��करता�है�?

a) उ�ण�एवं�आ�� �
b) उ�ण�एवं�शु�क�
c) शीत�एवं�आ�� �
d) उपरो��म��से�कोई�नह��
Correct Answer: A

Answer

Explanation

Solution (a) भूवै�ा�नक, जलवाय�वक, �थलाकृ �तक�एवं�वान�प�तक�कारक���ारा��न�म�त�होती�ह�।


जलवायु�का��वशेष�मह�व�है।�
अप�य����या��हेतु�प�र��थ�तयां�अनेक�ज�टल�
न�के वल�अप�य����याएं�एक�जलवायु�से��सरी�जलवायु�म���भ��होती�ह�, अ�पतु�अप�य�आ�छादन�क��गहराई�भी��भ��होती�है।�

रासाय�नक�अप�य�

अप�य����या��का�समूह�A अथा�त��म�ण, काब�नेशन, जलयोजन�, ऑ�सीकरण�व�अवकरण�च�ान�को�ऑ�सीजन, सतही�और/अथवा


मृदा�जल�व�अ�य�अ�ल��क��सहायता�से�रासाय�नक�अ�भ��या��ारा�अपघ�टत�करने, उसे�घोलने�अथवा�घटाकर�छोटे �टु कड़े�हेतु�काय��करता�है।�

रासाय�नक�अ�भ��या��म��तेजी�लाने�के ��लए�उ�मा�के �साथ�जल�व�वायु�( ऑ�सीजन�और�काब�न�डाइऑ�साइड�) का�उप��थत�होना


आव�यक�है।�इससे�भी��यादा, वायु�म��उप��थत�काब�न�डाइऑ�साइड, पादप��व�जंतु��का�अपघटन�भू�मगत�काब�न�डाइऑ�साइड�
क�मा�ा�को�बढ़ा�दे ते�ह��।

�व�भ��ख�नज��पर�होने�वाली�रासाय�नक�अ�भ��याएं��कसी��योगशाला�म��होने�वाली�रासाय�नक�अ�भ��या��के �स�श�ही�ह�।�

QUESTION 21.
�न�न�ल�खत�म��से�कौन�प�ण�( foliation ) ���या�क��सव��म��ा�या�करता�है�?

a) यह�काया�त�रत�च�ान��म��ख�नज��अथवा�कण��का�परतीय��म�(जमाव) है�जो��ववत��नक�अप�पण�के �साथ�उ�च�तापमान�अथवा�दबाव


अथवा�दोन��के ��ारा��कए�गए��व�पण�के �कारण�होता�है।�
b) यह�आ�नेय�च�ान��म��ख�नज��अथवा�कण��का�परतीय��म�(जमाव) है�जो��ववत��नक�अप�पण�के �साथ�उ�च�तापमान�अथवा�
दबाव अथवा�दोन��के ��ारा��कए�गए��व�पण�के �कारण�होता�है।�

IASbaba
Score:
Web:
Email:
Page 83 150
https://telegram.me/UPSCMaterials https://telegram.me/FreeUPSCMaterials https://telegram.me/MaterialforExam

2019 - Test 2-Geography


Exam Title :
& C...
Email
Contact :

c) यह�अवसाद��च�ान��म��ख�नज��अथवा�कण��का�परतीय��म�(जमाव) है�जो��ववत��नक�अप�पण�के �साथ�उ�च�तापमान�अथवा�दबाव


अथवा�दोन��के ��ारा��कए�गए��व�पण�के �कारण�होता�है।�
d) यह�एक����या�है��जसम���व�भ��वग��के �ख�नज�अथवा�पदाथ��आ�नेय�च�ान��म��गोलाकार����टल��के ��प�म���व��थत�होती�ह�।�
Correct Answer: A
Answer
Explanation

Solution (a)

टू टने�व�चूर-चूर�होने�के �कारण�तथा�पया��त�रासाय�नक�प�रवत�न��के ��बना�च�ान��के �भीतर�के �मूल�ख�नज��का�यां��क��वघटन�व�


पुनस�गठन ग�तक�कायांतरण�कहलाता�है।�

तापीय�कायांतरण�के �कारण�च�ान��क��साम��यां�रासाय�नक��प�से�प�रव�त�त�और�पुन
होते�ह��— संपक� �कायांतरण�तथा��े�ीय�कायांतरण�। संपक� �कायांतरण�म��च�ान��गम� �भेदक�मैृ �त�होती�ह�
:���टलीक मा�और�लावा�क े �सं
।�दो��कार�क प� क� �म��आती�ह��और�च�ान
े तापीय�
के �ख�नज�उ�च�तापमान��के �अंतग�त�पुन
:���टलीकृ त�होते�ह�।� कायांतरण�

कई�बार�मै�मा�और�लावा�से�बनी�साम��यां�भी�च�ान�से�जुड़�जाती�ह�।��े�ीय�कायांतरण�म��च�ान��उ�च�तापमान�अथवा�दाब�अथवा�दोन��के
साथ�होने�वाले��ववत��नक�अप�पण�के �कारण� पुन:���टलीकृ त�होती�ह�।�कु छ�च�ान��म��कायांतरण�क�����या�के �दौरान�रवे�अथवा�ख�नज
परत��अथवा�पं��य��म���व��थत�हो�जाते�ह��। कायांत�रत�च�ान��म��ख�नज��अथवा�रव��क��ऐसी��व�था�पण�न�अथवा��ल�नएशन
कहलाती�है�।�कई�बार��व�भ��वग��के �ख�नज�अथवा�साम��यां�पतली�अथवा�मोट��वैक��पक�परत��म���व��थत�हो�जाती�ह��और�हलके �व�गहरे
रंग�क���तीत�होती�ह�।�कायांत�रत�च�ान��म��ऐसी�संरचना�ब��ड�ग�कहलाती�है�तथा�ब��ड�ग��द�श�त�करने�वाली�च�ान��बं�धत�च�ान��कहला
ती�ह�।�

QUESTION 22.
महासागरीय�कटक�क��मौजूदगी�भारतीय�और�अंटाक� �टक��लेट�सीमा�क���वशेषता�है।�ऐसा�कौनसी��लेट�सीमा�के �कारण�संभव�होता�है
? �

a) महा��प�- महा��प�अ�भसरण�
b) महा��प�- महासागरीय�अ�भसरण�
c) महा��प�- महासागरीय�अपसरण�
d) महासागरीय�- महासागरीय�अपसरण�
Correct Answer: D
AnswerExplanation

Solution (d)

भारतीय��लेट�के �अंतग�त��ाय��पीय�भारत�और�ऑ��े �लयाई�महा��पीय�भाग�आते�ह�।��हमालय�के �साथ�लगता��न�न�खलन��े��


(सबड�शन
ज़ोन) महा��प�(महा��प�अ�भसरण) के ��प�म��उ�री��लेट�क��सीमा�बनाता�है।�पूव��म��यह
ऑ��े �लया�के �पूव��म����थत�है।�प��मी��कनारा,ा प �क�तान�के ��करथर�पव�त�का�अनुसरण�करता�है ।�यह�आगे
, जावा�गत� �के ��साथ�वाली���पीय�चाप�क��ओर
मकरान�तट�के �साथ -साथ
�व�ता�रत�है �और�चागोस���पसमू
�यांमार�के �रा�क�योमा�पव� ह�के �साथ�द��ण
त�के �मा�यम�से -पूव��म�।��पू
��व�ता�रत�है ��थत�लाल�सागर��ं श�वाले��सा�रत��थान�से
व���कनारा�एक��सा�रत��थान�है �जो�द �जाकर�जुड़ता�है।�
.प.�शांत�म��एक�महासागरीय�कटक�के ��प�म
महासागरीय�कटक�(अपसरण�सीमा) भारतीय�और�अंटाक� �टक��लेट�के �बीच�क��सीमा�क���वशे � षता�है�जो�मोटे �तौर�पर�प��म�से�पूव��क�
ओर�फै ली�है�और��यूज़ीलै�ड�के �थोड़ा�द��ण�म���सा�रत��थान�से��मल�जाती�है।�

QUESTION 23.
भूकं पीय�तरंग��के �संदभ��म���न�न�ल�खत�कथन��पर��वचार�क��जए।�

1. भूगभ�य�तरंग�, उ�म�क� ��से��नमु����ई�उजा��के �कारण�उ�प��होती�ह��और�सबसे��वनाशकारी�मानी�जाती�ह�।�

2. P तरंग���व�न�तरंगो�के �स�श�होती�ह���य��क�ये�गैस,ीयठोस�एवं�तरल�मा�यम��म��गमन�कर�सकती�ह�।�

3. S- तरंग��का�छाया��े�, P- तरंग��के �छाया��े��से�अ�धक�द�घ��होता�है।�

IASbaba
Score:
Web:
Email:
Page 84 150
https://telegram.me/UPSCMaterials https://telegram.me/FreeUPSCMaterials https://telegram.me/MaterialforExam

2019 - Test 2-Geography


Exam Title :
& C...
Email
Contact :

सही�कू ट�का�चुनाव�क��जए�

a) के वल�1 और�2
b) के वल�2 और�3
c) के वल�1 और�3
d) उपयु���सभी�
Correct Answer: B
AnswerExplanation

Solution (b)

भूकं पीय�तरंगे�मूलत
: दो��कार�क��होती�ह��— भूगभ�य�तरंग��तथा�धरातलीय�तरंग�।�भूगभ�य�तरंग�, भूकं प�मूल�पर��नमु���उजा��के
कारण उ�प��होती�ह��और�पृ�वी�के �अंदर�सभी��दशा��म��गमन�करती�ह�: इ�ह�
।�अत�भूगभ�य�तरंग��नाम��दया�गया�है।�

भूगभ�य�तरंग,� धरातलीय�च�ान��के �साथ�अंत:��या�करती�ह��और�धरातलीय�तरंग�नामक�तरंग��के �नए�समु�चय�को�ज�म�दे ती�ह�।�ये�


तरंगे धरातल�के �साथ�चलती�ह�।�इन�तरंग��का�वेग��भ��घन�व��वाली�साम��य��म��से�गमन�के �समय�बदल�जाता�है
साम�ी��जतनी�सघन�होगी,
।� वेग
उतना�अ�धक�होगा�।��भ��घन�व��वाली�साम��य��म��से�गमन�के �समय�अपवत�न�अथवा�परावत�न�के �कारण�इनक���दशा�भी�बदलती�है।�

भूगभ�य�तरंग��के �दो��कार�ह�।�इ�ह�P� तथा�S- तरंगे�कहा�जाता�है।�P- तरंगे�तेजी�से�गमन�करती�ह��और�धरातल�पर�सबसे�पहले�


प�ँचती ह��। इ�ह��‘�ाथ�मक�तरंग’े भी�कहा�जाता�है�। P- तरंगे��व�न�तरंग��के �समान�होती�ह�।,�येठोस�एवं
�ये�गैसीय
�तरल�मा�यम��म��गमन�करती�ह�।�
S- तरंगे�कु छ�समय�प�ात�धरातल�पर�प�ँचती�ह�।�इ�ह����तीयक�तरंगे�कहा�जाता�हैS- ।� तरंग��के �बारे�म��एक�मह�वपूण��त�य�यह�है��क�ये
के वल�ठोस�मा�यम�म��ही�गमन�कर�सकती�ह�।�S - तरंग��क��यह��वशेषता�काफ��मह�वपूण��है।�इसने�वै�ा�नक��को�पृ�वी�के �आंत�रक�भाग�
क�संरचना�समझने�म��मदद�क��है।�

�या�आप�जानते�ह��?

सभी��ाकृ �तक�भूकं प��थलमंडल�म��आते�ह�।�


‘�स�मो�ाफ’ नामक�उपकरण�धरातल�पर�प�ँचने�वाली�तरंग��को�दज��करता�है।�

छाया��े��का�उ�व�
जाता�है।��व�भ��घटना��का�अ�ययन�दशा�ता�है��क���येक�भूकं प�हेतु�एक��भ��छाया��े��होता�है।�
भूकं प�तरंगे��र���थत��स�मो�ाफ�पर�दज��होती�ह�।�हालां�क
यह�दे खा�गया��क�अ�धके ���से105°
� के �भीतर�रखे��स�मो�ाफ�ने �P और�S-
, कु छ��व�श���े ��ह�तरं ग��दोन��को�दज�
�जहाँ �जहाँ�ये�तरंग�े��कया।�हालां �क�, अ�धक
दज��नह��होती।�ऐसे े ���से
��े��को� �145°�’सेकहा
‘छाया��े �परे
रखे��स�मो�ाफ�ने�P तरंग��को�दज���कया, तथा�S- तरंग��को�दज��नह���कया।�इस��कार�, अ�धके ���से�105° से�145° तक�का��े��दोन�
तरंग��के ��लए�छाया��े��के �तौर�पर�पहचाना�गया�है105°
।� से�परे�का�पुरा��े��S- तरंग��को�दज��नह��करता।�S- तरंग��का�छाया��े��P- तरं
ग��से�कह��बड़ा�है।�

P- तरंग��का�छाया��े��पृ�वी�के �चार��ओर�एक�ब�ड�क��तरह��दखता�है�जो�अ�धके ���से


105° � से�145° के �बीच�होता�है।�

S- तरंग��का�छाया��े��न�के वल�बड़ा�होता�है
, अ�पतु�यह�पृ�वी�क��सतह�के 40
� ��तशत�से�कु छ�अ�धक�होता�है।�

QUESTION 24.
काया�तरण�क�����या�होने�पर�चूना�प�थर��न�न�ल�खत�म��से��कस�च�ान�म��प�रव�त�त�हो�जाता�है�?

a) संगमरमर�
b) नाइस�
c) �श�ट�
d) �वाट� जाइट�
Correct Answer: A
AnswerExplanation

IASbaba
Score:
Web:
Email:
Page 85 150
https://telegram.me/UPSCMaterials https://telegram.me/FreeUPSCMaterials https://telegram.me/MaterialforExam

2019 - Test 2-Geography


Exam Title :
& C...
Email
Contact :

Solution (a)

च�ान��लंबे�समय�तक�अपने�मूल��व�प�म��नह��रहती�तथा�उनका�काया�तरण�हो�जाता�है।�च�ान�च��एक��नरंतर�चलने�वाली����या�है��जसके
मा�यम�से�च�ान��नए�आकार�म��कायांत�रत�हो�जाती�ह�।�आ�नेय�च�ान���ाथ�मक�च�ान��ह��और�अ�य�च�ान��( अवसाद��अथवा��पांत�रत�) इन
�ाथ�मक�च�ान��से��न�म�त�होती�ह�।�

QUESTION 25.
�वालामुखी�के �संदभ��म���न�न�ल�खत�कथन��पर��वचार�क��जए
-

1. ये��वालामुखी�अ�य�धक�तरल�लावा��वा�हत�करते�ह��जो�लंबी��री�तक�बह�कर��व�तृत��े��म��फै लता�है।�

2. भारत�का�द�कन��ै प��जसके �अंतग�त�महारा��पठार�का�अ�धकतर�भाग�आता�है, इस��वालामुखी�का�उदाहरण�है।�

यहाँ�कौनसे��वालामुखी�को�संद�भ�त��कया�गया�है�?

a) शी�ड��वालामुखी�
b) �म��त��वालामुखी�
c) शंकु ��वालामुखी�
d) का�डेरा�
Correct Answer: A
AnswerExplanation

Solution (a)

IASbaba
Score:
Web:
Email:
Page 86 150
https://telegram.me/UPSCMaterials https://telegram.me/FreeUPSCMaterials https://telegram.me/MaterialforExam

2019 - Test 2-Geography


Exam Title :
& C...
Email
Contact :

�लड�बेसा�ट��ो�व�स�अथवा�शी�ड��वालामुखी�– ये��वालामुखी�अ�य�धक�तरल�लावा�पैदा�करते�ह��जो�लंबी���रय��तक�बहता�है�।
�व��के �कु छ�भाग�बेसा�ट�लावा�बहाव�के �हजार���कमी�क��मोट��परत��ारा�आवृत�ह�।�बहाव�क��एक��ृंखला�पाई�जा�सकती�है��जसम��कु छ
बहाव��क��मोटाई�50 मीटर�से�भी�अ�धक�हो�सकती�है।�

���गत�बहाव�हजार���कमी�तक��व�ता�रत�हो�सकते�ह�।�भारत�का�द�कन��ै प��जसके �अंतग�त�महारा��पठार�का�अ�धकतर�भाग�आता


है, एक�ब�त��वशाल��लड�बेसा�ट��ो�व�सस�है�।

यह�माना�जाता�है��क�आरंभ�म��इस��ै प�के �अंतग�त�वत�मान�से�कह��अ�धक��े��आता�था।�

�या�आप�जानते�ह��?

�म��त��वालामुखी�

इन��वालामु�खय��म��बेसा�ट�क��अपे�ा�ठं डे�और�अ�धक��चप�चपे�लावा�उदगार�होते�ह��।�इन��वालामु�खय��म���ाय: �व�फोटक�उदगार�दे


खे�जाते�ह�।�लावा�के �साथ
, �वलख�डा�शम�पदाथ��और�राख�क��बड़ी�मा�ा�भी�धरातल�पर�आ�जाती�है।�यह�साम�ी��नकासमुख�के �आस -पास
जमा�हो�जाती�है��जससे�परत��का��नमा�ण�होता�है�तथा�इससे�पव�त��म��त��वालामु�खय��क��भां�त��दखाई�दे ते�है।�

QUESTION 26.
�न�न�ल�खत�म��से�कौनसा��ब�लतामंडल�क��सव��म��ा�या�करता�है�?

a) यह�म�टल�के �ऊपरी�भाग�का��ह�सा�है।�
b) यह�म�टल�से�परे�का��े��है।�
c) यह�पृ�वी�का�वह�भाग�है��जसम��पप�ट��और�ठोस�म�टल�का�ऊपरी�भाग�शा�मल�है।�
d) यह�पप�ट��से�परे�आंत�रक�भाग�का��ह�सा�है।�
Correct Answer: A
Answer
Explanation

Solution (a)

पृ�वी�क��संरचना�

पप�ट��

यह�पृ�वी�का�सबसे�बाहरी�ठोस�भाग�है�।�इसक���कृ �त�भुरभुरी�होती�है।�पप�ट��क��मोटाई�महासागरीय�और�महा��पीय�भाग��म���भ��
होती है।�महासागरीय�पप�ट��महा��पीय�पप�ट��क��अपे�ा�पतली�होती�है �
।�यह�भारी�च�ान��से ��न�म�त�होती�है��जनका�घन�व�3 g/cm3
होता है।�बेसा�ट, महासागरीय�पप�ट��म��पाई�जाने�वाली�च�ान�का��कार�है।महासागरीय�पप�ट��क��साम�ी�का�औसत�घन�व� 2.7 g/cm3 है।�

मै�टल�

पप�ट��से�आगे�का�भाग�मै�टल�कहलाता�है�। मै�टल�मोहो�असंब�ता�से�लेकर�
भाग��ब�लतामंडल�कहलाता�है।�ए�थेनो�श�द�का�अथ��है- कमजोर।�इसका��व�तार�200
2,900 �कमी�से
�कमी�. �क��गहराई�तक��व�तृ
700 �कमी. तक�माना�जाता�है �( नोट:
त�है।�मै�टल�का�ऊपरी
अलग-अलग���ोत��ब�लतामंडल�क��गहराई�अलग-अलग�बताते�ह�।�हालां�क, यही�सवा��धक��वीकाय��है�) । यह�मै�मा�का�मु�य���ोत�है�जो
�वालामुखीय�उदगार��के �समय�पृ�वी�क��सतह�पर�प�ँचता�है।इसका�घन�व�पप�
� ट��के �घन�व�(3.4 g/cm3) से�अ�धक�है।�

पप�ट��और�मै�टल�का�सबसे�ऊपरी�भाग��थलमंडल�कहलाता�है।�इसक��मोटाई�10-200 �कमी�. तक�है।�मै�टल�का��नचला�भाग��ब�लतामंडल


से�भी�आगे�तक��व�तृत�है।�यह�ठोस�अव�था�म��है।�

IASbaba
Score:
Web:
Email:
Page 87 150
https://telegram.me/UPSCMaterials https://telegram.me/FreeUPSCMaterials https://telegram.me/MaterialforExam

2019 - Test 2-Geography


Exam Title :
& C...
Email
Contact :

�ोड़�

�ोड़�- मै�टल�सीमा�2,900 �कमी�. क��गहराई�पर���थत�है।�बाहरी��ोड़�तरल�अव�था�म��है�जब�क�आंत�रक��ोड़�ठोस�अव�था�म�।��ो


ड़�- मै�टल�सीमा�पर�साम�ी�का�घन�व�लगभग�5 g/cm3 है�और�पृ�वी�के �क� ��म� �
6,300 km क��गहराई�पर�घन�व�का�यह�मान�लगभग�1
3g/cm3 है।�

�ोड़�भारी�धातु��का�बना�है��जसम��अ�धकांशत: �नकल�और�लौह�है।�इसे�सामा�यत: �नफे �परत�कहा�जाता�है।�

QUESTION 27.
संभा�वत�संसाधन��के �संदभ��म���न�न�ल�खत�म��से�कौन�सा
/से�कथन�स�य�है/ह��?

1. �व��भर�म��इनका��ापक��योग�हो�रहा�है।�

2. वे�अ�यंत�अ�प�मा�ा�म���व�मान�ह�।�

3. उ�ह��वत�मान�म���योग�नह���कया�जा�रहा��क�तु��ौ�ो�गक��का��वकास�होने�पर�भ�व�य�म���योग��कया�जा�सकता�है।�

�न�न�म��से�सही�कू ट�का�चयन�क��जए�
:

a) 1 और�2
b) 2 और�3
c) के वल�3
d) उपयु���सभी�
Correct Answer: C
AnswerExplanation

Solution (c)

संभा�वत�संसाधन�वे�ह���जनक��संपूण��मा�ा�क��जानकारी�नह��और�वत�मान�म��उनका��योग�नह���कया�जा�रहा।�इन�संसाधन��को�भ�व�य�म
��योग��कया�जा�सकता�है।�

�ौ�ो�गक��का�जो�वत�मान��तर�हमारे�पास�है�वह�इतना��वक�सत�नह���जसके �मा�यम�से�इन�संसाधन��को�आसानी�से��योग��कया�जा�सके । ल�ाख�


म��पाया�जाने�वाला�यूरे�नयम�संभा�वत�संसाधन�का�उदाहरण�है��जसे�भ�व�य�म���योग��कया�जा�सकता�है।�उ�च�ग�त�क��पवन��दो�सौ�वष�

IASbaba
Score:
Web:
Email:
Page 88 150
https://telegram.me/UPSCMaterials https://telegram.me/FreeUPSCMaterials https://telegram.me/MaterialforExam

2019 - Test 2-Geography


Exam Title :
& C...
Email
Contact :
पूव��एक�संभा�वत�संसाधन�थी।�वत�मान�म��यह�एक�वा�त�वक�संसाधन�ह��और��व�ड�फाम��पवनच��कय��का��योग�कर�उजा��उ�प��करते�ह
� , जैसे
�क�नीदरल�ड्स�म�।�कु छ�को�हम�त�मलनाडु �के �नागरकोइल�और�गुजरात�तट�पर�दे ख�सकते�ह�।�

QUESTION 28.
म��चंग( Mulching ) मृदा�संर�ण�क��एक��व�ध�है।��न�न�म��से�कौन
-सा�कथन�म��चंग�क�����या�को�सही��कार�से�प�रभा�षत�करता�है�

a) एका�तर�पं��य��म���व�भ��फसल��उगाई�जाती�ह��और�उ�ह��अलग
-अलग�समय�पर�बोया�जाता�है�ता�क�मृदा�को�वषा��जल�म��बहने�से
बचाया�जा�सके ।�
b) ढलान�से�वषा��जल�को�बह�जाने�से�रोकने�हेतु��ाकृ �तक�बाधा�बनाने�के ��लए�पव�त�क��ढलान�के �समानांतर�जुताई।�
c) मृदा�आवरण�के �संर�ण�हेतु�पवन��क��ग�त�को��नयं��त�करने�के ��लए�वृ���क��पं��याँ�लगाई�जाती�ह�।�
d) वृ���के �म�य�के ��र���े��को�सूखी�घास�जैसे�जै�वक�पदाथ��क��परत�से�ढका�जाता�है।�
Correct Answer: D
Answer
Explanation

Solution (d)
�योग, वषा�-कटाव, भू�खलन�तथा�बाढ़�आ�द�शा�मल�है।�
मृदा�अपरदन�और�अव�य�एक�संसाधन�के ��प�म��मृदा�पर�मंडरा�रहे�सबसे�बड़े�खतरे�ह�।�मानवीय�और��ाकृ �तक�दोन��ही�कारक�मृदा�का
मृ दा�संर�ण�के �कु छ�तरीक
�न�नीकरण�कर�सकते �ह�।�मृदे ा��न�नीकरण�करने
�नीचे��दए�गए�ह�
: � �वाले�कारक��म���नव�नीकरण
, अ�तचराई, रासाय�नक�उव�रक��व�क�टनाशक��का�अ�य�धक
म��चंग�: पौध��के �बीच�के �खाली��े��को�सूखी�घास�जैसे�जै�वक�पदाथ��क��परत�से�ढका�जाता�है।�यह�वषा��जल�को�बचाकर�रखने�म��सहायता
करता�है।�

समो�च�बाधाएं�: प�थर, घास, �म���को�समो�च�रेखा��के ��नकट�बाधाएं��न�म�त�करने�हेतु��योग��कया�जाता�है।�इन�बाधा��के ��नकट�जल


सं�हण�हेतु�गत��बनाए�जाते�ह�।�

रॉक�डैम�: जल�का�बहाव�घटाने�के ��लए�प�थर��का�ढे र�लगा��दया�जाता�है।�यह�अवना�लका�को�और�होने�वाली�मृदा���त�को�रोकता�है।�

टे रेस�फा�म�ग�: खड़ी�ढलान��पर��व�तृत�समतल�कदम��अथवा�टे रेस�का��नमा�ण��कया�जाता�है�ता�क�फसल��उगाने�के ��लए�समतल�


सतह�उपल�ध�रह�।�ये�सतह�से�बह�जाने�वाले�जल�और�मृदा�अपरदन�को�घटाते�ह�।�

अंतफ� सल� : वैक��पक�कतार��म��अलग-अलग�फसल��उगाई�जाती�ह��और�इ�ह��अलग-अलग�समय�पर�बोया�जाता�है�ता�क�मृदा�को�वषा��जल�


म�बहने�से�बचाया�जा�सके ।�

समो�च�जुताई�: ढलान�से�वषा��जल�को�बह�जाने�से�रोकने�के ��लए�एक��ाकृ �तक�बाधा��न�म�त�करने�हेतु�पहाड़ी�क��ढलान�के �समानांतर


जुताई।�

शे�टर�बे�ट् स�: तट�य�व�शु�क��े���म�, मृदा�आवरण�को�पवन��क��ग�तय��से�बचाने�के ��लए�वृ���क��कतार��लगा��जाती�ह�।�

QUESTION 29.
भू- सतह�के ��नकट���थत�ख�नज��को��ाय: खोद�कर��नकाल��लया�जाता�है।�ख�नज��नकालने�क��यह����या�कहलाती�है-

a) शा�ट�खनन�
b) ���ल�ग�
c) उ�खनन�
d) �ै �क�ग�
Correct Answer: C
AnswerExplanation

IASbaba
Score:
Web:
Email:
Page 89 150
https://telegram.me/UPSCMaterials https://telegram.me/FreeUPSCMaterials https://telegram.me/MaterialforExam

2019 - Test 2-Geography


Exam Title :
& C...
Email
Contact :

Solution (c)

पृ�वी�क��सतह�के �नीचे�दबी�च�ान��से�ख�नज��को��नकालने�क�����या�खनन�कहलाती�है।��छछली�गहराइय��म��दबे�ख�नज��को�सतह�क��परत
हटाकर��नकाल��लया�जाता�है��जसे�ओपन�का�ट�माइ�न�ग�कहते�ह�।�गहरे��छ���ज�ह��शा�ट�कहा�जाता�है, काफ��गहराई�म��दबे�ख�नज��न�ेप�
तक�प�ँचने�के ��लए��कए�जाते�ह�।�इसे�शा�ट�खनन�कहा�जाता�है।�पे�ो�लयम�और��ाकृ �तक�गैस�पृ�वी�क��सतह�के �ब�त�नीचे��मलते�ह�।�उ�ह�
�नकालने�के ��लए�गहरे��छ��बनाने�पड़ते�ह���जसे����ल�ग�कहते�ह�।�जो�ख�नज�सतह�के ��नकट�दबे,�होते
उ�ह��ह�आसानी�से��नकाल��लया�जाता
है, �जसे�उ�खनन�कहा�जाता�है।�

QUESTION 30.
�न�न�ल�खत�म��से�कौनसे�ख�नज�को�‘बेरीड�सनशाइन( Buried Sunshine )’ के �तौर�पर�संद�भ�त��कया�जाता�है�?

a) कोयला�
b) तांबा�
c) पारा�
d) पे�ो�लयम�
Correct Answer: A
AnswerExplanation

Solution (a)

कोयला�

यह�जीवा�म��म��ब�त��चुरता�म��पाया�जाता�है।�इसे�घरेलु��धन, लौह�और�इ�पात�जैसे�उ�ोग�, �ट�म�इंजन��तथा��व�ुत�उ�प��करने�हेतु��योग


�कया�जाता�है।�कोयले�से�उ�प���व�ुत�तापीय�उजा��कहलाती�है।�आज�जो�कोयला�हम��योग�कर�रहे�ह�, व �लाख��वष��पूव��तब�बना�था�जब
�वशाल�फन��पौधे�व�दलदली��े��पृ�वी�क��परत��के �भीतर�दब�गए�थे।�कोयले�को�इसी�लए�ब�रड�सनशाइन�कहा�जाता�है।�

QUESTION 31.
पवन�नवीकरणीय�उजा��का�एक���ोत�है।��ाचीन�काल�से�पवनच��कय��का��योग��कया�जाता�रहा�है।��न�न�म��से�कौनसे�काय��पवनच��कय��क�
सहायता�से��कये�जा�सकते�थे�?

1. अनाज�पीसना�

2. जल�ख�चना�

3. �व�ुत�उ�पादन�

4. ग�म�य��के �दौरान�लोग��को�वायु��दान�करना�

�न�न�ल�खत�म��से�सही�कू ट�का�चयन�क��जए�
:

a) 1,2 और�3
b) 2,3 और�4
c) 1,3 और�4
d) उपयु���सभी�
Correct Answer: A
AnswerExplanation

Solution (a)

पवन�उजा��
IASbaba
Score:
Web:
Email:
Page 90 150
https://telegram.me/UPSCMaterials https://telegram.me/FreeUPSCMaterials https://telegram.me/MaterialforExam

2019 - Test 2-Geography


Exam Title :
& C...
Email
Contact :

पवन�उजा��का�एक�न�समा�त�होने�वाला���ोत�है।�पवनच��कयां�पुराने�समय�से�ही�अनाज�पीसने�और�पानी�ख�चने�के ��लए��योग�म��लाई�जाती
रही�ह�।�आधु�नक�पवनच��कय��म��तेज�ग�त�क��पवन��च�क��को�घुमाती�ह��जो��व�ुत�उ�पादन�हेतु�एक�ज�न��(जेनरेटर) से�जुड़ी�होती�है।�अनेक
पवनच��कय��वाले��व�ड�फाम��उन�तट�य��े���तथा�पव�तीय�दर��म��लगाए�जाते�ह��जहाँ�तेज�और���थर�पवन��बहती�ह�।�नीदरल�ड्स, जम�नी,
डेनमाक�, ��टे न, अमे�रका�और��पेन�म��पाए�जाने�वाले��व�ड�फाम��अपने�पवन�उजा��उ�पादन�के ��लए�जाने�जाते�ह�।�भारत�का�त�मलनाडु �रा�य
नवीकरणीय�उजा��के �मामले�म���व��भर�म��अ�णी�है।�इसके ��व�ड�टरबाइन��क��संयु���था�पत��मता�7.9 गीगावाट�(GW) है�।

QUESTION 32.
पृ�वी�क��भूतापीय�उजा��के ��न�न�ल�खत�म��से�कौन
-से�कारण�है?

1. पृ�वी�के �आंत�रक�भाग�म��त�व��के �रे�डयोधम���य�के �कारण�

2. पृ�वी�क��आरं�भक�उ�मा�के �कारण�

3. सौर��व�करण�के �अवशोषण�के �कारण�

उपयु���म��से�कौन�से�कथन�सही�ह��?

a) 1 और�2
b) 2 और�3
c) 1 और�3
d) उपयु���सभी�
Correct Answer: A
AnswerExplanation

Solution (a)

भूतापीय�उजा��

भूतापीय�उजा��पृ�वी�से�उ�प��उ�मा�है।�यह��व�छ�और�सतत�है।�भूतापीय�उजा��के �संसाधन��म���छछली�सतह�से�लेकर�गम��जल�तथा�पृ�वी�क
�सतह�के �कु छ�मील�नीचे�पाई�जाने�वाली, उससे�भी�गहराई�म��पाई�जाने�वाली�उ�ण�च�ान�से�लेकर�मै�मा�नाम�क��उ�च�तापमान�वाली��पघली
च�ान�भी�शा�मल�होती�है।�

भूपप�ट��क��भूतापीय�उजा�
, �ह�के �मूल��व�यास�तथा�साम��य��के �त�व��के �रे�डयोधम���य� (वत�मान�म��अ�न��त��क�तु�संभवत
भूतापीय�उजा��का��नरंतर�चालन�करता�है।� : समान�मा�ा�म�)
से�उ�प��होती�है।�भूतापीय��वणता�जो��ह�के ��ोड़�और�इसक��सतह�के �तापमान�का�अंतर�होता�है
QUESTION 33. , जो��ोड़�से�सतह�तक�उ�मा�के ��प�म
� �?
‘ धा�रयां( Streak ) ’ ( ख�नज��के �संबंध�म��) के �संदभ��म���न�न�ल�खत�म��से�कौन�से�कथन�स�य�ह�

1. �कसी�ख�नज�के ��पसने�के �प�ात�बने�पाउडर�का�रंग�धा�रयां�कहलाता�है।�

2. धा�रय��का�रंग�ख�नज�के �समान�अथवा��भ��रंग�हो�सकता�ह�।�

�न�न�म��से�सही�कू ट�का�चुनाव�क��जए�

a) के वल�1
b) के वल�2
c) 1 और�2 दोन��
d) न�तो�1 न�ही�2
Correct Answer: C
Answer

IASbaba
Score:
Web:
Email:
Page 91 150
https://telegram.me/UPSCMaterials https://telegram.me/FreeUPSCMaterials https://telegram.me/MaterialforExam

2019 - Test 2-Geography


Exam Title :
& C...
Email
Contact :
Explanation

Solution (c)

धा�रयां�— �कसी�ख�नज�को��पसने�के �प�ात�पाउडर�के �रंग�को�संद�भ�त�करता�है।�

यह�ख�नज�जैसे�अथवा��भ��रंग�का�हो�सकता�है�— मैलकाइट�हरा�होता�है�और�हरी�धा�रयां��दान�करता�है, �लोराइट�ब�गनी�अथवा�हरा�होता


है��क�तु�सफ़े द�धा�रय��का��नमा�ण�करता�है।�

QUESTION 34.
�न�न�म��से�कौन�सा�कथन�‘ पे�ोलोजी�’ को�सही��कार�प�रभा�षत�करता�है�?

a) यह�मृदा��व�ान�है।�
b) यह�च�ान��से�संबं�धत��व�ान�है।�
c) यह�पे�ो�लयम�से�संबं�धत��व�ान�है।�
d) यह�कोयले�से�संबं�धत��व�ान�है।�
Correct Answer: B

Answer

Explanation

Solution (b)

पे�ोलोजी�च�ान��क���व�ान�को�कहते�ह�।�

एक�पे�ोलो�ज�ट�च�ान��के �सभी�पहलु��नामत
: ख�नज�संरचना, गठन, संरचना, मूल, उ�व, प�रवत�न�और�अ�य�च�ान��के �साथ�संबंध��

का
अ�ययन�करता�है।�

सो�चए�

मृदा��व�ान�(पेडोलॉजी) ?

a) पाव��तक�(35.
QUESTION orography )
b) महादे श�रचना�(
�न�न�ल�खत�म� Epeirogeny ) त��के
�से�कौन�सी����या�व�लत�पव� ��नमा�ण�का�कारण�है�
c) �वालामुखीयता�( Volcanism )
d) भूकं प�
Correct Answer: A

Answer

Explanation

Solution (a) , उसे�उठाती�ह��अथवा�इसके �भाग��का��नमा�ण�करती�ह�


, पटल�व�पण�के �अंतग�त�आती�ह�।
इनम���न�न�ल�खत�शा�मल�है�:
वे����याएं�जो�भूपप�ट��का��थान�प�रव�त�त�करती�ह
(i) ओरोजे�नक����या��म��गंभीर�वलन�के �मा�यम�से�पव�त��नमा�ण�तथा�भूपप�ट��क��लंबी�व�संकरी�प��य��को��भा�वत�करना�शा�मल�है; �

(ii) एपाइरोजे�नक����या��म��भूपप�ट��के ��वशाल�भाग��का�ऊपर�उठाना�और�मुड़ना�शा�मल�है
; �

(iii) आइसो�टे �टक�ग�त�व�धयाँ�पप�ट��का�संतुलन�बनाए�रखती�ह��;

(iv) �लेट��ववत�नीक��म��पप�ट�य��लेट��क���ै�तज�ग�त�व�ध�शा�मल�है।�

IASbaba
Score:
Web:
Email:
Page 92 150
https://telegram.me/UPSCMaterials https://telegram.me/FreeUPSCMaterials https://telegram.me/MaterialforExam

2019 - Test 2-Geography


Exam Title :
& C...
Email
Contact :
QUESTION 36.
�न�न�ल�खत�म��से�कौनसी����याएं�अपने��ेरक�बल�/उजा���के �साथ�सही�सुमे�लत�ह�?�

���या��ेरक�बल�

1. अप�य�आण�वक�तनाव�/ रासाय�नक���या�

2. वृहत��रण�गु��वाकष�ण�बल�

3. अपरदन�ग�तक�उजा��

�न�न�म��से�सही�कू ट�का�चयन�क��जए�
:

a) 1 और�3
b) 2 और�3
c) 1 और�2
d) उपयु���सभी�
Correct Answer: D

Answer

Explanation

Solution (d)

सभी�सही�सुमे�लत�ह�।� , बालू, आवण����तर


(रेगो�लथ) और�च�ान��कसी���मान�क��भां�त�ढलान�के �नीचे�क��ओर�गमन�करते�ह��।�ये�ऐसा�गु��वाकष�ण�के ��भाव�म��करते�ह���क�तु��नरंतर
वृ हत��रण�, �जसे�ढलान�ग�त�व�ध�अथवा�वृ
जल�और�जल�साम�ी�(जै हत�ग�त�व�ध�भी�कहते
से�उपसमु���पया�वरण�व�पं �ह�, एक�भू�रहते
कबहाव) से��भा�वत�होते -आक �ह�ृ �।�तक����या�है��जसके ��ारा�
मृ दा �जस�बात�का��यान�रखे�जाने�क��ज�रत�है, वह�ये��क�वृहत��रण�के वल�गु��वीय��ख�चाव�के �कारण�होता�
नोट�:

है , तथा��कसी�अ�य
बा��कारक�के �कारण�नह�।�

QUESTION 37.
लवण�अप�य�एक��कार�का�:

a) भौ�तक�अप�य�है�
b) रासाय�नक�अप�य�है�
c) जै�वक�अप�य�है�
d) उपरो��म��से�कोई�नह��
Correct Answer: A

Answer

Explanation

Solution (a)

लवण�अप�य�एक��कार�का�भौ�तक�अप�य�है।�
3
लवण�अप�य�o
0 से�50 C के �बीच�का�रे�ग�तान��म���मलने�वाला�उ�च�तापमान�लवण��व�तार�के �अनुकू ल�होता�है।�सतह�के ��नकट�के �रोम�छ���के �लवण
च�ान��म��उप��थत�लवण�तापीय���या, जलयोजन�और����टलीकरण�के �कारण�फै लता�है।�कै ��शयम , सो�डयम, मै�नी�शयम, पोटै �शयम�और
बे�रयम�जैसे�लवण��म��फै लने�क���कृ �त�होती�है।�इन�लवण��का��व�तार�तापमान�और�उनक��तापीय��वशेषता��पर��नभ�र�करता�है।�सतह�का�
IASbaba
Score:
Web:
Email:
Page 93 150
https://telegram.me/UPSCMaterials https://telegram.me/FreeUPSCMaterials https://telegram.me/MaterialforExam

2019 - Test 2-Geography


Exam Title :
& C...
Email
Contact :
���टल, च�ान��के �भीतर�वैय��क�रव��के ��वभाजन�का�कारण�बनते�ह��जो��वत
: अलग�हो�जाते�ह�।�वैय��क�रव��के �अलग�होने�क��इस����या
का�प�रणाम�कणीय��वखंडन�अथवा�कणीय�पण�न�के ��प�म��सामने�आ�सकता�है।�

QUESTION 38.
�वसप�, नद��के �माग��म��आने�वाले�स�प�ल�मोड़�ह��।��वसप��के �संदभ��म���न�न�ल�खत�म��से�कौन�से�कथन�स�य�ह�
? �

1. �वसप��अपरदना�मक�और��न�ेपा�मक�दोन�����या��का�प�रणाम�ह�।�

2. ये�नद��के �ऊपरी�माग��म���न�म�त�होते�ह�।�

�न�न�म��से�कू ट�चु�नए�

a) के वल�1
b) के वल�2
c) 1 और�2 दोन��
d) न�तो�1 न�ही�2
Correct Answer: A

AnswerExplanation

Solution (a)

�वसप��:

· एक��वसप��, आमतौर�पर�एक�स�प�ल�जलमाग��अथवा�नद��म��आने�वाला�मोड़�ह��।

· �वसप��अपरदना�मक�और��न�ेपा�मक�दोन�����या��का�प�रणाम�ह�।�एक��वसप��का��नमा�ण�तब�होता�है�जब�एक�धारा�का�बहता�जल
घाट��के �बाहरी��कनार��का�अपरदन�करता�है�और�इसे�चौड़ा�कर�दे ता�है�तथा�नद��के �आंत�रक�भाग��म��कम�उजा��तथा�गाद�का�जमाव�होता�है।�

· ये�आमतौर�पर�नद��के म�य�अथवा��नचले
� �माग��म��पाए�जाते�ह

�।� QUESTION 39.


�कसी�नद��चैनल�के �सबसे�गहरे��ब����को�जोड़ने�वाली�का�प�नक�रेखा�कहलाती�है�

a) �वसप��
b) थालवेग( Thalweg )
c) क�टु र( Contour )
d) आयसोहाइट( Isohyte )
Correct Answer: B

Answer

Explanation

Solution (b)

थालवेग�

भूगोल�व�नद��भूआकृ �तक��व�ान�म

� , थालवेग��कसी�घाट��अथवा�जलमाग��के �भीतर��न�नतम�उभार�क��रेखा�होती�है�।�अंतरा��ीय��व�ध�के
अंतग�त, एक�थालवेग�एक�जलमाग��के ��ाथ�मक�नौवहन�यो�य�चैनल�का�म�यभाग�होता�है, जो�दे श��के �बीच�सीमा�रेखा�प�रभा�षत�करता�है�।

IASbaba
Score:
Web:
Email:
Page 94 150
https://telegram.me/UPSCMaterials https://telegram.me/FreeUPSCMaterials https://telegram.me/MaterialforExam

2019 - Test 2-Geography


Exam Title :
& C...
Email
Contact :

QUESTION 40.
�न�न�म��से�कौन�सा�कथन�भू-संतुलन�( Isostacy ) को�सही��प�से�प�रभा�षत�करता�है�?
Explanation
a) भू-संतुल(d)
Solution न�एक����या�है��जसके ��ारा�महासागरीय��थलमंडल�सबड�शन
( subduction ) �े���म��समा�हत�हो�जाते�ह�।�
आइसो�टसी�
b) भू-संतुलन, म�य�महासागरीय�कटक��पर�महासागरीय�पप�ट��का��नमा�ण�है।�
भूc)-संतभूुल-न�(आइसो�टसी)
संतुलन, कठोर�पप�, टपृ��के �सव��च�भाग�पर���थत�पृ
�वी�क �वी�के �ऊपरी�म�टल�का�संतुलन�है�।सै� �ां�तक�संतुलन�होता�है�जैसे��क�वे�अपने�से�
े ��थलमंडल�के �सभी��वशाल�भाग��का�एक�ऐसा�आदश�
d) भू-संतुलन�एक�प�रघटना�है�जो�पृ�वी�क��कठोर�पप�ट��के �ऊपरी�म�टल�के �सव��च�भाग�पर�संतुलन�क���ा�या�करता�है।�
सघन
Correct Answer: D
�नचली�परत�(अथा�त��ब�लतामंडल, जो�ऊपरी�म�टल�का�एक�भाग�होता�है�और�कमजोर, �ला��टक�च�ान�से��न�म�त�होता�है�तथा�जो�सतह�से लगभग�
110
Answer �कमी�या�70 मील�नीचे���थत�होता�है�) पर�तैर�रहे�ह�।�भू-संतुलन�महा��पीय�तथा�महासागरीय�तल��क���े�ीय�ऊँ चाइय��को उनक��आधार�
च�ान��के �घन�व�के �अनुसार��नयं��त�करता�है।�
QUESTION 41.
नीचे�बताया�गया��व�डयो�भू-संतुलन�क��अवधारणा�को�और�अ�धक��प��
�न�न�म��से�कौन�‘�े टो�नक�ल�ड�के (पCratonic Landscape )’ क��सही��ा�या�करता�है�?
बनाएगा।�https://www.britannica.com/science/isostasy-
a) अ�पाइन�पव�त��ृंखला��के �त�ण��े��
b) म�य�महासागरीय�कटक�के �समीप���थत��े��
geology
c) हॉट��पॉट�ग�त�व�ध�के ��े��
d) पुराने�और��ववत��नक��प�से�महा��प�के ���थर�भाग�
Correct Answer: D
Answer

IASbaba
Score:
Web:
Email:
Page 95 150
https://telegram.me/UPSCMaterials https://telegram.me/FreeUPSCMaterials https://telegram.me/MaterialforExam

2019 - Test 2-Geography


Exam Title :
& C...
Email
Contact :
Explanation

Solution (d)

�े टन�

�े टन, �कसी�महा��प�का���थर�आंत�रक�भाग�होता�है�जो��ाचीन����टलाइन�बेसम�ट�च�ान�से��न�म�त�होता�है।��े टन�श�द�का��योग�ऐसे��े��


को�मोबाइल��जयो�स�कलाइनल��ॉ�स�(अवसाद�जमाव�क��रेखीय�प��याँ, �जनके �धंस�जाने�क��संभावना�होती�है) से�अलग��दखाने�के ��लए
�कया�जाता�है�। महा��प��के ��व�तृत�क� ��य��े टन�म��शी�ड् स�और��लेटफॉ�स��दोन��हो�सकते�ह�।�एक�शी�ड, �े टन�का�वह�भाग�होती�है��जसम�
सामा�यत: सतह�पर��ीक� ��यन�च�ान���ापक��प�से��दखाई�दे ने�लगती�ह�।�इसके �उलट
, एक��लेटफाम��म��बेसम�ट, �ै�तज�अथवा�उप�ै�तज
अवसाद���ारा�ढक��होती�है।�

उपरो��मान�च��म��नारंगी/लाल��े���व��के ��े टन��े���को�दशा�ते�ह�।�

QUESTION 42.
भूपप�ट��पर��ंशन�और��वभंग�पाए�जाते�ह�।��न�न�ल�खत�म��से�कौन�से�कथन��ंशन�के �संदभ��म��?सही�ह��

1. वे�के वल�तनाव�के �कारण�बनते�ह�।�

2. �ंशन�के �कारण��ंश�घा�टयां��न�म�त�होती�ह�।�

3. आ�नेय�च�ान��म���ंशन�के �प�रणाम�व�प�अ�भन�तय��एवं�अपन�तय��का��नमा�ण�होता�है।�

4. संपीडन�क��दशा�म���ु��म��ंश��न�म�त�होते�ह�।�

�न�न�म��से�सही�कू ट�का�चयन�क��जए�
:

a) 1, 2 और�3
b) 2, 3 और�4
c) 2 और�4
d) 1, 3 और�4
Correct Answer: C
AnswerExplanation

IASbaba
Score:
Web:
Email:
Page 96 150
https://telegram.me/UPSCMaterials https://telegram.me/FreeUPSCMaterials https://telegram.me/MaterialforExam

2019 - Test 2-Geography


Exam Title :
& C...
Email
Contact :

Solution (c)

�ंशन�:
एक��ंशन�भूपप�ट��म��तनाव�बल�के �कारण�आई�एक�दरार�होता�है।�यह�संपीडन�के �कारण�कठोर�और�भुरभुरी�च�ान��म��भी�हो�सकता�है।�

· जब�तनाव�होता�है, भूपप�ट��म��दरार�आती�है।�एक��लॉक�ऊपर�क��ओर�तथा�अ�य�नीचे�क��ओर�फ� क��दया�जाता�है।�ऊपर�क��ओर�फ� का�गया


�लॉक�को�हो�ट� �कहा�जाता�है�जब�क�नीचे�क��ओर�फ� के �गए��लॉक�को�
�बन�कहा�जाता�है।��जस�रेखा�के �साथ��ंशन�होता�है�उसे���ाइक�कहते
ह�।�इस��ंशन�को�सामा�य��ंशन�कहा�जाता�है�और�यही�आमतौर�पर�पाया�जाता�है।�एक�सामा�य��ंशन�क��अव�था�म�, खड़ी�ढाल�के ��प�म��नई
सतह�का��नमा�ण�होता�है।�

· कठोर�च�ान��के �संबंध�म��जहाँ�वलन�क��बजाए�संपीडन�होता�है, �ंशन�बनते�ह�।��नलं�बत�द�वार�वाला��लॉक�ऊपर�क��ओर�फ� क��दया�जाता


है�जब�क��नचली�द�वार�(फु टवाल) वाला��लॉक�नीचे�क��ओर�फ� क��दया�जाता�है।�इसे��ु��म��ंश�कहते�ह�।�एक��ु��म��ंश�क��दशा�म�
सतह�का��वशु���वंस�होता�है।�

· जब�बल�एक��सरे�के �समानांतर�काय��कर�रहे�होते�ह�
, �ंशन�रेखा�के �साथ��लॉक
, �बना�नीचे�अथवा�ऊपर�गए�एक��सरे�के �पास�से��नकल
जाते�ह�।�यह�पा���क��ंशन�कहलाता�है।�

�ंशन�के �कारण�बनी�भूआकृ �तयाँ:�ह��लॉक�पव�त, �ंश�घाट�, �टे प�पव�त, �ह�ज�पव�त, कत�नी��ंश�आ�द�।

QUESTION 43.
�कलाउआ( Kilauea ) �वालामुखी�अपनी��नरंतर�स��यता�एवं�लावा��नग�म�के �कारण�कु छ�समय�से�सु ख़�य��म��रहा।�यह�कहाँ���थत�है
? �

a) हवाई�
b) जापान�
c) इटली�
d) इंडोने�शया�

IASbaba
Score:
Web:
Email:
Page 97 150
https://telegram.me/UPSCMaterials https://telegram.me/FreeUPSCMaterials https://telegram.me/MaterialforExam

2019 - Test 2-Geography


Exam Title :
& C...
Email
Contact :
Correct Answer: A

AnswerExplanation

Solution (a)

�कलाउआ��वालामुखी�सं. रा. अमे�रका�के �हवाई�रा�य�म��है।�

https://www.theguardian.com/us-news/2018/jul/24/hawaiis-kilauea-volcano-eruption-could-last-
for-years-geologists-warn

https://www.youtube.com/watch?v=xgiZCrkRMhE

QUESTION 44.
�न�न�ल�खत�म��से�कौन�सी�भेदक( intrusive ) �वालामुखीय�भू-आकृ �त�
है�?

1. �े टर�

2. �सल�

3. डाइक�

4. बैथो�लथ�

�न�न�म��से�कू ट�का�चयन�क��जए�
:

a) 1, 2 और�3
b) 2, 3 और�4
c) 1, 3 और�4
d) उपयु���सभी�
Correct Answer: B

AnswerExplanation

Solution
:
(b)

भेदक��वालामुखीय�भूआकृ �त�

IASbaba
Score:
Web:
Email:
Page 98 150
https://telegram.me/UPSCMaterials https://telegram.me/FreeUPSCMaterials https://telegram.me/MaterialforExam

2019 - Test 2-Geography


Exam Title :
& C...
Email
Contact :

भूपप�ट��के �भीतर�मै�मा�के �ठं डे�होने�से�बनी�च�ान�


‘ �लू
� टो�नक�च�ान��’ अथवा�‘ भेदक�’ भूआकृ �तयाँ�कहलाती�ह�।�

बैथो�लथ�: बैथो�लथ�सतह�के �नीचे�ब�त�गहराई�म��जब��न�म�त�होती�ह��जब�मै�मा�क���वशाल�मा�ा�ठं डी�होकर�ठोस��प�धारण�कर�लेती�है।


चूँ�क�मै�मा�धीरे-धीरे�ठं डा�होता�है, बड़े�रवे�च�ान��(जैसे��ेनाइट) का��नमा�ण�करते�ह�।�बैथो�लथ��ाय: गुंबदाकार�होते�ह��और�प�ातवत��अपरदन
�ारा�अनावृत�हो�जाते�ह�।�

लेको�लथ�: कई�बार�मै�मा�क��छोट��मा�ा�एक�ल�स�आकृ �त�का��नमा�ण�करती�है�जो�च�ान�क��परत��के �बीच�धंसा�होता�है�।�तब�यह�ऊपरी


सं�तर�( च�ान�क��परत�) पर�ऊपर�क��ओर�चापाकार�आकृ �त�लेने�व�गुंबद�का��नमा�ण�करने�का�दबाव�डालता�है।�इसी��वशेषता�को�लेको�लथ�
कहा�जाता�है�और�यह�प�ातवत��अप�य�एवं�अपरदन�क��ओर�अनावृत�होकर�पहा�ड़य��क��छोट���ृंखला��का��नमा�ण�करती�है।�उदाहरण,
�कॉटल�ड�क��सीमा��पर�बनी�ए�डन�पहा�ड़यां।�

डाइक�: ये��ै�तज�ठं डी�दरार��वाले�लंबवत�भेदन�ह�।�ये�च�ान��के �बे�ड�ग��ले�स�तक��व�तृत�होते�ह���जनम��इनका�भेदन�हो�गया�होता�है।�डाइक�प्


राय: समूह�म���न�म�त�होते�ह��जहाँ�इ�ह��डाइक��वा�स��कहा�जाता�है।�

IASbaba
Score:
Web:
Email:
Page 99 150
https://telegram.me/UPSCMaterials https://telegram.me/FreeUPSCMaterials https://telegram.me/MaterialforExam

2019 - Test 2-Geography


Exam Title :
& C...
Email
Contact :

मेघालयन�काल�

भूवै�ा�नक�टाइम��के ल�म��मेघालयन�काल�सबसे�नवीन�है�अथा�त�चार�भाग��
( �वाटन�री�) म��सबसे�ऊपरी�चरण�है।�यह�होलोसीन�युग�अथवा
�ृंखला�के �तीन�उप�वभाजन��म��सबसे�ऊपरी�अथवा�नवीन�भी�है।�इसका��लोबल�बाउं�ी���ै टोटाइप�से�शन�एंड�पॉइंट(GSSP)
� पूव��र�भारत
के �मेघालय�क��माव�लुह�गुफाएं�ह�।�

मेघालयन�काल�आज�अथा�त�1950 से�4,200 वष��पूव��आरंभ�होता�है�, �जससे�1950 के �बाद�से�ऐ��ोपोसीन�क��रचना�क��संभावना


खुलती�है।�यह�काल�200- वष��के �अकाल�के �साथ�आरंभ��आ��जसने��म��, यूनान, सी�रया, फ़ली�तीन, मेसोपोटा�मया, �स�धु�घाट��एवं
यांग�जी�नद��घाट��म��मानव�स�यता�को��भा�वत��कया।�

इंटरनेशनल�यू�नयन�ऑफ़��जयोला�जकल�साइंसेज�के �महास�चव��टै नले��फ�े�के �अनु" यह�त�य��क�इस�काल�क


सार� े �आरंभ�का�संयोग�एक
वै��क�जलवाय�वक�घटना��ारा��ए�एक�सां�कृ �तक�प�रवत�न�के �साथ�जुड़ता�है
, इसे��व�श��बनाता�है”।�

ICS �ारा�आ�धका�रक��प�से�इस�काल�क��अ�भपु���जुलाई�2018 म���ीनल��डयन�तथा�नॉथ����पयन�के �साथ�क��गई�थी।�

QUESTION 51.
‘ बेस�इरोजन�तथा��ॉ�फट��श��टं ग�(Base Erosion and Profit Shifting) ’ के �बारे�म���न�न�ल�खत�कथन��पर��वचार�क��जए।�

1. यह�ब�रा�ीय�कं प�नय���ारा��कसी�दे श�के �आनुवां�शक�संसाधन��के �दोहन�से�संबं�धत�है।�

2. BEPS प�रयोजन�का��नमा�ण��व��आ�थ�क�मंच�तथा�संयु��रा��पया�वरण�काय��म�
(UNEP) �ारा��कया�गया�है।�

सही�कथन�का�चुनाव�क��जए�

a) के वल�1
b) के वल�2
c) 1 और�2 दोन��
d) न�तो�1 न�ही�2
Correct Answer: D
Answer
Explanation

Solution (d)

बेस�इरोजन�तथा��ॉ�फट��श��टं ग�(BEPS)

· इससे�आशय�उन�कर�कं चन�रणनी�तय��से�जो�कृ ��म��प�से�लाभ��को�कम�अथवा��बना�कर�वाले��थान��क��ओर��थानांत�रत�करने�हेतु�कर


�नयम��म��मौजूद�अंतराल��एवं�असंतुलन��का�दोहन�करती�ह�।�

· BEPS पर�समावेशी��े मवक,� �115 से�अ�धक�दे श��और�उनके ��याया�धकार��े���को�


OECD/ जी�20 बेस�इरोजन�तथा��ॉ�फट��श��टं ग�
(BEPS) पैके ज�के �काया��वयन�हेतु�साथ�लाता�है।�

· बेस�इरोजन�तथा��ॉ�फट��श��टं ग�पर�OECD/G20 का�समावेशी��े मवक� �और�इसके �सद�य�


, BEPS प�रयोजना�के �मा�यम�से��वीकृ त
उपाय��(जो�ब�रा�ीय�उ�म��के �आ�ामक�कर��नयोजन��वहार��को�ल��त�करते�ह�) का�काया��वयन�सु�न��त�कर�कर�वंचना�से��नपटते�ह�।�

· �वशेषकर�, चार�“ �यूनतम�मानक�” BEPS उपाय��के �मूल�म��ह��


: हा�नकारक�कर��वहार�, सं�ध���पयोग�, दे श�दार�दे श��रपो�ट�ग�तथा
fair-trade/article24505566.ece
�ववाद�समाधान�तं�।�

QUESTION 52.
Source: https://www.thehindu.com/business/us-reacting-to-beps-with-barriers-to-free-
�न�न�ल�खत�म��से�यूने�को�क��मानवता�क��अमू���सां�कृ �तक�धरोहर
( UNESCO’s List of the Intangible Cultural
Heritage of Humanity )’ क��सूची�म��शा�मल��कया�गया�है�?

IASbaba
Score:
Web:
Email:
Page 106 150
https://telegram.me/UPSCMaterials https://telegram.me/FreeUPSCMaterials https://telegram.me/MaterialforExam

2019 - Test 2-Geography


Exam Title :
& C...
Email
Contact :

1. कु ��य�म�

2. रामलीला�

3. कुं भ�मेला�

सही�कू ट�का�चयन�क��जए�:

a) 1 और�2
b) के वल�2
c) 1, 2 और�3
d) 1 और�3
Correct Answer: C

Answer

Explanation

Solution (c)
शा�मल��कया�गया�था।�
कुं भ�मेले�को�2017 म��यूने�को�क��मानवता�क��अमू���सां�कृ �तक�धरोहर�क��सूची�म��डाला�गया�था।�रामलीला�तथा�कु ��य�म�को�
Source: http://pib.nic.in/PressReleseDetail.aspx?PRID=1539899 (Implied Question) 2008 म

सु ख़�य��का�शीष�क�: ‘Safeguarding the Intangible Cultural Heritage and Diverse Cultural Traditions of
India’

· काया��वयन�– सं�कृ �त�मं�ालय�

· उ�े �य�– �व�भ��सं�थान��, समूह��, ���य��इ�या�द�को�पुनज�वन��दान�करना�ता�क�वे�भारत�क��समृ��अमू���सां�कृ �तक�धरोहर�को


मजबूत�बनाने�, संर��त�करने�और��ो�सा�हत�करने�वाली�ग�त�व�धय�/प�रयोजना��से�खुद�को�जोड़�सक� ।�

· यह�योजना�सं�कृ �त�मं�ालय�के ��वायत�संगठन�संगीत�नाटक�अकादमी��ारा�काया���वत�क��जा�रही�है।�

कु ��य�म�- https://ich.unesco.org/en/RL/kutiyattam-sanskrit-theatre-00010

रामलीला�- https://ich.unesco.org/en/RL/ramlila-the-traditional-performance-of-the-
ramayana-00110

कुं भ�मेला�- https://ich.unesco.org/en/RL/kumbh-mela-01258

QUESTION 53.
�न�न�ल�खत�म��से�कौन-से�भारतीय�रा�य�‘भूटान’ के �साथ�सीमा�बनाते�ह��?

1. प��मी�बंगाल�

2. �स��कम�

3. अ�णाचल��दे श�

4. मेघालय�

सही�कू ट�का�चयन�क��जए�:

a) के वल�1 और�2
b) के वल�2 और�3

IASbaba
Score:
Web:
Email:
Page 107 150
https://telegram.me/UPSCMaterials https://telegram.me/FreeUPSCMaterials https://telegram.me/MaterialforExam

2019 - Test 2-Geography


Exam Title :
& C...
Email
Contact :

c) के वल�1, 2 और�3
d) के वल�2, 3 और�4
Correct Answer: C

Answer
Explanation

Solution (c)

भूटान�से�भारत�क��सीमा�क��ल�बाई�699 �कमी�है�और�असम�(267 �कमी�), अ�णाचल��दे श�(217 �कमी�) , प. बंगाल�(183 �कमी�),


तथा��स��कम�(32 �कमी�) के �साथ�लगती�है।�

सो�चए�!

· �यांमार�के �साथ�सीमा�बनाने�वाले�भारतीय�रा�य�

Source: https://www.thehindu.com/opinion/op-ed/befriending-the-neighbour/
article24497565.ece

QUESTION 54.
‘AT2018Cow’ श�द�हाल�ही�म���कस�संदभ��को�लेकर�समाचार��म��रहा�था?

a) �वांटम�मैके �न�स�
b) MacOS उपकरण��पर�हमला�करने�वाला�नव�वक�सत�रैनसमवेयर�
c) एक�खगोलीय��प�ड�
d) ��वट् ज़रल�ड�म��खोजी�गई�गाय��तथा�भ�स��को��भा�वत�करने�वाली�बीमारी�
Correct Answer: C

Answer

Explanation

Solution (c)

AT2018cow

· यह�एक�बेहद�चमक�ला�खगोलीय��प�ड�है�जो�CGCG 137-068 नामक�छोट��आकाशगंगा�के ��नकट�दे खी�गई।�

· यह�एक�श��शाली�खगोलीय��व�फोट�है�जो�एक�सामा�य�सुपरनोवा�से�10 – 100 गुना�अ�धक�चमक�ला�है।�

· यह�Ic �कार�का�सुपरनोवा�है।�यह�एक�ऐसे�अ�यंत�बड़े�तारे�के ��व�फोट�से�होता�है�जो�अपनी�हाइ�ोजन�और�ही�लयम�का�बाहरी�आवरण�


खो�चुके �ह�।�

सुपरनोवा�

· यह�एक���णक�खगोलीय�घटना�है�जो��कसी�तारे�(जो�या�तो�एक�बड़ा�तारा�होता�है�या�एक��ेत�वामन��जसका��वनाश�एक�अं�तम��वशाल
�व�फोट��ारा����गोचर�होता�है) के �जीवन�के �अं�तम�न��ीय��वकास�चरण�के �दौरान�होती�है।�

· इससे�����से�धीरे-धीरे�ओझल�होने�से�पूव��कई�ह�त��या�महीन��या�वष��तक�अचानक�एक�
" नया�" चमक�ला�तारा��दखाई�दे ता�है।�

सो�चए�!

· एटलस�टे �ल�कोप�
science/article24499199.ece
Source: https://www.thehindu.com/todays-paper/tp-opinion/the-deep-and-far-of-

IASbaba
Score:
Web:
Email:
Page 108 150
https://telegram.me/UPSCMaterials https://telegram.me/FreeUPSCMaterials https://telegram.me/MaterialforExam

2019 - Test 2-Geography


Exam Title :
& C...
Email
Contact :
QUESTION 55.
‘ फॉरेन�डायरे�ट�इ�वे�टम�ट�कॉ��फड�स�इंडे�स( Foreign Direct Investment Confidence )’ �का�शत��कया�जाता�है�

a) �व��आ�थ�क�मंच��ारा�
b) �व���ापार�संगठन��ारा�
c) �व��ब�क��ारा�
d) उपरो��म��से�कोई�नह��
Correct Answer: D

Answer

Explanation

Solution (d)

फॉरे न�डायरे
दे खता�है �ट�इ�वे
) जो�ए. ट�.�टम� ट�कॉ��फड�
�कयन� ��ारा�तैसय�इं डे�स�एक�वा�ष�
ार��कया�गया�है ।�क�सव��(जो�1000 वै��क�कं प�नय��कCEOs,
े � CFOs, तथा�अ�य�उ�च�
काय�कारी अ�धका�रय��के ���य���वदे शी��नवेश�इराद��और��ाथ�मकता��पर�संभा�वत�राजनी�तक� , आ�थ�क�और��व�नयामक�य�प�रवत�न��के ��भाव�को
भारत�को�2018 के �सूचकांक�म��11 व��र�क��मली�थी।�

भारत�को�इस�सूचकांक�म���सरे�सबसे�बड़े�उभरते�बाजार�के ��प�म��र��क�ग�द��गई।�हाल�के �कई�सुधार��ने�इसके ��व�नयामक�य�वातावरण�को


अ�धक�कारोबार-�हतैषी�बना��दया�है�तथा�आ�थ�क�संवृ���के �इस�वष�:�पुलौटने
न �क��भ�व�यवाणी�क��गई�है।�

�या�आप�जानते�ह��?

· �वदे शी��नवेश�संवध�न�बोड��(FIPB) क��समा��त�के �प�ात�सरकार�के ����गत��वभाग��को�DIPP (जो�आवेदन��को����यागत�करने


हेतु�मानक��चालन����याएं�भी�जारी�करेगा) के �साथ�परामश��के �प�ात�
FDI ��ताव��को�मंजूरी�दे ने�क��श���दे �द��गई�है।�

· पा�क�तान�और�बां�लादे श�से�आने�वाले�सभी�FDI ��ताव��को�सुर�ा�एज��सय��का�अनुमोदन�लेना�आव�यक�होगा�तथा�छोटे �ह�थया��के


�व�नमा�ण�को�गृह�मं�ालय�अनुमो�दत�करेगा।�जहाँ�NRIs �ारा�और�खुदरा�व��नया�तो�मुख�इकाईय���कया�गया��वदे शी��नवेश�DIPP �ारा
अनुमो�दत��कया�जाएगा�वह��ब�क��म���कया�गया�FDI �व�ीय�सेवाएं��वभाग��ारा�अनुमो�दत��कया�जाएगा।�DIPP अथवा�आ�थ�क�मामले
�वभाग�FDI ��ताव��क���तमाही�समी�ा�करेगा।�

· जहाँ��व�ीय�सेवा��का��कसी��व�नयामक��ारा��व�नयमन�नह��है�अथवा�जहाँ�एक�से�अ�धक��व�नयामक�है�अथवा��जसके �संबंध�म�
�व�नयामक�को�लेकर���थ�त��प��नह��है�, उ�ह���व��मं�ालय�के �आ�थ�क�मामले��वभाग��ारा�अनुमो�दत��कया�जाएगा।�

Source: https://www.thehindu.com/business/Economy/india-drops-three-ranks-in-at-
kearney-fdi-confidence-index/article24497785.ece

QUESTION 56.
‘ शेकटकर�स�म�त�’ �न�न�ल�खत�म��से��कससे�संबं�धत�नह��है�?

1. सश���बल��क��यु�क��मता�बढाने�से�

2. सश���बल��के �र�ा��य�के �पुनस�तुलन�से�

3. भारत-पा�क�तान�सीमा�पर�बाड़बंद��करने�से�

सही�कू ट�का�चयन�क��जए�:

a) 1 और�3
b) 2 और�3
c) के वल�3
d) के वल�2

IASbaba
Score:
Web:
Email:
Page 109 150
https://telegram.me/UPSCMaterials https://telegram.me/FreeUPSCMaterials https://telegram.me/MaterialforExam

2019 - Test 2-Geography


Exam Title :
& C...
Email
Contact :
Correct Answer: C
AnswerExplanation

Solution (c)

र�ा�मं�ालय�ने�ले��टन�ट�जनरल�(से.�न.) डॉ. डीबी�शेकटकर�क��अ�य�ता�म���वशेष���क��एक�स�म�त�का�गठन��कया�है��जसे�‘टू थ�टू �टे ल


अनुपात’ (एक�सै�य�श�दावली) बढाने�के �उ�े �य�के �साथ�सश���बल��क��यु�क��मता�बढाने�तथा�र�ा��य�के �पुनस�तुलन�का�अ�धदे श��दया
गया�है।�

Recommendations & Source: http://pib.nic.in/newsite/PrintRelease.aspx?relid=180848

QUESTION 57.
‘ युगांडा�’ के �बारे�म���न�न�ल�खत�कथन��पर��वचार�क��जए।�

1. यह�अ��का�के �महान�झील��े��का�भाग�है।�

2. इसक��सीमा�के �या, रवांडा�और�बु�ं डी�से�लगती�है।�

सही�कथन�का�चुनाव�क��जए:

a) के वल�1
b) के वल�2
c) 1 और�2 दोन��
d) न�तो�1 न�ही�2
Correct Answer: A
AnswerExplanation

Solution (a)

युगांडा�पूव��अ��का�म��एक�भूआब��दे श�है।�इसक��सीमा�पूव��म��के �या� �द��णी�सूडान�, तथा�प��म�म��कांगो�लोकतां��क�गणरा�य�,


, उ�र�म�
द��ण-प��म�म��रवांडा�एवं�द��ण�म��तंजा�नया�से�लगती�है।�दे श�के �द��णी�भाग�म���व�टो�रया�झील�का�एक�बड़ा��ह�सा�शा�मल�है�जो�के �या
और�तंजा�नया�से�भी�लगता�है।�युगांडा�अ��का�के �महान�झील��े��म��पड़ता�है।�युगांडा�नील��ोणी��े��म��भी�पड़ता�है�तथा�यहाँ��भ�
-�भ�
�क�तु�सामा�यत: प�रव�त�त��वषुवतीय�जलवायु�है।�

अ��का�क��महान�झील��, झील��क��एक��ृंखला�का�नाम�है�जो�पूव��अ��क���ंश�के �इद� �गद� ��ंश�घाट��झील��का�भाग�है।�इनम���व�टो�रया


झील�(�े�फल�के ��हसाब�से�साफ़�पानी�क���व��क��तीसरी�सबसे�बड़ी�झील ) तथा�तांगा�नका�झील�(आयतन�और�गहराई�के ��हसाब�से�साफ़
पानी�क���व��क���सरी�सबसे�बड़ी�झील) शा�मल�ह�।�अ��का�के �महान�झील��े�(� �जसे�आमतौर�पर�महान�झील��े��भी�कहा�जाता�है�
) के दे श��म��बु�,ं कांडी�
गो�लोकतां��क�गणरा�य�, के �या�, रवांडा�, तंजा�नया�और�युगांडा�शा�मल�ह�।�

IASbaba
Score:
Web:
Email:
Page 110 150
https://telegram.me/UPSCMaterials https://telegram.me/FreeUPSCMaterials https://telegram.me/MaterialforExam

2019 - Test 2-Geography


Exam Title :
& C...
Email
Contact :

Source: https://economictimes.indiatimes.com/news/politics-and-nation/india-
donates-200-cows-for-rwandan-economic-development-project/
articleshow/65116913.cms

QUESTION 58.
‘ से�लको�न�या( Salicornia ) ’ के �बारे�म���न�न�ल�खत�कथन��पर��वचार�क��जए।�

1. आं��दे श�उ�च��यायालय�ने�रा�य�सरकार�को�से�लको�न�या�पर���तबंध�लगाने�संबंधी�कानून�बनाने�का�आदे श��दया�है��य��क�यह�जल


�नकाय��म���वषैले�रसायन��नमु���करता�है�जो�रा�य�म���वा��य�सम�या��का�कारण�बन�रहे�ह�।�

2. यह�म��ोव�आ�� भू�मय��के �लवणीय�दलदली��े���म���चुरता�म��पाया�जाता�है।�

सही�कथन�का�चुनाव�क��जए:

a) के वल�1
b) के वल�2
c) 1 और�2 दोन��
d) न�तो�1 न�ही�2
Correct Answer: B

Answer

Explanation

Solution (b)

से�लको�न�या�

सु ख़�य��म��: आं��दे श��व�भ���व�धय��के �मा�यम�से�इस�लवण�का��थानाप���ा�त�करने�क��संभावनाएं�तलाश�रहा�

है।� त�य�

· यह�गूदेदार�, लवण-स�ह�णु�पु�प�पादप��क��एक��जा�त�है।�
IASbaba
Score:
Web:
Email:
Page 111 150
https://telegram.me/UPSCMaterials https://telegram.me/FreeUPSCMaterials https://telegram.me/MaterialforExam

2019 - Test 2-Geography


Exam Title :
& C...
Email
Contact :

· यह�ऐसा�पौधा�है�जो�म��ोव�आ�� भू�मय��के �लवणीय�दलदली��े���म���चुरता�म��उगता�है।�

· यह�पौधा�सो�डयम�क��कम�मा�ा�वाले�नमक�के ��थानाप��का�उ�पादन�करता�है।�

· ये�पौधे�उ�री�गोला�� �एवं�द��णी�अ��का�म���ापक��प�से�पाए�जाते�ह��तथा�उपो�णक�टबंधीय�से�लेकर�उपआक� �टक��े���तक�इनका


�व�तार�है।�ये�द��णी�अमे�रका�व�ऑ��े �लया�म��नह���मलते।�

सो�चए�

· हैलोफाइट�

· �यूमैटोफोर�

Source: https://www.thehindu.com/todays-paper/state-to-tap-benefits-of-salicornia/
article24490729.ece

QUESTION 59.
�न�न�ल�खत�कथन��पर��वचार�क��जए।�

1. GDP �ड�लैटर�नो�मनल�GDP का�वा�त�वक�GDP से�अनुपात�होता�है।�

2. मु�ा�फ��त�समायोजन�के ��बना�आ�थ�क�उ�पादन�को�नो�मनल�
GDP कहते�ह�।�

3. वा�त�वक�GDP सामा�यत: नो�मनल�GDP से�अ�धक�होती�है।�

सही�कथन��का�चुनाव�क��जए�

a) 1 और�2
b) 2 और�3
c) 1 और�3
d) उपयु���सभी�
Correct Answer: A

Answer

Explanation

Solution (a)

GDP �ड�लैटर�

· इसे�अंत�न��हत��ाइस��ड�लैटर�भी�कहा�जाता�है।�

· यह�मु�ा�फ��त�का�मापक�है।�

· लयः��कसी�वष���वशेष�म���कसी�अथ��व�था��ारा�वत�मान�मू�य��पर�उ�पा�दत�व�तु��और�सेवा��के �मू�य�का�आधार�वष��के �मू�य��के �साथ


का�अनुपात�है।�

· यह�अनुपात�यह��दखाने�म��मदद�करता�है��क�सकल�घरेलु�उ�पाद�म��उ�पादन�वृ���क��बजाए�उ�च�मू�य��के �कारण��कस�सीमा�तक�वृ����ई
है।�

· चूँ�क��ड�लैटर�थोक�अथवा�उपभो�ा�मू�य�सूचकांक��क��सी�मत�व�तु�टोकरी�क��बजाए�अथ��व�था�म��उ�पा�दत�व�तु��और�सेवा��क
�स�पूण��र�ज�को�कवर�करता�है�, अत: इसे�मु�ा�फ��त�का�अ�धक��ापक�माप�माना�जाता�है।�

वा�त�वक�बनाम�नो�मनल�GDP

· GDP �ड�लैटर�वा�त�वक�GDP तथा�नो�मनल�GDP के �अंतर�को�मापता�है।�


IASbaba
Score:
Web:
Email:
Page 112 150
https://telegram.me/UPSCMaterials https://telegram.me/FreeUPSCMaterials https://telegram.me/MaterialforExam

2019 - Test 2-Geography


Exam Title :
& C...
Email
Contact :

· नो�मनल�GDP म��मु�ा�फ��त�शा�मल�नह��होती।�वा�त�वक�GDP म��मु�ा�फ��त�शा�मल�होती�है।�

· प�रणाम�व�प�, �कसी�बढ़ती�अथ��व�था�म��नो�मनल�GDP �ाय: वा�त�वक�GDP से�अ�धक�ही�होगी।�( मु�ा�फ��त�सामा�यत: एक


सकारा�मक�अंक�है�) ।

GDP मू�य��ड�लैट�र�का�सू��:

· GDP �ड�लैट�र�= ( नो�मनल�GDP ÷ वा�त�वक�GDP) x 100

WPI एवं�CPI

· उपभो�ा�मू�य�सूचकांक�(CPI) , एक��न��त�समय�म��प�रवार���ारा�उपभोग�के �उ�े �य�से�खरीद��जाने�वाली�व�तु��और�सेवा��के �मू�य


�तर��म��प�रवत�न��को�मापता�है।�

· हालां�क�, चूँ�क�CPI चु�न�दा�व�तु��क��टोकरी�पर�आधा�रत�है�और�इसम��शा�मल�मू�य��के �आधार�पर��नकाला�जाता�है


, यह� पूरी
अथ��व�था�क��मु�ा�फ��त�का�पता�नह��लगा�पाता।�

· थोक�मू�य�सूचकांक�टोकरी�सेवा��े��का��ब�कु ल���त�न�ध�व�नह��करती�और�इसक��सभी�मद� �के वल�व�तुए�ं ह���जनक��क�मत�/उ�पादक


�थोक
�तर�पर�सं�ान�म��ली�जाती�ह�।�

�ड�लैट�र�<> CPI

· उपभोग��ा�प��म��प�रवत�न�अथवा�व�तु��और�सेवा��के �आगमन�को��वत
: GDP �ड�लैट�र�म����त�ब��बत��कया�जाता�है।�

· इससे�GDP �ड�लैट�र�अथ��व�था�के �उपभोग�अथवा��नवेश��ा�प��म��आए�प�रवत�न��को�अवशो�षत�कर�लेता�है।�

· �ाय: , GDP �ड�लैट�र�के ��झान�


CPI के �स�श�ही�होते�ह�।�

· GDP �ड�लैट�र��य�के �अ�तन��ा�प��द�श�त�करता�है।�

· GDP �ड�लैट�र�GDP अनुमान��के �साथ��तमाही�म��एक�बार�जब�क�


CPI और�WPI डेटा���येक�माह�जारी��कया�जाता�है।�

Source: https://www.thehindu.com/business/Economy/what-is-the-gdp-deflator/
article24489279.ece

QUESTION 60.
यौन�अपराध��से�बालक��का�संर�ण�अ�ध�नयम�(POCSO) के �बारे�म��कौन�सा/से�कथन�स�य�है/ह��?

1. यह�अ�ध�नयम�16 वष��से�कम�आयु�के �����को�बालक�के ��प�म��प�रभा�षत�करता�है।�

2. बाल�यौन�शोषण�क��ऑनलाइन��शकायत�करने�हेतु�म�हला�एवं�बाल��वकास�मं�ालय��ारा�POCSO ई�- बॉ�स�लॉ�च��कया�गया�था।�

सही�कथन�का�चुनाव�क��जए:

a) के वल�1
b) के वल�2
c) 1 और�2 दोन��
d) न�तो�1 न�ही�2
Correct Answer: B

Answer

Explanation

Solution (b)

यौन�अपराध��से�बालक��का�संर�ण�अ�ध�नयम�
IASbaba
Score:
Web:
(POCSO)
Email:
Page 113 150
https://telegram.me/UPSCMaterials https://telegram.me/FreeUPSCMaterials https://telegram.me/MaterialforExam

2019 - Test 2-Geography


Exam Title :
& C...
Email
Contact :

समाचार�: बालक��पर�यौन�हमल��के �मामल��म���मलने�वाली�सजा�म��वृ���का���ताव�

· यह�ब�च��क��यौन�शोषण�, यौन�उ�पीड़न�और�पोन��ाफ��से�र�ा�हेतु�बनाया�गया�था।�

· यह�बाल-�हतैषी�सुनवाई�तं���न�म�त�करने�हेतु�बनाया�गया�था।��जससे�अपराधी�को�दं ड��दया�जा�सके ।�

· रा�ीय�बाल�अ�धकार�संर�ण�आयोग�(NCPCR) इसके �काया��वयन�क���नगरानी�करेगा।�

· यह�अ�ध�नयम�अठारह�वष��से�कम�आयु�के �����को�बालक�के ��प�म��प�रभा�षत�करता�है।�यह�ब�चे�को��या�यक�तं��के �हाथ��पु


: पी�ड़तन
बनने�से�रोकने�का�भी��ावधान�करता�है।�

· यह�ऐसे�मामल��क���रपोट� �करना�अ�नवाय��बनाता�है।�यह�अपराध�क��जानकारी�रखने�वाले������ारा�यौन�शोषण�क���रपोट� �करने�को�उसका


कानूनी�कत���बनाता�है।�

· यह�उन����य��के ��लए�भी�सजा�का��ावधान�करता�है�जो�यौन�उ�े �य��के ��लए�बालक��का���ा�पार�करते�ह�।�

· यह�कहता�है��क�बालक�का�सा�यांकन�तीस��दन��क��अव�ध�के �भीतर�होना�चा�हए।�

· मामले�का�सं�ान�लेने�वाले��वशेष��यायालय��को�उस�अपराध�का�सं�ान�लेने�के �एक�वष��क��अव�ध�के �भीतर�सुनवाई�पूरी�कर�लेनी�चा�हए।�

· यह��ावधान�करता�है��क��वशेष��यायालय��क��काय�वा�हयाँ�कै मरे�म���रकॉड��क��जानी�चा�हएं�और�सुनवाई�माता
-�पता�अथवा��कसी�ऐसे����
क��उप��थ�त�म��होने�चा�हए��जस�पर�बालक�का�भरोसा�हो।�

· अ�ैल�2018 का�अ�यादे श�- उन�अ�भयु���को�मौत�क��सजा�सुनाई�जाएगी��जन�पर�12 वष��आयु�तक�के �बालक�के �बला�कार�का


अ�भयोजन�है।�

· POCSO ई�- बॉ�स�POCSO अ�ध�नयम�, 2012 के �अंतग�त��कसी�यौन�शोषण�के �मामले�क���रपोट� �करने�का�एक�सरल�व���य�


मा�यम�है।�यह�NCPCR क��वेबसाइट�पर��मुख��प�से���यमान�है�जहाँ��यो�ा�को�के वल�
POCSO e-box नामक�बटन�दबाना�होता�है।
ऐसा�करने�पर�यह�हम��एक�पृ��पर�ले�जाएगा�जहाँ�एक�लघु�एनीमेशन��फ�म��दखाई�दे गी।�

Source: https://www.thehindu.com/todays-paper/tp-national/wcd-to-move-proposal-to-
amend-pocso-act/article24491808.ece

QUESTION 61.
हाल�ही�म����ता�वत�‘ �ीमान�योजना�’ का��या�उ�े �य�है�?

a) ��यात��वदे शी��श�ण�सं�थान��को�भारत�म��अपने�क� पस�खोलने�म��स�म�बनाना।�


b) �नजी��े��और�समुदाय�क��मदद�से�सरकारी��कू ल��म���श�ा�क��गुणव�ा�बढ़ाना।�
c) सरकार�के �शोध�ढाँचे�के ��भावी�उपयोग�से��व�भ��मु���का�समाधान�करना।�
d) �ाथ�मक�और�मा�य�मक��कू ल��क��बु�नयाद��ढ़ांचा�सु�वधा��को�सुधारने�हेतु��नजी����य��और�संगठन��से��वै��छक�मौ��क�सहयोग
को��ो�सा�हत�करना�।�
Correct Answer: C

Answer

Explanation

Solution (c) (Scientific Research Infrastructure


Management and Networks)
साइं�ट�फक��रसच��इं�ा���चर�फॉर�म�टेने�स�ऑफ�नेटवक�
समाचार�: �ीमान�, सरकारी��योगशाला��म��मौजूद�वै�ा�नक�उपकरण��को�एक��कयाया�आमदनी�दे ने�वाली�प�रसंप�तय��म��प�रव�त�त�करने
�का���ताव�करता�है।�

त�य�

IASbaba
Score:
Web:
Email:
Page 114 150
https://telegram.me/UPSCMaterials https://telegram.me/FreeUPSCMaterials https://telegram.me/MaterialforExam

2019 - Test 2-Geography


Exam Title :
& C...
Email
Contact :

· इसक��शोध�अवसंरचना�के ��भावी�उपयोग�हेतु��व�भ��मु���का�समाधान�करना।�

· यह�शोधकता���को�वे�सभी��योगशाला�उपकरण��कराए�पर�दे ने�क��योजना�बनाती�है��जनका�मू�य�₹10 लाख�से�अ�धक�है।�

उ�े �य�

· वै�ा�नक�उपकरण�और�अवसंरचना�क��खरीद�और�रख-रखाव�का�सरलीकृ त��क�तु�समझदारीपूण��तरीका।�

· दे श�भर�म��फै ले�एक�नेटवक� �के �मा�यम�से�भारत�सरकार�और�इसक��एज��सय���ारा��व�पो�षत�उपकरण��तक��ापक�प�ँच�उपल�ध


करवाना।�

· पुराने�पड़�चुके �उपकरण��और�अवसंरचना�के �बु��म�ापूण��तरीक��से��नपटान�का��े मवक� ��दान�करना।�

· शोध�अवसंरचना�के �संचालन�क��द�ता�को�सुधारने�और��ो�सा�हत�करने�हेतु�एक��े मवक� ��दान�करना।�

· A broad template to monitor utilization of expensive research infrastructure.

लाभ�

· यह�दे श�भर�के �वै�ा�नक��


, शोधकता���और�उ�ोग�पेशेवर��को��ापक�प�ँच�उपल�ध�करवाने�वाले��ासं�गक��हतधारक��को�जुटाकर�एक
�े�ीय�पा�रतं��के ��नमा�ण�को��ो�सा�हत�करेगा।�

· यह�उपकरण��के �संचालन�और�रखरखाव�हेतु���श��त�संचालक��के �एक�समूह�के ��नमा�ण�म��भी�सहायता�करेगा।�

Source: https://www.thehindu.com/education/rent-a-lab-policy-to-bring-revenues-to-
institutions/article24477344.ece

QUESTION 62.
हाल�ही�म��सु ख़�य��म��रहा�‘ �रवस��चाज��मैके �न�म
( Reverse Charge Mechanism ) ’ �कससे�संबं��है?

a) व�तु�और�सेवा�कर�(GST)
b) इ�न�शयल�प��लक�ऑफ�र�ग�(IPO)
c) अनाद�रत�(बाउंसड) चेक��पर�मुकदमा�
d) �े�ीय�कने��ट�वट��योजना�(RCS)
Correct Answer: A

Answer

Explanation

Solution (a)

GST �रवस��चाज��मैके �न�म� (RCM) का�मूल�अथ��है��क�GST को�व�तु�/सेवा��के ��ा�तकता���ारा�भुगतान��कया�और�सरकार�के


पास जमा�करवाया�जाना�है � न��क�व�तु �/सेवा��के �आपू�त�कता���ारा।�
Source: http://pib.nic.in/newsite/PrintRelease.aspx?relid=180825

QUESTION 63.
‘ कमसाले�’ के �बारे�म���न�न�ल�खत�कथन��पर��वचार�क��जए।�

1. यह�के वल�म�हला���ारा��कया�जाने�वाला�ओ�डशा�का�एक�लोक�नृ�य�है।�

2. यह�भगवान�जग�ाथ�के �भ����ारा��कया�जाता�है।�

सही�कथन�का�चुनाव�क��जए:

IASbaba
Score:
Web:
Email:
Page 115 150
https://telegram.me/UPSCMaterials https://telegram.me/FreeUPSCMaterials https://telegram.me/MaterialforExam

2019 - Test 2-Geography


Exam Title :
& C...
Email
Contact :

a) के वल�1
b) के वल�2
c) 1 और�2 दोन��
d) न�तो�1 न�ही�2
Correct Answer: D

Answer

Explanation

Solution (d)

कमसाले�

· यह�मैसूर��े��का�एक�लोक�नृ�य�है।�

· यह�एक��व�श��लोक�कला�है��जसका�मंचन�भगवान�महादे �र�के �भ����ारा��कया�जाता�है।�

· कमसाले�पीतल�से��न�म�त�एक�साधारण�वा��य��।�इसका�उदगम�पौरा�णक�समय�म��माना�जाता�है।�

· कमसाले�यं��म��एक�गारी�( �लेट�) और�एक�आ�द�ब�ालू�( कप�) लगा�होता�है।�जहाँ�गारी�आकाश�का��तीक�है�वह��आ�द�ब�ालू�पृ�वी�


का। एक�साथ�आने�पर�कमसाले�ताल�ई�र�क��संपूण��रचना�का�एक��तीक�है।�

· पारंप�रक��प�से�, के वल�पु�ष�इस�नृ�य�को��कया�करते�थे।�हालां�क�
, वत�मान�समय�म��लड़�कय���ारा�भी�इसे�सीखने�पर�कोई���तबंध�नह�
है।�
the-god/article24470913.ece

Source: https://www.thehindu.com/entertainment/dance/cling-clang-fly-high-all-for-
QUESTION 64.
�धान�मं�ी�सहज��बजली�हर�घर�योजना(सौभा�य) के �बारे�म���न�न�ल�खत�कथन��पर��वचार�क��जए।�

1. सामा�जक-आ�थ�क�एवं�जा�त�जनगणना�(SECC) 2011 म��पहचाने�गए�गरीब�प�रवार��को��न:शु�क��बजली�उपल�ध�करवाई�जाएगी।�

2. द�नदयाल�उपा�याय��ाम��यो�त�योजना�(DDUGJY) एवं�एक�कृ त��व�ुत��वकास�योजना�(IPDS) को�‘ सौभा�य�’ योजना�के


अंतग�त �मला��दया�गया�है।�

सही�कथन�का�चुनाव�क��जए�

a) के वल�1
b) के वल�2
c) 1 और�2 दोन��
d) न�तो�1 न�ही�2
Correct Answer: D

Answer

Explanation

Solution (d)

सौभा�य�: �धान�मं�ी�सहज��बजली�हर�घर�योजना�

· दे श�म��साव�भौम�घरेलु��व�ुतीकरण�हा�सल�करना।�

· �ामीण�और�शहरी��े���म��आ�खरी��ब���तक�सभी�घर��को��बजली�कने�शन�उपल�ध�करवाना।�

IASbaba
Score:
Web:
Email:
Page 116 150
https://telegram.me/UPSCMaterials https://telegram.me/FreeUPSCMaterials https://telegram.me/MaterialforExam

2019 - Test 2-Geography


Exam Title :
& C...
Email
Contact :

· SECC आंकड़��के �आधार�पर�कम�से�कम�एक�वंचना�वाले�शेष�अ�व�ुतीकृ त�घर��को��ामीण��े���म��तथा�आ�थ�क��प�से�गरीब�शहरी


प�रवार��को��न:शु�क��बजली�कने�शन��दया�जाएगा।�

· अ�य��से���त�प�रवार�500 �पए�क��रा�श��बल�म��दस�समान��क�त��म��ली�जाएगी।�

· �र�थ�और��ग�म��े���म����थत�घर��को�सोलर�फोटोवो�टे इक�(SPV) आधा�रत��णा�लयाँ�LED लाइट��, पंखे�, पॉवर��लग�इ�या�द�के �साथ


द��जाएँगी।�

· लाभा�थ�य��को�सामा�जक�आ�थ�क�एवं�जा�त�जनगणना�(SECC) 2011 के �आंकड़��के �आधार�पर�पहचाना�जाएगा।�

· ल�य�– दे श�म��31 माच��2019 तक�साव�भौम�घरेलु��व�ुतीकरण।�

· सौभा�य�योजना�DDUGJY व�IPDS के �अंतराल��को�भरने�तथा��वेश�बाधा�के �मु���के ��ापक�समाधान�


, अं�तम�घर�तक�कने�शन�, तथा
उन�सभी�अ�व�ुतीकृ त��ामीण�एवं�शहरी�घर��हेतु��बजली�कने�शन�जारी�करना।�

· ‘ सौभा�य�’ वेब�- पोट� ल�– साव�भौम�घरेलु��व�ुतीकरण�क��पारदश��तरीके �से��नगरानी�हेतु�एक��लेटफाम�।�

�या�आप�जानते�ह��?

द�नदयाल�उपा�याय��ाम��यो�त�योजना�(DDUGJY)

· यह��ामीण��े���म���व�ुत�आपू�त��क��गुणव�ा�और��व�सनीयता�को�सुधारने�हेतु��ाम�/अ�धवास��म��आधारभूत��व�ुत�अवसंरचना�का��नमा�ण
करने�, �व�मान�अवसंरचना�को�मजबूत�करने�, �व�मान�फ�डर��क��मीट�र�ग/�ांसफाम�स�/उपभो�ा��के ��वतरण�क��प�रक�पना�करती�है।�

· अं�तम�घर�तक�कने��ट�वट��और�के वल�रा�य���ारा�अपनी�सूची�के �आधार�पर�पहचाने�गए�


BPL प�रवार��को��न:शु�क��बजली�कने�शन�भी
उपल�ध�करवाए�जाते�ह�।�

एक�कृ त��व�ुत��वकास�योजना�(IPDS)

· शहरी��े���म��24x7 गुणव�ापरक�और��व�सनीय��व�ुत�आपू�त�।�

· यह��व�ुत�प�ँच��दान�करने�के ��लए�आव�यक�अवसंरचना��नमा�ण�का��ावधान�करता�है��क�तु�कु छ�घर��तक�उनके �कमजोर�आ�थ�क�हालात�


के �कारण�अभी�तक��बजली�नह��प�ँची�है�, �य��क�वे�कने�शन�के �आरं�भक��भार�दे ने�म��स�म�नह��ह�।�

Source: http://pib.nic.in/PressReleseDetail.aspx?PRID=1539203

QUESTION 65.
‘ ��े टे�जक�आ�स���रड�शन���ट�( Strategic Arms Reduction Treaty ) ’ �कनके �बीच�क��सं�ध�
थी
?

a) युगो�ला�वया�से�पृथ�क�होकर�बने�दे श��के �बीच�


b) ��तीय��व��यु��के �प�ात�जम�न,ी इटली�और��ांस�के �बीच�
c) सं.रा. अमे�रका�और�सो�वयत�संघ�(USSR) के �बीच�
d) जापान�और�सं.रा. अमे�रका�के �बीच�
Correct Answer: C

Answer

Explanation

Solution (c)

�टाट� �( ��े टे�जक�आ�स���रड�शन���ट��)

�टाट� �1

IASbaba
Score:
Web:
Email:
Page 117 150
https://telegram.me/UPSCMaterials https://telegram.me/FreeUPSCMaterials https://telegram.me/MaterialforExam

2019 - Test 2-Geography


Exam Title :
& C...
Email
Contact :

· यह�संहारक�साम�रक�ह�थयार��म��कमी�लाने�व�उ�ह��सी�मत�करने�हेतु�अमे�रका�और�सो�वयत�संघ�(USSR) के �बीच�क��एक���प�ीय�सं�ध
थी।�

· यह�इसके �ह�ता�र�कता���को�1,600 अंतर-महा��पीय�बै�ल��टक��मलाइल��(ICBMs) व�बमवष�क��हेतु�आव�यक�6,000 से�अ�धक


आ��वक�ह�थयार��क��तैनाती�करने�से���तबं�धत�करती�है।�

· �टाट� �I सं�ध�5 �दसंबर�2009 को�समा�त��ई।�

नई��टाट� �

· नई��टाट� �( ��े टे�जक�आ�स���रड�शन���ट��) सं�ध�अमे�रका�और��सी�संघ�के �बीच�आ��वक�ह�थयार��को�घटाने�क��सं�ध�है��जसे�औपचा�रक


�प�से�मे�ज़��फॉर�द�फद� र��रड�शन�एंड��ल�मटे शन�ऑफ़���े टे�जक�अफे ��सव�आ�स��है।�

· इसे�8 अ�ैल�2010 को��ाग�म��ह�ता��रत��कया�गया�था।�

· यह�2021 म��समा�त�होगी।�

सो�चए�!

��े टे�जक�अफे ��सव��रड�शन���ट��

Source: https://www.thehindu.com/opinion/editorial/rocky-summit/article24454560.ece

QUESTION 66.
‘ इंटरनेशनल�क�पै�रजन��ो�ाम�( International Comparison Programme)’ के �बारे�म���न�न�ल�खत�कथन��पर��
वचार क��जए।�

1. यह��य�श���समता�( PPP ) का��योग�कर�सकल�घरेलु�उ�पाद�(GDP) और�इसके �अवयव��क��तुलना�का�समथ�न�करता�है।�

2. यह�संयु��रा��सां��यक��आयोग�के �त�वाधान�के �अंतग�त��व��ब�क��ारा�संचा�लत��कया�जाता�है।�

सही�कथन�का�चुनाव�क��जए:

a) के वल�1
b) के वल�2
c) 1 और�2 दोन��
d) न�तो�1 न�ही�2
Correct Answer: C

AnswerExplanation

Solution (c)

इंटरनेशनल�क�पै�रजन��ो�ाम�

(ICP)

· ICP एक�वै��क�सां��यक��पहल�है��जसका�नेतृ�व�सं
. रा. सां��यक��आयोग�के �त�वाधान�के �अंतग�त��व��ब�क�कर�रहा�है।�इसका��मुख
उ�े �य�इस��े��और�इसके �परे�के ��े���के �दे श��के �बीच�
PPPs
GDP और�इसके ��य�समु�चय��के �तुलनायो�य�मू�य�एवं�मा�ा�माप�उपल�ध�कराना�है।�
· PPPs व�तु��और�सेवा��क��उस�कु ल�मा�ा�को�मापती�है��जसे��कसी�एक�दे श�क��मु�ा�अ�य�दे श�म���य�कर�सकती�है।�
· अंतरा��ीय�, �े�ीय�, उप-�े�ीय�और�रा�ीय�एज��सय��के �साथ�भागीदारी�म�ICP
� , �व��अथ��व�था��क���य�श���समता�(पीपीपी) का
अनुमान�लगाने�और�उ�ह���का�शत�करने�हेतु�मू�य�डेटा�और�
IASbaba GDP �य��का�सं�ह�और�उनक��तुलना�करता�है।� Score:
Web:
Email:
Page 118 150
https://telegram.me/UPSCMaterials https://telegram.me/FreeUPSCMaterials https://telegram.me/MaterialforExam

2019 - Test 2-Geography


Exam Title :
& C...
Email
Contact :

· PPPs का��ापक��योग�GDP जैसे�रा�ीय�लेखा�डेटा�को�सामा�य�मु�ा�म��प�रव�त�त�करने�हेतु��कया�जाता�है�तथा�साथ�ही�दे श��के �बीच�के


मू�य�अंतर��को�हटा��दया�जाता�है।�

समाचार�: ICP क��50 व��वष�गाँठ�

QUESTION 67.
संचार�संगतता�और�सुर�ा�समझौते�( Communications Compatibility and Security

Agreement ) के �बारे�म���न�न�ल�खत�कथन��पर��वचार�क��जए।�

1. यह�संयु��रा�य�अमे�रका�के �बु�नयाद��समझौत��म��से�एक�है�जो�भारत�के �साथ���प�ीय�र�ा�सहयोग�बढाने�हेतु�मह�वपूण��है।�

2. यह�भारतीय�सेना�को�मह�वपूण�, सुर��त�और�कू टब�( encrypted ) र�ा��ौ�ो�गक���ा�त�करने�म��स�म�

बनाएगा।�सही�कथन�का�चुनाव�क��जए�

a) के वल�1
b) के वल�2
c) 1 और�2 दोन��
d) न�तो�1 न�ही�2
Correct Answer: C

Answer

Explanation

Solution (c)
(LEM
सं
OA)चार�सं गतता�और�सुर�ा�समझौता�( COMCASA ) सं. रा. अमे�रका�के �तीन�बु�नयाद��समझौत��म��से�एक�है�जो�र�ा��े��म��अ�य�दे श��
तथा�BECA
के
COMCASA भारतीय�सेना�हेतु�आधु�नक�, सुर��त�और�नेट-स�म�ह�थयार��णा�लयाँ�जैसे����सशन�आमा�म�ट�, एयर-टू -एयर��मसाइल��, अं
साथ�अमे �रका�क�े और�नौवहन��णा�लयाँ
त�र���णा�लयाँ �उ�च��ौ�ो�गक��सहयोग�को��नद� �शत�करता�है
�आ�द��ा�त�करने ।�अ�य�दो�समझौते
�क��दशा��का��नमा� ण�करे�गह�ा।�ये
�लो�ज��ट�स�ए�सच�
�लड़ाकू ज�मेमोर�डम�ऑफ़�ए�ीम�ट�
��वमान��और�मानव�र�हत��वमान��जै से
�लेटफॉ�स��हेतु�मह�वपूण��अवयव�ह�।�

अमे�रका�का�एक�‘ �मुख�र�ा�सहयोगी�’ होने�के �नाते�भारत�के ��लए�यह�आव�यक�है��क�इन�तीन�बु�नयाद��समझौत��पर�ह�ता�र�करे�जो


मह�वपूण���ौ�ो�गक�य��और�गोपनीय�सूचना��के �सुचा��सरलीकरण�के �म�य�अ�धक�अ�तरसं��यता�क��अनुम�त�दे ते�ह�।�

अब�तक�, भारत�ने�तीन�म��से�के वल�समझौते�पर�ह�ता�र��कए�ह���जसका�नाम�लो�ज��ट�स�ए�सच�ज�मेमोर�डम�ऑफ़�ए�ीम�(LEMOA)


ट� है
।�यह�समझौता��धन�के �पुनभ�रण�के �उ�े �य��से�एक
-�सरे�के �सै�य���त�ान��तक�प�ँच�उपल�ध�कराता�है।�

Source: https://www.thehindu.com/news/national/india-going-ahead-with-purchase-of-
s-400-air-defence-systems-from-russia-nirmala-sitharaman/article24413391.ece

QUESTION 68.
वोटर�वे�रफाईड�पेपर�ऑ�डट��ायल�(VVPAT) के �बारे�म���न�न�ल�खत�कथन��पर��वचार�क��जए।�

1. यह�मतदाता��को�यह�स�या�पत�करने�म��स�म�बनाता�है��क�उनका�वोट�उनके �इ��छत���याशी�को��मला�है।�

2. यह�इले��ॉ�नक�वो�ट�ग�मशीन��(EVMs) को�पूण�त: ��त�था�पत�कर�दे गा।�

सही�कथन�का�चुनाव�क��जए�

a) के वल�1

IASbaba
Score:
Web:
Email:
Page 119 150
https://telegram.me/UPSCMaterials https://telegram.me/FreeUPSCMaterials https://telegram.me/MaterialforExam

2019 - Test 2-Geography


Exam Title :
& C...
Email
Contact :

b) के वल�2
c) 1 और�2 दोन��
d) न�तो�1 न�ही�2
Correct Answer: A

AnswerExplanation

Solution (a)

VVPAT �या�है�?

· एक�वोटर�वे�रफाईड�पेपर�ऑ�डट��ायल�(VVPAT) इकाई�EVMs का��योग�करने�वाले�मतदाता��को���तपु���उपल�ध�करवाती�है।�

· VVPAT , EVMs के ��लए�एक��वतं��स�यापन��णाली�के �तौर�पर�काय��करती�है�और�मतदाता��को�यह�स�या�पत�करने�म��स�म�बनाती


है��क�उनक��वोट�उनके �इ��छत���याशी�को�गई�है।�

काय��णाली�

· जब�मतदाता�EVM इकाई�पर�अपने�मनपसंद���याशी�के �नाम�के �सामने�वाला�बटन�दबाता�है�


, तब�VVPAT इकाई�एक�पेपर���लप�उ�प�
करती�है��जसे�‘ बैलट���लप�’ भी�कहा�जाता�है।�

· इस�पेपर���लप�म��चुने�गए���याशी�का�नाम�
, �म�सं. और�उसका�चुनाव��च��होता�है।�

· मतदाता�इसे�एक�����ड��व�डो�के �मा�यम�से�दे ख�सकता�है�


, जहाँ�यह�सात�सेकं ड�के ��लए�ठहरती�है�और�त�प�ात��वत : कटकर�एक�सीलबंद
�ॉप�बॉ�स�म���गर�जाती�है।�इस��कार�, यह�बैलट���लप�न�तो�मतदाता�के �हाथ�म��जाती�है�और�न�ही�कोई�अ�य�इसे�दे ख�पाता�है।�

· बैलट���लप�, �ल�भतम�से�भी��ल�भ�मामल��म��के वल�मतदान�अ�धका�रय���ारा��ा�त�क��जा�सकती�है।�

· यह�मतदाता�को�पहली�बार�यह�अवसर�दे ती�है��क�वह�पेपर��रसी�ट�के �आधार�पर�अपने�मत�को�चुनौती�दे �सके ।�

�या�आप�जानते�ह��?

· भारत�के �चुनाव�आयोग�को�सं�वधान�के �अनु�छे द�


324 से�329 �ारा�त�समय��भावी��व�ध�के �अनुसार�चुनाव��के �संचालन�हेतु�अधी�ण�
, �न
द� शन�और��नयं�ण��दान�करने�तथा��कसी��कार�अटकल��अथवा�बहस��को�पो�षत�न�करने�का�अ�धदे श��ा�त�है।�

सो�चए�!

· नोटा( NOTA )

· फ�ट� -पा�ट-द-पो�ट�(FPTP) चुनावी��णाली�


म�टे �यू-चे�सफोड����ताव�संबं�धत�थे-
· एकल�गैर-ह�ता�तरणीय�मत�
a) सामा�जक�सुधार��से�
Source: http://pib.nic.in/PressReleseDetail.aspx?PRID=1540055
b) संवैधा�नक�सुधार��से�
c) �श�ा�संबंध69.
QUESTION ी�सुधार��से�
d) �ेस�क���वतं�ता�पर�बंधन�आरो�पत�करने�से�
Correct Answer: B
Answer

IASbaba
Score:
Web:
Email:
Page 120 150

Вам также может понравиться